UPSC OCTOBER - 2020 DAILY CURRENT AFFAIRS CLASS TEST – 01 1. Consider the following pairs: above 80 years of age and people with Name of the Port Country disabilities, the Election Commission has 1. Sitwe Port - Myanmar come out with a set of new instructions. 2. Chabahar Port - Iran • The form required to opt for the postal 3. Gwadar Port - Pakistan ballot would be delivered at the residence 4. Hambantota Port - Sri Lanka of all those above 80 years of age and Which of the pairs given above are correct? people with disabilities by the booth level (a) 1, 2 and 3 only officer under his polling station. (b) 2, 3 and 4 only • It would be up to these two category of (c) 1, 3 and 4 only voters to opt for postal ballot. (d) All the above • Others who can avail the facility are as Answer: d) follows: Explanation: ✓ Members of the armed forces like the • All the above pairs are correctly matched. Army, Navy and Air Force, members of the armed police force of a state 2. Who among the following are eligible for (serving outside the state), government Postal Ballot System? employees posted outside India and 1. Members of the armed forces their spouses are entitled to vote only by 2. Spouses of government employees posted post. outside India ✓ In other words, they can’t vote in 3. Citizens above 80 years of age person. Voters under preventive 4. Persons under Preventive Detention detention can also vote only by post. 5. Persons under Punitive Detention ✓ Special voters such as the President of Select the correct answers using the code given India, Vice President, Governors, below: Union Cabinet ministers, Speaker of the (a) 1, 2 and 3 only House and government officers on poll (b) 2, 3 and 4 only duty have the option to vote by post. (c) 1, 2, 3 and 4 only ✓ But they have to apply through a (d) All the above prescribed form to avail this facility. Answer: c)

Explanation:

• Seeking to make the procedure to opt for

postal ballot more convenient for those

www.iasgatewayy.com  Daily Current Affairs | Monthly 044- Magazines | Online Tests 1 | P a g e

26265326/9884421666/9884472636 UPSC OCTOBER - 2020 DAILY CURRENT AFFAIRS CLASS TEST – 01 3. Which of the following is/are not the • The 91st Amendment also barred the safeguards for the anti-defection law? appointment of defectors as Ministers until 1. The Anti-Defection Law provides a their disqualification period is over or they safeguard for defections made on genuine are re-elected, whichever is earlier. ideological differences. • But, obviously, such laws have not put to 2. It accepts split within a party if at least one- rest the trend of defections. third of the members of the legislative party defect. 4. Which of the following could be the reason/s 3. It also allows the formation of a new party for Current Account Deficit? or merger with other political party if not 1. Overvalued exchange rate less than two-thirds of the party’s members 2. Increase in exports commit to it. 3. Long periods of consumer-led Economic Select the correct answer using the code given growth below: 4. High inflation a) 2 only Choose the correct option: b) 2 and 3 only (a) 1, 3 and 4 only c) 3 only (b) 2, 3 and 4 only d) 1, 2 and 3 (c) 2 only Answer: d (d) 1 and 4 only Explanation: Answer: a) • The Anti-Defection Law provided a Explanation: safeguard for defections made on genuine • The current account deficit is a ideological differences. measurement of a country’s trade where the • It accepted “split” within a party if at least value of the goods and services it imports one-third of the members of the legislative exceeds the value of its exports. party defect, and allowed the formation of • If the currency is overvalued, imports will a new party or “merger” with other political be cheaper, and therefore there will be a party if not less than two-thirds of the higher quantity of imports resulting in a party’s members commit to it. Current Account Deficit. • The 91st Constitutional Amendment • One of the reasons for the Current Account introduced in 2003 deleted the provision Surplus is an increase in exports. allowing split.

www.iasgatewayy.com  Daily Current Affairs | Monthly 044- Magazines | Online Tests 2 | P a g e

26265326/9884421666/9884472636 UPSC OCTOBER - 2020 DAILY CURRENT AFFAIRS CLASS TEST – 01 • A period of consumer-led economic • Brahmos is named after the rivers growth will cause deterioration in the Brahmaputra of India and Moskva of current account. Higher consumer Russia. spending will lead to higher spending on • BrahMos is a two-stage missile with a solid imports. The recession of 2009 also led to propellant booster engine as its first stage a temporary improvement in the deficit as which brings it to supersonic speed and consumers cut back on spending. then gets separated. The liquid ramjet or the • If a country’s inflation rises faster than its second stage then takes the missile closer to main competitors then it will make the 3 Mach speed in cruise phase. exports less competitive and imports more • It can be launched from land, air, and sea. competitive for that country. This will lead 6. Consider the following statements with respect to deterioration in the current account.

to Copernicus Programme: 5. Consider the following statements with respect 1. It is an Earth observation programme. to BrahMos: 2. The programme is co-ordinated and 1. It is a supersonic cruise missile developed managed by National Aeronautics and by the Defence Research and Development Space Administration (NASA). Organisation of India (DRDO) and Russia. 3. It was previously known as Global 2. It is a two-stage missile with a solid Monitoring for Environment and Security propellant engine in the first stage and a programme (GMES). liquid ramjet in the second. Which of the given statement/s is/are correct? 3. It can be launched from land, air, and sea. (a) 1 only Which of the given statement/s is/are correct? (b) 1 and 3 only (a) 1 only (c) 1, 2 and 3 only (b) 1 and 3 only (d) 1 and 2 only (c) 1, 2 and 3 Answer: b) (d) 1 and 2 only Explanation: Answer: c) • Copernicus is an ambitious Earth Explanation: observation programme. • BrahMos is a joint venture between the • It is coordinated and managed by the Defence Research and Development European Commission in partnership with Organisation of India (DRDO) and the the European Space Agency, the EU NPOM of Russia. Member States and EU Agencies. www.iasgatewayy.com  Daily Current Affairs | Monthly 044- Magazines | Online Tests 3 | P a g e

26265326/9884421666/9884472636 UPSC OCTOBER - 2020 DAILY CURRENT AFFAIRS CLASS TEST – 01 • Copernicus is the new name for the Global • The Alliance intends to advocate for Monitoring for Environment and Security freedom of religion or belief for all, which programme, previously known as GMES. includes the right of individuals to hold any belief or none, to change religion or belief 7. Which of the following country has recently and to manifest religion or belief, either launched the International Religious Freedom alone or in community with others, in Alliance? worship, observance, practice and (a) USA teaching. (b) India • The Alliance is intended to bring together (c) Russia senior government representatives to (d) China discuss actions their nations can take Answer: a) together to promote respect for freedom of Explanation: religion or belief and protect members of International Religious Freedom Alliance: religious minority groups worldwide.

• The United States has recently announced the launch of a 27-nation International 8. Which of the following have been given the Religious Freedom Alliance, which will try Geographical Indication Tag? to adopt a collective approach in protecting 1. Sawantwadi toys and preserving religious freedom across the 2. Channapatna toys world. 3. Etikoppaka toys • Among the prominent countries to join the 4. Kondapalli toys alliance are Austria, Brazil, the United 5. Kinhal toys Kingdom, Israel, Ukraine, the Netherlands Choose the correct option: and Greece. (a) 1, 2, 3 and 4 only • The Alliance is a network of likeminded (b) 2, 3, 4 and 5 only countries fully committed to advancing (c) 2, 3 and 5 only freedom of religion or belief around the (d) 2 and 4 only world. Answer: b) • The Alliance is predicated on the idea more Explanation: must be done to protect members of • Channapatna toys – Karnataka, Etikoppa religious minority groups and combat Toys and Kondapalli Bommalu (Toys) – discrimination and persecution based on Andhra Pradesh have been given the religion or belief. Geographical Indication Tag. www.iasgatewayy.com  Daily Current Affairs | Monthly 044- Magazines | Online Tests 4 | P a g e

26265326/9884421666/9884472636 UPSC OCTOBER - 2020 DAILY CURRENT AFFAIRS CLASS TEST – 01 • Sawantwadi toys refer to handmade works winter and spring were warmer than usual of art made of wood in Sawantwadi, a town during 2019-20. in Sindhudurg district of Maharashtra. • Temperature in Siberia in 2020 had gone • To mark the occasion of 151 years of through the roof, with the region recording postcards in India, Maharashtra Circle of a severe heatwave. India Post will release picture postcards on • Nearly all of this year’s fires inside the ‘Sawantwadi Toys’. The postcard is Arctic Circle occurred on continuous inspired by the Prime Minister’s speech on permafrost, with over half of these burning the country’s ability to become a “toy hub”. on ancient carbon-rich peat soils. • The tundra is drying up and vegetation 9. With reference to Zombie Fire, consider the there like moss, grass, dwarf shrubs, etc are following statements: starting to catch fire. These fires in the 1. It is a peat fire which persists from year to Arctic spreading to areas which were year inside the Arctic Circle. formerly fire-resistant is a more worrying 2. It occurs only in the regions with Tundra feature. However, they are not limited to vegetation. regions with Tundra vegetation.

Which of the statements given above is/are correct? 10. With reference to Index of Industrial (a) 1 only Production (IIP), Consider the following (b) 2 only statements: (c) Both 1 and 2 1. It is compiled and published quarterly by (d) Neither 1 nor 2 the National Statistical Office (NSO). Answer: a) 2. It is a composite indicator of three broad Explanation: sectors, namely Mining, Manufacturing, • Zombie Fire is a fire from a previous and Electricity. growing season that can smoulder under 3. Base Year for IIP calculation is 2014-2015. the ground which is made up of carbon-rich Which of the statements given above is/are peat. When the weather warms, the fire can correct? reignite. These are also known as holdover (a) 1 and 2 only fires. (b) 2 only • The reason for Zombie fires is that, due to (c) 2 and 3 only the climate change the temperatures in (d) 1, 2 and 3 Answer: b) www.iasgatewayy.com  Daily Current Affairs | Monthly 044- Magazines | Online Tests 5 | P a g e

26265326/9884421666/9884472636 UPSC OCTOBER - 2020 DAILY CURRENT AFFAIRS CLASS TEST – 01 Explanation: • The Index of Industrial Production (IIP) is an index that shows the growth rates in different industry groups of the economy in a fixed period of time. • It is compiled and published monthly by the National Statistical Office (NSO), Ministry of Statistics and Programme Implementation. • IIP is a composite indicator that measures the growth rate of industry groups classified under: • Broad sectors, namely, Mining, Manufacturing, and Electricity. Hence, statement 2 is correct. • Base Year for IIP calculation is 2011-2012. • The eight core sector industries included in IIP are coal, crude oil, natural gas, refinery products, fertilisers, steel, cement and electricity. • These eight industries account for 40.27% in the Index of Industrial Production (IIP).

www.iasgatewayy.com  Daily Current Affairs | Monthly 044- Magazines | Online Tests 6 | P a g e

26265326/9884421666/9884472636 UPSC OCTOBER - 2020 DAILY CURRENT AFFAIRS CLASS TEST – 05 1. Consider the following statements: • However, a moderate level of inflation is 1. Inflation measures the average price required in the economy to ensure that change in a basket of commodities and production is promoted. So, statement 3 is services over time. correct. 2. Inflation is indicative of the decrease in the • In India, the NSO under the Ministry of purchasing power of a unit of a country’s Statistics and Programme Implementation currency which would ultimately lead to a measures inflation. deceleration in economic growth. • In India, inflation is primarily measured by 3. A moderate level of inflation is required in two main indices — WPI (Wholesale Price the economy to ensure that production is Index) and CPI (Consumer Price Index) promoted. which measure wholesale and retail-level Which of the statements given above are price changes, respectively. correct? (a) 1 and 2 only 2. If a particular group raises their concerns or (b) 2 and 3 only grievance against a law made by the (c) 1 and 3 only government, then they are protected under (d) 1, 2 and 3 which article of the Indian constitution? Answer: c) (a) Article 14 Explanation: (b) Article 15 (c) Article 18 • Inflation refers to the rise in the prices of (d) Article 19 most goods and services of daily or Answer: d) common use, such as food, clothing, Explanation: housing, recreation, transport, consumer staples, etc. • The right to protest, to publicly question and force the government to answer, is a • Inflation measures the average price fundamental political right of the people change in a basket of commodities and that flows directly from a democratic services over time. So, statement 1 is reading of Article 19. correct. • The courts have continuously reiterated • Inflation is indicative of the decrease in the that the right to protest is a fundamental purchasing power of a unit of a country’s right. currency. This could ultimately lead to a deceleration in economic growth. So, • One may examine and find that the word Statement 2 is correct. ‘protest’ is missing in the Indian www.iasgatewayy.com  Daily Current Affairs | Monthly 044- Magazines | Online Tests 1 | P a g e

26265326/9884421666/9884472636 UPSC OCTOBER - 2020 DAILY CURRENT AFFAIRS CLASS TEST – 05 Constitution. But Article 19 has mentioned when provided by a registered medical it implicitly. practitioner at a registered medical facility. • For example, the right to free speech and • As per the provisions of the MTP Act, only expression may be taken to mean that the consent of woman whose pregnancy is everyone has a right to express their being terminated is required. So, statement personal opinion on, say, a film, or on the 1 is correct. In case of below the age of 18 condition of the city they inhabit. years, or a mentally ill woman, consent of • The right to associate to mean the right to guardian (MTP Act defines guardian as form self-regulating clubs, professional someone who has the care of the minor. associations or societies. This does not imply that only parent/s are • The right of peaceful assembly to mean the required to consent) is required for right to have a picnic in a park or to termination. So, statement 2 is correct. participate in religious festivals. • Medical Termination of Pregnancy (MTP) Bill, 2020 was passed recently in the Lok 3. Consider the following statements: Sabha.

1. As per the provisions of the MTP Act, only

the consent of Woman whose Pregnancy is 4. Consider the following statements with respect being Terminated is Required. to Beijing Declaration: 2. In case of below the age of 18 years, or a 1. It is an agenda for women’s empowerment mentally ill woman, consent of guardian is adopted by the UN. required for Termination. 2. The Beijing Declaration was adopted at the Which of the statement(s) given above is/are end of the 2nd World Conference on correct? Women (WCW). (a) 1 only 3. It set strategic objectives and actions for the (b) 2 only achievement of gender equality in 12 (c) Both 1 and 2 Critical Areas of concern. (d) Neither 1 nor 2 Answer: c) Which of the given statement/s is/are correct? Explanation: (a) 1 and 3 only • The Medical Termination of Pregnancy (b) 2 only (MTP) Act 1971 is a law that legalized (c) 2 and 3 only abortion in India up to 20 weeks of (d) 1, 2 and 3 pregnancy, based on certain conditions and Answer: a)

www.iasgatewayy.com  Daily Current Affairs | Monthly 044- Magazines | Online Tests 2 | P a g e

26265326/9884421666/9884472636 UPSC OCTOBER - 2020 DAILY CURRENT AFFAIRS CLASS TEST – 05 Explanation: 3. The body is constituted under the • The Beijing Declaration is an agenda for provisions of the Environment (Protection) women’s empowerment and is considered Act, 1986. a key global policy document on gender Which of the given statement/s is/are correct? equality, adopted by the UN. (a) 1 and 2 only • The United Nations has organized 4 world (b) 3 only conferences on women: Mexico (1975), (c) 1, 2 and 3 Copenhagen (1980), Nairobi (1985) and (d) None of the above Beijing (1995). Answer: b) • The Beijing Declaration was adopted Explanation: unanimously by the UN at the end of the • Environment Pollution Control Authority 4th World Conference on Women (WCW) is a Supreme Court-mandated body tasked held in Beijing. with taking various measures to tackle air • The Beijing Declaration and Platform for pollution in the Delhi-NCR (National Action identified 12 critical areas of Capital Region). concern and states strategic interventions to • The body is constituted under the deal with these concerns provisions of the Environment (Protection) • Twenty-five years have passed since the Act, 1986. Beijing Declaration and Platform for • It is mandated to enforce the Graded Action was ratified. Response Action Plan (GRAP) in the Delhi-NCR (National Capital Region). 5. Consider the following statements with respect • The System of Air Quality and Weather to the Environment Pollution Control Forecasting And Research (SAFAR) is a Authority (EPCA): national initiative introduced by the 1. EPCA is a Supreme Court-mandated body Ministry of Earth Sciences (MoES) to tasked with taking various measures to measure the air quality of a metropolitan tackle air pollution in all the metropolitan city, by measuring the overall pollution cities across India. level and the location-specific air quality of 2. System of Air Quality and Weather the city. Forecasting And Research (SAFAR) is a national initiative introduced by EPCA.

www.iasgatewayy.com  Daily Current Affairs | Monthly 044- Magazines | Online Tests 3 | P a g e

26265326/9884421666/9884472636 UPSC OCTOBER - 2020 DAILY CURRENT AFFAIRS CLASS TEST – 05 6. Consider the following statements: (c) Both 1 and 2 1. The regulatory and administrative (d) Neither 1 nor 2 jurisdiction of Minor Minerals falls under Answer: b) the Purview of State Governments. Explanation: 2. River Sand is considered a minor mineral. • Exercise Bongosagar, whose first edition 3. Power to notify the minor minerals lies was held in 2019, is aimed at developing with the Central Government. inter-operability and joint operational skills Which of the given statement/s is/are correct? through the conduct of a wide spectrum of (a) 2 and 3 only maritime exercises and operations. (b) 1 and 2 only • It is a Bilateral Exercise between the Indian (c) 1 only Navy (IN) – Bangladesh Navy (BN). (d) 1, 2 and 3 8. With reference to the ‘Ambedkar Social Answer: d) Innovation and Incubation Mission (ASIIM)’, Explanation: consider the following statements: • The Power to frame policy and legislation 1. It is launched under Venture Capital Fund on the minor minerals are entirely for Scheduled Castes (SCs). subjected and delegated to the State 2. It aims to promote entrepreneurship among Governments while policy and legislation the SC Youth with special preference to relating to the major minerals are dealt with Divyangs. by the Ministry of Mines under the Central Which of the following statements is/ are Government of India. correct? • Power to notify the minor minerals lies

with the Central Government. (a) 1 only • River sand is considered a Minor Mineral. (b) 2 only

7. Consider the following statements with respect (c) Both 1 and 2 to ‘Bongosagar’: (d) Neither 1 nor 2 1. It is a trilateral Naval Exercise between Answer: c) India, Bangladesh and Myanmar. Explanation: 2. Its first edition was held in 2019. • The Ministry of Social Justice & Which of the given statement/s is/are correct? Empowerment has launched the (a) 1 only ‘Ambedkar Social Innovation and (b) 2 only Incubation Mission (ASIIM)’ under www.iasgatewayy.com  Daily Current Affairs | Monthly 044- Magazines | Online Tests 4 | P a g e

26265326/9884421666/9884472636 UPSC OCTOBER - 2020 DAILY CURRENT AFFAIRS CLASS TEST – 05 Venture Capital Fund for Scheduled Castes • Amal is set to reach Mars in February 2021, (SCs) with a view to promoting innovation the year UAE will celebrate 50 years of its and enterprise among SC students studying formation. in higher educational institutions. • UAE has also set a goal to build a human Major Objectives of the Mission: colony on Mars by 2117. • To promote entrepreneurship among the • In 2019, the UAE sent its first astronaut to SC Youth with special preference to the International Space Station. Divyangs. • The ‘Hope Orbiter’ will be lifted on an H- • To support (1,000) innovative ideas till IIA rocket from Mitsubishi Heavy 2024 through a synergetic work with the Industries, a machinery maker in Japan. It Technology Business Incubators (TBIs) set will be launched from Tanegashima Island up by the Department of Science and in Japan. Technology.

9. With reference to the ‘Hope mission’, consider the following statements: 1. It is the first interplanetary mission from Arab world. 2. It was launched from the Kennedy Space Center. Which of the statements given above is/are correct? (a) 1 only (b) 2 only (c) Both 1 and 2 (d) Neither 1 nor 2 Answer: a) Explanation: UAE’s Space Mission: • In July 2020, the UAE launched a Mars probe named Amal (Hope) from Japan, marking the Arab world’s first interplanetary mission. www.iasgatewayy.com  Daily Current Affairs | Monthly 044- Magazines | Online Tests 5 | P a g e

26265326/9884421666/9884472636 UPSC OCTOBER - 2020 DAILY CURRENT AFFAIRS CLASS TEST – 06 1. “Madhuca Diplostemon” recently in News is: Choose the correct option: (a) A species of plant commonly known as (a) 1, 2, 4, 3 mangrove apple, an indicator of (b) 2, 1, 3, 4 environmental changes in mangrove (c) 1, 4, 2, 3 ecosystem. (d) 1, 3, 2, 3 (b) A threatened species of tree in the Western Answer: a) Ghats rediscovered after a gap of about Explanation: 180 years. • North to south: Marshall Island, Soloman (c) A Bacteria that causes peptic ulcer. Island, Vanuatu, New Caledonia. (d) A is a semi-aquatic frog indigenous to Western Ghats that actively breeds in the 3. Consider the following statements: monsoon. 1. Sun’s magnetic field reverses its polarity Answer: b) roughly every 11 years. Explanation: 2. A plage is a bright region in the • The tree “Madhuca Diplostemon” which chromosphere of the Sun typically found in was long believed to be extinct has been regions of the chromosphere near sunspots. rediscovered after a gap of more than 180 3. Solar filaments are large regions of very years from a sacred grove in Kerala. dense, cool gas, held in place by magnetic • The tree Madhuca Diplostemon belongs to fields. the Sapotaceae family. Which of the given statement/s is/are • It is a threatened species of the Western Incorrect? Ghats whose specimen was first collected (a) 1 only in 1835. (b) 2 and 3 only • The specimens were never collected again (c) 3 only and botanical explorations in the Eastern (d) None of the above and Western Ghats failed to locate the Answer: d) species. Explanation: • The magnetic field of the Sun is unique. 2. Arrange the following from North to South: Unlike many other celestial bodies Sun’s 1. Marshall Island magnetic field reverses its polarity roughly 2. Soloman Island every 11 years. 3. New Caledonia 4. Vanuatu www.iasgatewayy.com  Daily Current Affairs | Monthly 044- Magazines | Online Tests 1 | P a g e

26265326/9884421666/9884472636 UPSC OCTOBER - 2020 DAILY CURRENT AFFAIRS CLASS TEST – 06 • A plage is a bright region in the • Tungro: Caused by the combination of two chromosphere of the Sun typically found in viruses, which are transmitted by regions of the chromosphere near sunspots. leafhoppers. • Plages provide information about magnetic • Bakanae: Seedborne fungal disease. field locations and strength. • Blast (node and neck): Caused by the • Solar filaments are large regions of very fungus Magnaporthe oryzae. dense, cool gas, held in place by magnetic • Xanthomonas oryzae pv. oryzae (Xoo) fields. causes a serious bacterial leaf blight disease • They are formed along magnetic neutral in rice. It is also known as Bacterial blight. lines. Their presence represents the 5. In context to the Compensatory Afforestation magnetic field distribution beneath Fund (CAF), consider the following

4. Which of the following diseases affect the Statements: cultivation of rice crops? 1. The fund is collected under the Forest 1. Tungro Conservation Act, 1980. 2. Bakanae 2. The CAF is shared between Centre and 3. Sheath blight States. Select the correct answer using the codes given Which of the statements given above is/are below: correct? (a) 1 and 2 only (a) 1 only (b) 2 and 3 only (b) 2 only (c) 1 and 3 only (c) Both 1 and 2 (d) 1, 2 and 3 (d) Neither 1 nor 2 Answer: d) Answer: b) Explanation: Explanation: • Disease in rice is mainly caused by • The Compensatory Afforestation Fund Act bacteria, viruses, or fungi. Few major was passed by the centre in 2016 and the diseases are: related rules were notified in 2018. • Sheath Blight: Caused by fungus • The CAF Act was enacted to manage the Rhizoctonia solani. funds collected for compensatory • Brown Spot: One of the most common and afforestation which till then was managed most damaging fungal diseases. by Ad hoc Compensatory Afforestation Fund Management and Planning Authority www.iasgatewayy.com  Daily Current Affairs | Monthly 044- Magazines | Online Tests 2 | P a g e

26265326/9884421666/9884472636 UPSC OCTOBER - 2020 DAILY CURRENT AFFAIRS CLASS TEST – 06 (CAMPA). Hence, statement 1 is not • Nuclear triad is the capability of launching correct. nuclear weapons from land, sea and air- • According to Compensatory Afforestation based assets. Management and Planning Authority 7. Consider the following statements: (CAMPA) rules, 90% of the CAF money is 1. GST Council is a constitutional body for to be given to the states while 10% is to be making recommendations to the Union and retained by the Centre. State Government on issues related to 6. Consider the following statements, with Goods and Services Tax. reference to the K Family of Missiles: 2. The GST Council is chaired by the Union 1. It is primarily a Submarine Launched Finance Minister and other members are Ballistic Missiles (SLBMs). the Union State Minister of Revenue or 2. It is transferred from Russia under the Finance and Ministers in-charge of Finance Missile Technology Control regime. or Taxation of all the States. Which of the statements given above is/are not 3. The decisions made by the council are out correct? of consensus only. (a) 1 only Which of the statements given above are (b) 2 only correct? (c) Both 1 and 2 (a) 1 and 2 only (d) Neither 1 nor 2 (b) 2 and 3 only Answer: a) (c) 1 and 3 only Explanation: (d) 1, 2 and 3 • The K family of missiles are primarily Answer: a) Submarine Launched Ballistic Missiles Explanation: (SLBMs). Hence, statement 1 is correct. • It is a constitutional body (Article 279A) • These have been indigenously developed for making recommendations to the Union by Defence Research and Development and State Government on issues related to Organisation (DRDO). Hence, statement 2 Goods and Services Tax. So, statement 1 is not correct. is correct. • The development of these missiles began in • The GST Council is chaired by the Union the late 1990s as a step towards completing Finance Minister and other members are India’s nuclear triad. the Union State Minister of Revenue or Finance and Ministers in-charge of Finance www.iasgatewayy.com  Daily Current Affairs | Monthly 044- Magazines | Online Tests 3 | P a g e

26265326/9884421666/9884472636 UPSC OCTOBER - 2020 DAILY CURRENT AFFAIRS CLASS TEST – 06 or Taxation of all the States. So, statement 9. With respect to the National Commission for 2 is correct. Women, consider the following statements: • It is considered as a federal body where 1. It is a statutory body set up in 1992 under both the centre and the states get due the National Commission for Women Act, representation. 1990. • As per The Constitution (One Hundred and 2. The Commission must consist of a First Amendment) Act, 2016, in case of a minimum number of members which voting, every decision of the GST Council includes a chairperson, a member has to be taken by a majority of not less secretary, and other five members. than three-fourths of the weighted votes of 3. It enjoys all the powers of a civil and the members present. criminal court. • The vote of the central government has a Which of the statement(s) given above is/are weightage of one-third of the total votes correct? cast, and the votes of all the state (a) 1 only governments taken together have a (b) 1 and 2 only weightage of two-thirds of the total votes (c) 2 and 3 only cast in that meeting. So, statement 3 is not (d) 1, 2 and 3 correct. Answer: b) Explanation: 8. KV Kamath Committee, sometimes seen in • The National Commission for Women is a news is related to statutory body set up in 1992 under the (a) Restructure the MSME criteria National Commission for Women Act, (b) COVID-19 related debt restructuring 1990. This body was established to review (c) FRBM Rules the constitutional and legal safeguards for (d) Scrutinizing the applications for new bank women. So, statement 1 is correct. licenses • It recommends the remedial legislative Answer: b) measures, facilitates redressal of Explanation: grievances and advises the government on • The Supreme Court recently asked the all policy matters affecting women. It Centre and the Reserve Bank of India (RBI) enjoys all the powers of a civil court, not to place on record the KV Kamath criminal court. So, statement 3 is not committee's recommendations on COVID- correct. 19 related debt restructuring. www.iasgatewayy.com  Daily Current Affairs | Monthly 044- Magazines | Online Tests 4 | P a g e

26265326/9884421666/9884472636 UPSC OCTOBER - 2020 DAILY CURRENT AFFAIRS CLASS TEST – 06 • The Commission must consist of a 3. In case of a difference of opinion between minimum number of members which the LG and his Ministers on any matter, the includes a chairperson, a member Administrator is bound to refer it to the secretary, and other five members. So, President for a decision. statement 2 is correct. Which of the statements given above are • The Chairperson and the five members are correct? nominated by the central government from (a) 1 and 2 only amongst the person of ability, integrity, and (b) 2 and 3 only standing. They should possess experience (c) 1 and 3 only in various fields like law or legislation, (d) 1, 2 and 3 trade unionism, management of industry Answer: d) potential of women, women’s voluntary Explanation: organization, education, administration, • The Government of Union Territories Act, economic development and social good- 1963 provides for a Legislative Assembly being. of Pondicherry (as Puducherry was then • The Central Government also nominates called), with a Council of Ministers to member secretary. He/ she should be either govern the “Union Territory of an expert in the field of management, Pondicherry”. organization or an officer who is a member. • The same Act says that the UT will be administered by the President of India 10. With reference to the powers of the Lieutenant through an Administrator (LG). Governor of the Union Territory, consider the ✓ Section 44 of the Act, says the Council of following statements: Ministers headed by a Chief Minister will 1. The Council of Ministers headed by a Chief “aid and advise the Administrator in the Minister will aid and advise the exercise of his functions in relation to Administrator in the exercise of his matters with respect to which the functions in relation to matters which the Legislative Assembly of the Union Union Territory has power to make laws. Territory has power to make laws”. 2. It also allows the LG to act in his discretion ✓ The same clause also allows the LG to “act in the matter of lawmaking, even though in his discretion” in the matter of the Council of Ministers has the task of lawmaking, even though the Council of aiding and advising him. Ministers has the task of aiding and

www.iasgatewayy.com  Daily Current Affairs | Monthly 044- Magazines | Online Tests 5 | P a g e

26265326/9884421666/9884472636 UPSC OCTOBER - 2020 DAILY CURRENT AFFAIRS CLASS TEST – 06 advising him. So, both statement 1 and 2 are correct. • In case of a difference of opinion between the LG and his Ministers on any matter, the Administrator is bound to refer it to the President for a decision and act according to the decision given by the President. • However, the Administrator can also claim that the matter is urgent and take immediate action as he deems necessary. So, statement 3 is also correct.

www.iasgatewayy.com  Daily Current Affairs | Monthly 044- Magazines | Online Tests 6 | P a g e

26265326/9884421666/9884472636 UPSC OCTOBER - 2020 DAILY CURRENT AFFAIRS CLASS TEST – 07 1. Consider the following statements with respect 2. It oversees Insolvency Professional to Nagorno-Karabakh: Agencies, Insolvency Professionals and 1. It is a mountainous territory bordered by Information Utilities in India. the Caspian Sea in the east. 3. The members of IBBI include 2. It is internationally recognized as part of representatives from the Ministries of Armenia. Finance, Law and Corporate Affairs, and 3. Lake Sevan is located within Nagorno – the Reserve Bank of India. Karabakh. Which of the given statement/s is/are Which of the given statement/s is/are Incorrect? Incorrect? (a) 1 only (a) 1 only (b) 2 and 3 only (b) 1 and 2 only (c) 1, 2 and 3 (c) 1, 2 and 3 (d) None of the above (d) None of the above Answer: a) Answer: c) Explanation: Explanation: • Insolvency and Bankruptcy Board of India • Nagorno-Karabakh, a landlocked (IBBI) is the regulator for overseeing mountainous territory has been at the centre insolvency proceedings and entities like of the clashes between Armenia and Insolvency Professional Agencies (IPA), Azerbaijan. Insolvency Professionals (IP) and • It is internationally recognized as part of Information Utilities (IU) in India. Azerbaijan. • It was established in 2016 and given • Lake Sevan is located in Armenia and is not statutory powers through the Insolvency within Nagorno •Karabakh. and Bankruptcy Code. • The members of IBBI include 2. Consider the following statements with respect representatives from the Ministries of to Insolvency and Bankruptcy Board of India Finance, Law and Corporate Affairs, and (IBBI): the Reserve Bank of India. 1. Established under the Reserve Bank of India Act of 1934, it is the regulator for overseeing Insolvency Proceedings.

www.iasgatewayy.com  Daily Current Affairs | Monthly 044- Magazines | Online Tests 1 | P a g e

26265326/9884421666/9884472636 UPSC OCTOBER - 2020 DAILY CURRENT AFFAIRS CLASS TEST – 07 3. Consider the following statements: (c) 1 and 3 only 1. India has set a target of 450 GW of (d) 1, 2 and 3 renewable capacity by 2030, comprising Answer: a) 100 GW of solar capacity. Explanation: 2. India is third among the countries with the • Americans Harvey J Alter and Charles M largest solar capacity in the world. Rice, and British scientist Michael Which of the given statement/s is/are correct? Houghton were awarded the Nobel Prize (a) 1 only for Medicine or Physiology, 2020 for the (b) 2 only discovery of the Virus. Hence, (c) Both 1 and 2 statement 1 is correct. (d) Neither 1 nor 2 • In 2018 the National Viral Hepatitis Answer: b) Control Programme (NVHCP) was Explanation: launched which has the target to eliminate • India has set a target of 450 GW of Hepatitis C by 2030. The program is the renewable capacity by 2030, comprising largest program for and C 300 GW of solar capacity. diagnosis and treatment in the world. • India is third among the countries with the Hence, statement 2 is correct. largest solar capacity in the world. • A for the disease has still not been developed but it can be treated with the 4. Consider the following statements: help of anti-viral drugs. 1. The Nobel Prize for Physiology in 2020 has • Hepatitis B is included under India's been awarded for the discovery of the Universal Programme (UIP) . which provides free of cost 2. The National Viral Hepatitis Control against a total of 12 vaccine-preventable Programme (NVHCP) was launched to diseases. Under UIP free of cost eliminate Hepatitis C by 2030 in India. vaccination is provided against twelve 3. Hepatitis C vaccine is included under vaccine-preventable diseases i.e. India's Universal Immunization Tuberculosis, Diphtheria, Pertussis, Programme (UIP). Tetanus, Polio, Hepatitis B, Pneumonia and Select the correct answer using the codes given Meningitis due below: to Haemophilus Influenzae type b (Hib), (a) 1 and 2 only Measles, Rubella, Japanese Encephalitis (b) 2 and 3 only (JE) and Rotavirus diarrhoea. www.iasgatewayy.com  Daily Current Affairs | Monthly 044- Magazines | Online Tests 2 | P a g e

26265326/9884421666/9884472636 UPSC OCTOBER - 2020 DAILY CURRENT AFFAIRS CLASS TEST – 07 5. In context to the World Habitat Day, consider 6. Consider the following statements, with the following statements: reference to the Lokpal and Lokayukta Act, 1. The World Habitat Day 2020 is hosted by 2013: the Habitat International Coalition and the 1. The jurisdiction of Lokpal does not include Government of Indonesia. the Prime Minister. 2. First World Habitat Day was celebrated in 2. The term of office fixed for Lokayukta is of 1986 in Kenya. 5 years duration or 65 years of age, Which of the statements given above is/are whichever is earlier. correct? Which of the statements given above is/are (a) 1 only Incorrect? (b) 2 only (a) 1 only (c) Both 1 and 2 (b) 2 only (d) Neither 1 nor 2 (c) Both 1 and 2 Answer: b) (d) Neither 1 nor 2 Explanation: Answer: c) • The United Nations has designated the first Explanation: Monday of October every year as World • The jurisdiction of Lokpal includes the Habitat Day. In 2020, the day is co-hosted Prime Minister, Ministers, Members of by UN-Habitat and the Government of Parliament and Groups A, B, C and D Indonesia. UN-Habitat is a United Nations officers and officials of the Central agency for Urban development that Government. Hence, statement 1 is not promotes sustainable human settlements. correct. Hence, statement 1 is not correct. • The Lokpal and Lokayukta Act, 2013 • World Habitat Day was first celebrated in provides freedom to the states to decide 1986 in Nairobi, Kenya with the theme upon the contours of Lokayukta “Shelter is My Right”. Hence, statement 2 mechanism in their respective states. is correct. • Thus, it has not fixed the term of office for • Habitat International Coalition (HIC) is an Lokayukta independent, nonprofit alliance with 7. Consider the following statements regarding hundreds of organizations and individuals, the Godavari Cauvery Interlinking Project: which has been working in housing and 1. Water will be diverted from Godavari human settlements for more than 30 years. River to Nagarjuna Sagar dam and further

www.iasgatewayy.com  Daily Current Affairs | Monthly 044- Magazines | Online Tests 3 | P a g e

26265326/9884421666/9884472636 UPSC OCTOBER - 2020 DAILY CURRENT AFFAIRS CLASS TEST – 07 south to meet the demands of Krishna, ✓ the Krishna (Nagarjunasagar) – Pennar Pennar and Cauvery basins. (Somasila) and 2. The Godavari basin extends over states of ✓ the Pennar (Somasila)–Cauvery. Maharashtra, Andhra Pradesh, • The project will provide irrigation Chhattisgarh and Odisha in addition to facilities to 3.45 to 5.04 lakh hectares in smaller parts in Madhya Pradesh, Prakasam, Nellore, Krishna, Guntur, and Karnataka and Puducherry. Chittoor districts of Andhra Pradesh. 3. All the four rivers Cauvery, Godavari, • The Godavari basin extends over states of Krishna and Pennar drain into Bay of Maharashtra, Andhra Pradesh, Bengal. Chhattisgarh and Odisha in addition to Which of the statement(s) given above is/are smaller parts in Madhya Pradesh, correct? Karnataka and Yanam (Union territory of (a) 1 only Puducherry in East Godavari district of (b) 1 and 2 only Andhra Pradesh).So, statement 2 is (c) 1 and 3 only correct. (d) 1, 2 and 3 only • All the four rivers Cauvery, Godavari, Answer: d) Krishna and Pennar drain into Bay of Explanation: Bengal. So, statement 3 is correct. • The project envisages the diversion of 247 8. When a Minority Educational Institution is thousand million cubic feet (tmcft) of being regulated or framed guidelines that is unutilised water in the Indravati sub-basin against their fundamental right, then which of of the Godavari basin to meet the the following right is violated? requirements between the Godavari and the (a) Article 30 Cauvery rivers. So, statement 1 is correct. (b) Article 27 • Water will be diverted from Godavari (c) Article 21 River to Nagarjuna Sagar dam (through (d) Article 32 lifting) and further south to meet the Answer: a) demands of Krishna, Pennar and Cauvery Explanation: basins. The Godavari - Cauvery link • The Supreme Court recently held that the comprises three components namely, State is well within its rights to introduce a ✓ the Godavari (Inchampalli/Janampet) - regulatory regime in the “national interest” Krishna (Nagarjunasagar), to provide minority educational institutions www.iasgatewayy.com  Daily Current Affairs | Monthly 044- Magazines | Online Tests 4 | P a g e

26265326/9884421666/9884472636 UPSC OCTOBER - 2020 DAILY CURRENT AFFAIRS CLASS TEST – 07 with well-qualified teachers in order for ✓ Blue Notice – Additional Information them to “achieve excellence in education.” ✓ Purple Notice – Modus Operandi • The managements of minority institutions (someone’s mode of working) cannot ignore such a legal regime by saying ✓ Black Notice – Unidentified Bodies that it is their fundamental right under ✓ Interpol UN Security Special Notice – Article 30 of the Constitution to establish Groups and Individuals subject to and administer their educational UNSC Sanctions institutions according to their choice. • Interpol also issued a Purple Notice to seek • In a significant judgment, the Supreme or provide information on modus operandi, Court has also ruled that the National objects, devices and concealment methods Eligibility cum Entrance Test (NEET) for used by criminals on Ransomware attacks admission to graduate and postgraduate on health care institutions. medical and dental courses does not violate the rights of minorities under the 10. Consider the following statements: Constitution. 1. Phishing is the method of trying to gather personal information using deceptive e- 9. Recently Interpol issued ‘Purple Notice’ to mails and websites. many of its member countries during the wake 2. Malware is a type of malicious software of COVID-19 pandemic. Purple Notice designed to block access to a computer denotes which of the following? system until a sum of money is paid. (a) Imminent Threat Which of the statement(s) given above is/are (b) Wanted Persons correct? (c) Modus Operandi (a) 1 only (d) Warnings and Intelligence (b) 2 only Answer: c) (c) Both 1 and 2 Explanation: (d) Neither 1 nor 2 • There are various notices issued by the Answer: a) Interpol. They are as follows: Explanation: ✓ Red Notice – Wanted Persons • Phishing is the method of trying to gather ✓ Green Notice – Warnings and personal information using deceptive e- Intelligence mails and websites. So, statement 1 is ✓ Yellow Notice – Missing Persons correct. ✓ Orange Notice – Imminent Threat www.iasgatewayy.com  Daily Current Affairs | Monthly 044- Magazines | Online Tests 5 | P a g e

26265326/9884421666/9884472636 UPSC OCTOBER - 2020 DAILY CURRENT AFFAIRS CLASS TEST – 07 • Malicious software or malware can be described as unwanted software that is installed in your system without your consent. • Ransomware is a type of malicious software designed to block access to a computer system until a sum of money is paid. So, statement 1 is not correct.

www.iasgatewayy.com  Daily Current Affairs | Monthly 044- Magazines | Online Tests 6 | P a g e

26265326/9884421666/9884472636 UPSC OCTOBER - 2020 DAILY CURRENT AFFAIRS CLASS TEST – 08 1. Consider the following statements: 2. Basic Exchange and Cooperation 1. India follows the policy of No First Use of Agreement for Geo-Spatial Cooperation nuclear weapons against the countries with (BECA) nuclear weapons. 3. Logistics Exchange Memorandum of 2. Fissile Material Cut-off Treaty prohibits Agreement (LEMOA) the manufacturing and movement of 4. General Security of Military Information nuclear weapons. Agreement (GSOMIA) Which of the statements given above is/are Select the correct answer using the codes given correct? below: (a) 1 only (a) 1, 2 and 3 only (b) 2 only (b) 2, 3 and 4 only (c) Both 1 and 2 (c) 1, 3 and 4 only (d) Neither 1 nor 2 (d) 1, 2, 3 and 4 Answer: a) Answer: d) Explanation: Explanation: • India remains committed to the policy of • Logistics Exchange Memorandum of No First Use (NFU) against nuclear Agreement (LEMOA): It allows both weapon states and non-use against non- countries to have access to each other’s nuclear-weapon states. In 2019, there were designated military facilities for refueling hints of gradual shifts to circumstantial and replenishment. This is a logistics usage; however, there have been no official support agreement signed in 2016. revisions in the principle. • General Security of Military Information • Fissile Material Cut-off Treaty is a Agreement: GSOMIA allows militaries to proposed international agreement that share the intelligence gathered by them.It prohibits the production of two main was signed in 2002 between India and components of nuclear weapons: highly- USA. enriched Uranium and Plutonium. • Communications and Information Security Memorandum of Agreement: Valid for 10 2. Which of the following International years, COMCASA aims to provide a legal agreements is/ are signed between India and framework for the transfer of highly the US? sensitive communication security 1. Communications and Information Security equipment from the USA to India that will Memorandum of Agreement (COMCASA) www.iasgatewayy.com  Daily Current Affairs | Monthly 044- Magazines | Online Tests 1 | P a g e

26265326/9884421666/9884472636 UPSC OCTOBER - 2020 DAILY CURRENT AFFAIRS CLASS TEST – 08 streamline and facilitate interoperability Explanation: between their armed forces. It is the most • The Nobel Prize in Physics for the year recent agreement signed in 2018. 2020 was awarded to three astrophysicists • Industrial Security Annex (ISA): It is an Roger Penrose from the UK, Reinhard extension to the GSOMIA, was signed at Genzel from Germany, and Andrea Ghez the 2+2 dialogue in 2019. ISA provides a from the USA. framework for the exchange and protection • Roger Penrose received half of this year’s of classified military information between prize for the discovery that a black hole the USA and Indian defence industries. formation is a robust prediction of the • BECA: The USA is looking forward to general theory of relativity. India signing the Basic Exchange and • The term ‘black hole’ was coined in the Cooperation Agreement for Geo-Spatial mid-1960s by American Physicist John Cooperation (BECA), at the next India- Archibald Wheeler. USA 2+2 ministerial dialogue likely to be • Black hole refers to a point in space where held in October 2020. BECA is the last of matter is so compressed as to create a the four foundational agreements. It is yet gravity field from which even light cannot to be signed between the two nations. escape. • Black-holes were theorized by Albert 3. Consider the following statements: Einstein in 1915. 1. The Nobel Prize in Physics for the year

2020 has been awarded for the discoveries Tag: Agriculture related to black holes. 4. Consider the following statements: 2. The term ‘black hole’ was coined in the 1. Agriculture falls within the legislative mid-1960s by American Physicist John competence of State governments. Archibald Wheeler. 2. The Commission for Agricultural Costs Which of the statements given above is/are and Prices (CACP) announces MSP for correct? various crops based on the (a) 1 only recommendations of the Cabinet (b) 2 only Committee of Economic Affairs. (c) Both 1 and 2 3. Minimum Support Price (MSP) has no (d) Neither 1 nor 2 legislative backing. Answer: c) Which of the given statement/s is/are Incorrect? www.iasgatewayy.com  Daily Current Affairs | Monthly 044- Magazines | Online Tests 2 | P a g e

26265326/9884421666/9884472636 UPSC OCTOBER - 2020 DAILY CURRENT AFFAIRS CLASS TEST – 08 (a) 2 only (c) 2 and 3 only (b) 1 only (d) 3 only (c) 1 and 3 only Answer: c) (d) None of the above Explanation: Answer: a) • SMART is a missile-assisted release of Explanation: lightweight anti-submarine torpedo system • The Cabinet Committee of Economic for anti-submarine warfare (ASW) Affairs announces MSP for various crops at operations far beyond torpedo range. the beginning of each sowing season based Heavy-weight torpedoes are significantly on the recommendations of the larger in diameter. Commission for Agricultural Costs and • It is an addition to Anti-Submarine Warfare Prices (CACP). (ASW) capabilities of India. • Minimum Support Price (MSP) has no • It is an indigenously developed system by legislative backing. laboratories of Defence Research and • Agriculture falls within the legislative Development Organisation (DRDO). competence of State governments, through Entry 14 of the State List as mentioned in 6. Consider the following statements with respect schedule 7 of the Indian constitution to Hepatitis C: 1. Hepatitis C virus belongs to the same 5. Consider the following statements with respect Family as Zika Virus. to Supersonic Missile Assisted Release of 2. It causes Inflammation of the Liver. Torpedo (SMART): 3. It is a positive-sense single-stranded RNA 1. It is a missile-assisted release of Virus. heavyweight anti-submarine torpedo Which of the given statement/s is/are correct? system. (a) 1 and 2 only 2. It is an addition to Anti-Submarine Warfare (b) 2 and 3 only (ASW) capabilities of India. (c) 1 and 3 only 3. It is an indigenously developed system by (d) 1, 2 and 3 laboratories of Defence Research and Answer: d) Development Organisation (DRDO). Explanation: Which of the given statement/s is/are correct? • Both Hepatitis C and Zika Virus belong to (a) 1 only the family Flaviviridae. (b) 1 and 2 only www.iasgatewayy.com  Daily Current Affairs | Monthly 044- Magazines | Online Tests 3 | P a g e

26265326/9884421666/9884472636 UPSC OCTOBER - 2020 DAILY CURRENT AFFAIRS CLASS TEST – 08 • The Flaviviridae are a family of positive, rather than authoritatively dismissing it is a single-stranded, enveloped RNA viruses. more harmonious family. • Hepatitis C is a viral that causes • The Right to Dissent is an important liver inflammation, sometimes leading to addition of expressing one’s view, and on serious liver damage. The hepatitis C virus many of its most important constituent (HCV) spreads through contaminated elements. Blood. • It will enable practitioners and citizens to claim their rights and participate more 7. Which of the following best describes the effectively in the project of democracy. democratic principle ‘Right to Dissent’? • Disagreeing with each other is a (a) A strong difference of opinion on a fundamental human trait. There is not a particular subject, especially about an single individual who does not disagree official suggestion or plan or a popular with something or the other all the time. belief.

(b) Protesting against the laws and regulations 8. Which of the following belongs to the Quad made by the government. grouping? (c) Mobilizing school and college students for (a) South Korea, India, United States and a common purpose. Australia (d) Overthrowing the current government and (b) Japan, India, United States and South people taking the rule of their nation Korea Answer: a) (c) Russia, India, United States and Australia Explanation: (d) Japan, India, United States and Australia • Dissent means “a strong difference of Answer: d) opinion on a particular subject, especially Explanation: about an official suggestion or plan or a • The quadrilateral security dialogue popular belief”. So, the answer is a. includes Japan, India, United States and • India’s constitutional democracy is Australia. predicated on the people’s right to call state • All four nations find a common ground of power to account. being the democratic nations and common • Even at Household level also, there is no interests of unhindered maritime trade and family without dissent between parents and security. the children, or between the siblings. A

family which learns to deal with dissent www.iasgatewayy.com  Daily Current Affairs | Monthly 044- Magazines | Online Tests 4 | P a g e

26265326/9884421666/9884472636 UPSC OCTOBER - 2020 DAILY CURRENT AFFAIRS CLASS TEST – 08 • The idea was first mooted by Japanese • Three-tier system of panchayats at village, Prime Minister Shinzo Abe in 2007. intermediate block/taluk/mandal and However, the idea couldn’t move ahead district levels except in States with with Australia pulling out of it. population is below 20 lakhs (Article 243B). So, statement 3 is correct. 9. Consider the following statements: • Seats at all levels to be filled by direct 1. The 73rd Constitutional Amendment added elections Article 243C (2). Part IX titled “The Panchayats” to the • Reservation of seats: Constitution. ✓ Seats reserved for Scheduled Castes 2. Panchayats have been authorised to prepare (SCs) and Scheduled Tribes (STs) and plans for economic development and social the chairpersons of the Panchayats at justice in respect of subjects illustrated in all levels also shall be reserved for Eleventh Schedule. SCs and STs in proportion to their 3. Every state should have three-tier systems population. of panchayats at village, intermediate ✓ One-third of the total number of seats block/taluk/mandal and district levels to be reserved for women. except in States with population are below ✓ One-third offices of chairpersons at all 20 lakhs. levels reserved for women (Article Which of the statements given above are 243D). correct? • Uniform five-year term and elections to (a) 1 and 2 only constitute new bodies to be completed (b) 2 and 3 only before the expiry of the term. In the event (c) 1 and 3 only of dissolution, elections compulsorily (d) 1, 2 and 3 within six months (Article 243E). Answer: d) • Independent Election Commission in each Explanation: State for superintendence, direction and • The 73rd Constitutional Amendment added control of the electoral rolls (Article 243K). Part IX titled “The Panchayats” to the • Panchayats have been authorised to prepare Constitution. So, statement 1 is correct. plans for economic development and social • Basic unit of democratic system-Gram justice in respect of subjects illustrated in Sabhas (villages) comprising all the adult Eleventh Schedule (Article 243G).So, members registered as voters. statement 2 is correct.

www.iasgatewayy.com  Daily Current Affairs | Monthly 044- Magazines | Online Tests 5 | P a g e

26265326/9884421666/9884472636 UPSC OCTOBER - 2020 DAILY CURRENT AFFAIRS CLASS TEST – 08 • Source of Revenue (Article 243H): State 10. Which among the following statements are legislature may authorise the Panchayats correct about the removal of the judge of a with Supreme Court of India? ✓ Budgetary allocation from State 1. The judge of the Supreme Court may be Revenue. removed from his current office by an order ✓ Share of revenue of certain taxes. of the President ✓ Collection and retention of the revenue 2. There are only two grounds of removal, it raises. incapacity or proved misbehaviour. • Establish a Finance Commission in each 3. So far, no judge of the Supreme Court has State to determine the principles on the been removed by impeachment. basis of which adequate financial resources 4. The procedure for the removal of the judges would be ensured for panchayats and is regulated by the Judges Enquiry Act municipalities (Article 243I). (1968) • The following areas have been exempted Select the correct answer using the code given from the operation of the Act because of the below: socio-cultural and administrative (a) 1 and 2 only considerations: (b) 1, 2 and 3 only ✓ Scheduled areas listed under the (c) 1, 2 and 4only Schedule V in the states of Andhra (d) All the above Pradesh, Bihar, Gujarat, Himachal Answer: d) Pradesh, Madhya Pradesh, Explanation: Maharashtra, Orissa and Rajasthan. • All the statements are factual regarding the ✓ The states of Nagaland, Meghalaya and removal of the Judge of the Supreme Court. Mizoram. • The hill areas of the district of Darjeeling in the state of West Bengal for which Darjeeling Gorkha Hill Council exists. However, an Act called the Provisions of Panchayats (Extension to the Scheduled Areas) Act, 1996 passed by the Government of India for the mentioned scheduled areas.

www.iasgatewayy.com  Daily Current Affairs | Monthly 044- Magazines | Online Tests 6 | P a g e

26265326/9884421666/9884472636 UPSC OCTOBER - 2020 DAILY CURRENT AFFAIRS CLASS TEST – 09 1. With reference to the CRISPR technology, • A DNA strand, when broken, has a natural consider the following statements: tendency to repair itself but the auto-repair 1. This technology can add, remove and alter mechanism can lead to the re-growth of a specific DNA sequences. problematic sequence. 2. Nobel Prize for Chemistry 2020 has been • Scientists intervene during this auto-repair awarded for discovering CRISPR/Cas9 process by supplying the desired sequence genetic scissors. of genetic codes, which replaces the 3. It is based on a natural defence mechanism Original Sequence. in a bacterium, Streptococcus pyogenes. 2. Which of the chemicals mentioned below are Which of the statements given above is/are banned under the Stockholm Convention? correct? 1. Furans (a) 1 and 3 only 2. Chlordane (b) 1 and 2 only 3. DDT (c) 2 and 3 only 4. Hexachlorobenzene (d) 1, 2 and 3 Select the correct answer using the codes given Answer: d) below: Explanation: (a) 1 and 2 only • Using components of the CRISPR system, (b) 2 and 3 only researchers can add, remove and alter (c) 3 and 4 only specific DNA sequences. Hence, statement (d) 1, 2, 3 and 4 1 is correct. Answer: d) • The 2020 Nobel Prize in Chemistry has Explanation: been awarded to Emmanuelle Charpentier • India ratified the Stockholm Convention in of France and Jennifer A Doudna of the 2006 as per Article 25(4), which enabled it USA for developing CRISPR/Cas9 genetic to keep itself in a default "opt-out" position scissors. Hence, statement 2 is correct. such that amendments in various Annexes • CRISPR technology replicates a natural of the convention cannot be enforced on it defence mechanism in Streptococcus unless an instrument of pyogenes (bacterium) that uses a similar ratification/acceptance/approval or method to protect itself from virus attacks. accession is explicitly deposited with UN Hence, statement 3 is correct. depositary.

www.iasgatewayy.com  Daily Current Affairs | Monthly 044- Magazines | Online Tests 1 | P a g e

26265326/9884421666/9884472636 UPSC OCTOBER - 2020 DAILY CURRENT AFFAIRS CLASS TEST – 09 • The convention calls to ban nine of the (c) 1 and 3 only dirty dozen chemicals (key POPs), limit the (d) All the above use of DDT to malaria control, and curtail Answer: d) inadvertent production of dioxins and Explanation: furans. • Hydrocarbon Exploration and Licensing • The Convention listed twelve distinct Policy (HELP) was approved by the chemicals in three categories: government in March 2016 replacing New • Eight pesticides (aldrin, chlordane, DDT, Exploration Licensing Policy (NELP). dieldrin, endrin, heptachlor, mirex and Hence, statement 1 is correct. toxaphene) • The new policy promises simpler rules, tax • Two industrial chemicals (poly chlorinated breaks, pricing and marketing freedom and biphenyls and hexachlorobenzene) is part of a government strategy to double • Two unintended by-products of many oil and gas output by 2022-23. industrial processes involving chlorine • HELP provides for a uniform licensing such as waste incineration, chemical and system that will cover all hydrocarbons pesticide manufacturing and pulp and such as oil, gas, and coal bed methane. paper bleaching (poly chlorinated dibenzo- Hence, statement 2 is correct. p-dioxins and dibenzofurans, commonly • HELP provides for the revenue sharing referred to as dioxins and furans). model, the government will receive a share of the gross revenue from the sale of oil, 3. Consider the following with regard to and gas, etc and will not be concerned with Hydrocarbon Exploration and Licensing the cost incurred. Policy (HELP): • HELP has marketing and pricing freedom. 1. It replaced New Exploration Licensing Before HELP, contracts were based on Policy (NELP). production sharing with possibility of gold 2. It has marketing and pricing freedom plating (incorporation of costly and 3. It provides for a uniform licensing system unnecessary features) the investment and that will cover all hydrocarbons such as oil, causing loss to the government by gas, and coal bed methane. ‘manipulating profit’. To reduce the Which of the statements given above are complexity of handling contracts, it was correct? changed to revenue sharing. (a) 1 and 2 only

(b) 2 and 3 only www.iasgatewayy.com  Daily Current Affairs | Monthly 044- Magazines | Online Tests 2 | P a g e

26265326/9884421666/9884472636 UPSC OCTOBER - 2020 DAILY CURRENT AFFAIRS CLASS TEST – 09 • Under HELP, oil companies can select having knowledge and experience in the blocks of their choice under this Open Area field of economics or banking or finance or Licensing (OAL) regime. Earlier it was the monetary policy.” government which selected the blocks • The previous three external members were where oil exploration can be carried out. It appointed for a term of 4 years. will enable a faster coverage of the • All the central government nominees are available Geographical Area. not eligible to be re-appointed

4. Which of the following statement/s is/are 5. Which of the following statement/s is/are correct with respect to the appointment of incorrect with respect to the Production Linked external members to the Monetary Policy Incentive (PLI) Scheme of the Ministry of Committee? Electronics and IT? 1. They are appointed for a term of Three 1. It is applicable only to the domestic firms years. and global firms are barred from applying 2. They are not eligible for re-appointment. under the scheme. Options: 2. The scheme shall extend an incentive of (a) 1 only 40% to 60% on incremental sales (over the (b) 2 only base year) of goods manufactured in India (c) Both 1 and 2 and covered under target segments, to (d) Neither 1 nor 2 eligible companies, for a period of 10 years Answer: b) subsequent to the base year as defined. Explanation: Options: • The Monetary Policy Committee is (a) 1 only responsible for fixing the benchmark (b) 2 only interest rate in India. (c) Both 1 and 2 • The committee comprises six members – (d) Neither 1 nor 2 three officials of the Reserve Bank of India Answer: c) and three external members nominated by Explanation: the Government of India. • The Ministry of Electronics and IT has • According to the RBI Act, external approved 16 proposals by electronics members of the MPC will be appointed by manufacturers under its Production Linked the central government “from amongst Incentive (PLI) Scheme. The approved persons of ability, integrity and standing, www.iasgatewayy.com  Daily Current Affairs | Monthly 044- Magazines | Online Tests 3 | P a g e

26265326/9884421666/9884472636 UPSC OCTOBER - 2020 DAILY CURRENT AFFAIRS CLASS TEST – 09 firms include five international mobile 7. Poverty and Shared Prosperity Report is phone makers and five domestic firms. released by • The scheme offers a production linked (a) World Bank incentive to boost domestic manufacturing (b) UNHRC and attract large investments in mobile (c) ILO phone manufacturing and specified (d) IMF electronic components, including Answer: a) Assembly, Testing, Marking and Explanation: Packaging (ATMP) units. • The World Bank in its biennial Poverty • The scheme shall extend an incentive of 4% and Shared Prosperity Report mentions to 6% on incremental sales (over base year) that COVID-19 can add around 27-40 of goods manufactured in India and million new poor in Sub-Saharan Africa covered under target segments, to eligible and around 49-57 million in South Asia companies, for a period of five years region. subsequent to the base year as defined. 8. The Union Cabinet has recently approved the

ratification of seven chemicals listed under 6. Which of the following is not a report Stockholm Convention on Persistent Organic published by the International Monetary Fund? Pollutants (POPs) to be banned in India. Which (a) World Economic Outlook of the following were among them? (b) Global Financial Stability Report 1. Chlordecone (c) Fiscal Monitor 2. Hexabromobiphenyl (d) Global Competitiveness Report 3. Pentachlorobenzene Answer: d) 4. Hexachlorobenzene Explanation: 5. Hexachlorobutadiene • While the World Economic Outlook, Select the correct answer using the code given Global Financial Stability Report, and below: Fiscal Monitor are flagship reports of the (a) 1, 2 and 3 only IMF, the Global Competitiveness Report is (b) 1, 2, 3 and 4 only published by the World Economic Forum. (c) 1, 2, 3 and 5 only

(d) All the above

Answer: c)

www.iasgatewayy.com  Daily Current Affairs | Monthly 044- Magazines | Online Tests 4 | P a g e

26265326/9884421666/9884472636 UPSC OCTOBER - 2020 DAILY CURRENT AFFAIRS CLASS TEST – 09 Explanation: • which enabled it to keep itself in a default • The Cabinet further delegated its powers to "opt-out" position such that amendments in ratify chemicals under the Stockholm various Annexes of the convention cannot Convention to Union Ministries of External be enforced on it unless an instrument of Affairs (MEA) and Environment, Forest ratification/ acceptance/ approval or and Climate Change (MEFCC) in respect accession is explicitly deposited with UN of POPs for streamlining the procedure. depositary. • Considering its commitment towards • The convention calls to ban nine of the providing a safe environment and dirty dozen chemicals (key POPs), limit the addressing human health risks, the Ministry use of DDT to malaria control, and curtail of Environment, Forest and Climate inadvertent production of dioxins and Change (MoEFCC) had notified the furans. The convention listed twelve 'Regulation of Persistent Organic distinct chemicals in three categories: Pollutants Rules, in 2018 under the ✓ Eight pesticides (aldrin, chlordane, DDT, provisions of Environment (Protection) dieldrin, endrin, heptachlor, mirex and Act, 1986. toxaphene) • The regulation inter alia prohibits the ✓ Two industrial chemicals (poly chlorinated manufacture, trade, use, import and export biphenyls and hexachlorobenzene). So, of seven chemicals, namely: answer is c. ✓ Chlordecone, ✓ Two unintended by-products of many ✓ Hexabromobiphenyl, industrial processes involving chlorine ✓ Hexabromodiphenyl ether and Hepta such as waste incineration, chemical and Bromodiphenyl Ether (Commercial octa- pesticide manufacturing and pulp and BDE), paper bleaching (poly chlorinated dibenzo- ✓ Tetrabromodiphenyl ether and p-dioxins and dibenzofurans, commonly Pentabromodiphenyl ether (Commercial referred to as dioxins and furans). penta-BDE), 9. The GEF is a financial mechanism for which ✓ Pentachlorobenzene, of the Major International Environmental ✓ Hexabromocyclododecane, and Conventions? ✓ Hexachlorobutadiene. 1. UNFCCC • India already ratified the Stockholm 2. UNCBD Convention in 2006 as per Article 25(4), 3. Stockholm Convention POPs www.iasgatewayy.com  Daily Current Affairs | Monthly 044- Magazines | Online Tests 5 | P a g e

26265326/9884421666/9884472636 UPSC OCTOBER - 2020 DAILY CURRENT AFFAIRS CLASS TEST – 09 4. UNCCD ✓ The Stockholm Convention on Persistent 5. The Minamata Convention on Mercury Organic Pollutants (POPs), Select the correct answer using the code given ✓ The United Nations Convention to Combat below: Desertification (UNCCD), and (a) 1, 2 and 3 only ✓ The Minamata Convention on Mercury. (b) 1, 2, 3 and 4 only (c) 1, 2, 3 and 5 only 10. ‘Power sharing Agreement’, was recently seen (d) All the above in news. It is related to which of the following Answer: d) country? Explanation: (a) Israel • GEF was established with the Rio Earth (b) Afghanistan Summit of 1992. (c) Pakistan • It is headquartered at Washington, D.C., (d) China USA. Answer: b) • The GEF is jointly managed by the United Explanation: Nations Development Programme • Recently, Afghanistan’s President Ashraf (UNDP), the World Bank, and the United Ghani and his political rival Abdullah Nations Environment Programme (UNEP). Abdullah have signed a power-sharing • The financial mechanism was established agreement. to help tackle our planet’s most pressing • The deal has been signed two months after environmental problems. the presidential election dispute that arose • It provides funds to the developing in September 2019 between Mr. Ghani and countries and transition economies for Mr. Abdullah. projects related to climate change, • The Election Commission declared Mr. biodiversity, the ozone layer, etc. Ghani to be the winner of the presidential • It is a financial mechanism for 5 major election, but Mr. Abdullah and the international environmental conventions: Elections Complaint Commission charged ✓ The United Nations Framework widespread voting irregularities in the Convention on Climate Change election. (UNFCCC), • Following which both declared them the ✓ The United Nations Convention on winner of the presidential election and has Biological Diversity (UNCBD), been locked in a power struggle since then.

www.iasgatewayy.com  Daily Current Affairs | Monthly 044- Magazines | Online Tests 6 | P a g e

26265326/9884421666/9884472636 UPSC OCTOBER - 2020 DAILY CURRENT AFFAIRS CLASS TEST – 09 • According to the deal, Mr. Ghani will remain as president while both Mr. Ghani and Mr. Abdullah will choose an equal number of ministers in the cabinet. • The deal calls for Abdullah to lead the country’s National Reconciliation High Council. • The Reconciliation Council has been given the authority to handle and approve all affairs related to Afghanistan’s peace process including the peace talks with the Taliban.

www.iasgatewayy.com  Daily Current Affairs | Monthly 044- Magazines | Online Tests 7 | P a g e

26265326/9884421666/9884472636 UPSC OCTOBER - 2020 DAILY CURRENT AFFAIRS CLASS TEST – 10 1. Consider the following statements with 2. Consider the following statements: reference to Cat Que virus: 1. Brahmos is the air-to-air and air-to-surface 1. Domestic pigs are considered to be the cruise missile. primary mammalian hosts of the virus. 2. Nirbhay subsonic missile has a range that 2. The virus infects economically important can reach up to 1000 km. livestock species. 3. Akash missile has the capacity to engage all 3. Humans are not infected by the virus. aerial targets including fighter planes. Which of the above statements is/are correct? Which of the above statements is/are correct? (a) 1 and 2 only (a) 1 and 2 only (b) 2 and 3 only (b) 2 and 3 only (c) 1 and 3 only (c) 1 and 3 only (d) All of the above (d) All of the above Answer: a) Answer: d) Explanation: Explanation: • CQV belongs to the Simbu serogroup and • All of the given statements are correct. infects both humans and economically important livestock species. 3. Consider the following statements with • It was first isolated in 2004from reference to Sonification Project of NASA: mosquitoes during the surveillance of 1. The project transforms data from arbovirus activity in northern Vietnam. Astronomical Audio into high-quality images. • For CQV, domestic pigs are considered to 2. It is led by the Chandra X-ray Center in be the primary mammalian hosts. collaboration with NASA’s Universe of • Antibodies against the virus have been Learning Program. reported in swine reared locally in China, Which of the above statements is/are correct? which indicates that the virus has formed a (a) 1 only natural cycle in the local area and has the (b) 2 only ability to spread in pigs and other animal (c) Both 1 and 2 populations through mosquitoes. (d) None of the above

Answer: b)

Explanation:

• NASA’s Chandra X-Ray Center (CXC)

has unveiled a new sonification project.

www.iasgatewayy.com  Daily Current Affairs | Monthly 044- Magazines | Online Tests 1 | P a g e

26265326/9884421666/9884472636 UPSC OCTOBER - 2020 DAILY CURRENT AFFAIRS CLASS TEST – 10 • The project transforms data from Explanation: astronomical images into audio. • ‘Jana Gana Mana’ is the first stanza of the • The Sonification Project of NASA Bengali hymns ‘Bharoto Bhagyo transforms data from astronomical images Bidhata’, written by Nobel Laureate into audio. Rabindranath Tagore. • Data sonification refers to the use of sound • A slightly varied version of the song was values to represent real data. Simply put, it adopted by Subhash Chandra Bose’s Indian is the auditory version of data National Army in 1941 as the national visualisation. anthem, called ‘Shubh Sukh Chain’, • For example, In NASA’s recent Chandra which also became popular in India. project, data is represented using a number • ‘Jana Gana Mana’ was adopted as the of musical notes. With this data country’s National Anthem by the sonification project, users can now hear Constituent Assembly of India on January the birth of a star, a cloud of dust or even a 24, 1950, the last day of its last session. black hole as a high- or low-pitched sound. • Reverence to the National Anthem is a Fundamental duty in India. According to 4. Consider the following statements regarding Article 51A (a) of the Constitution: “It the ‘National Anthem’: shall be the duty of every citizen of India to 1. The National Anthem was first sung at the abide by the Constitution and respect its Calcutta session of the Congress on ideals and institutions, the National Flag December 27, 1911. and the National Anthem”. 2. According to the constitution of India,

reverence to the National Anthem is a 5. What are the possible ways to recover the Fundamental duty in India. NPAs in India? Which of the statement(s) given above is/are 1. SARFAESI Act, 2002 correct? 2. Debt Recovery Tribunal (a) 1 only 3. Lok Adalat (b) 2 only 4. Compromise Settlement (c) Both 1 and 2 5. Credit information Bureau (d) Neither 1 nor 2 Select the correct answer using the code given Answer: c) below: (a) 1, 2 and 3 only (b) 2, 3 and 4 only www.iasgatewayy.com  Daily Current Affairs | Monthly 044- Magazines | Online Tests 2 | P a g e

26265326/9884421666/9884472636 UPSC OCTOBER - 2020 DAILY CURRENT AFFAIRS CLASS TEST – 10 (c) 1, 2, 3 and 5 only 3. Manage the borrower’s security or appoint (d) All the above someone to manage the same. Answer: d) 2. Debt Recovery Tribunals Explanation: • Brought into being existence in year 1993 1. SARFAESI ACT, 2002 by the Indian Parliament, the Act allows • The SARFAESI empowers banks to deal financial institutions to speedily recover with NPAs, without the involvement of dues of ₹10 lakhs and above. DRTs are court, through three alternatives: capable of handling larger number of cases 1. Asset Reconstruction as compared to regular courts by cutting 2. Enforcement of Security down delays in the initial proceedings. 3. Securitization 3. Lok Adalats • Any outstanding amount of more than ₹1 • Small loans of ₹5 lakhs and less can be lakh can be dealt under SARFAESI. recovered through the Lok Adalats as per However, an amount that is less than 20% the guidelines issued by RBI in year 2001. or principal and the interest amount is not This alternative for dispute redressal considered under the Act. The Act also mechanism covers both suit and non-suit allows banks to: filed cases. 1. To release a notice to borrower (and their 4. Compromise Settlement guarantor) asking them to release the • This scheme helps in recovery of NPAs up payment within 60 days from the receipt to ₹10 crores through a simplified non- of notice. discretionary mechanism. 2. To release notice to anyone who acquires 5. Credit Information Bureau the borrower’s secured assets to produce • Third party agencies such as CIBIL help the same to the bank. banks with data on the financial health of 3. To advice any of the borrower’s debtors to the borrower. pay off the loan due with the bank. • The Credit Information Bureau maintains • In case of failure from the borrower’s end records of individual defaulters and shares with respect to the issue notice, the bank it with the respective banks to aid them in may: making effective lending decisions. 1. Take possession of the secured assets of • For this, banks may be charged a fee. the borrower 2. Sell or lease the security

www.iasgatewayy.com  Daily Current Affairs | Monthly 044- Magazines | Online Tests 3 | P a g e

26265326/9884421666/9884472636 UPSC OCTOBER - 2020 DAILY CURRENT AFFAIRS CLASS TEST – 10 6. Consider the following: 7. With reference to the National Commission for 1. Saudi Arabia Safai Karamcharis, consider the following 2. Oman statements: 3. Qatar 1. It is the statutory body constituted by 4. Bahrain National Commission for Safai Which of the following countries are bordered Karamcharis Act, 1993. by U.A.E? 2. It recommends to the Government (a) 1 and 3 only regarding specific programmes for welfare (b) 2 and 4 only of Safai Karamcharis, study and evaluates (c) 1, 2 and 3 only the existing welfare programmes for Safai (d) All the above Karamcharis. Answer: c) Which of the statement(s) given above is/are Explanation: correct? • The UAE shares a border with three (a) 1 only countries: Saudi Arabia, Oman, and Qatar. (b) 2 only • Bahrain, officially the Kingdom of (c) Both 1 and 2 Bahrain, is a sovereign state in the Persian (d) Neither 1 nor 2 Gulf. Answer: b) • The island nation comprises a small Explanation: archipelago centered on Bahrain Island, • The National Commission for Safai situated between the Qatar peninsula and Karamcharis (NCSK) was constituted on the north eastern coast of Saudi Arabia. 12th August 1994 as a statutory body by an Act of Parliament viz. ‘National Commission for Safai Karamcharis Act, 1993’. • The act “The National Commission for Safai Karamcharis Act, 1993” lapsed in February 2004. So statement 1 on not correct. • The Commission is acting as a non- statutory body of the Ministry of Social

www.iasgatewayy.com  Daily Current Affairs | Monthly 044- Magazines | Online Tests 4 | P a g e

26265326/9884421666/9884472636 UPSC OCTOBER - 2020 DAILY CURRENT AFFAIRS CLASS TEST – 10 Justice and Empowerment whose tenure is Answer: a) extended from time to time through Explanation: Government Resolutions. • The World Bank releases the South Asia • It recommends to the Government Economic Focus report and Poverty and regarding specific programmes for welfare Shared Prosperity Report. The latest South of Safai Karamcharis, study and evaluates Asia Economic Focus report estimated that the existing welfare programmes for Safai India’s Gross Domestic Product (GDP) can Karamcharis; investigate cases of specific contract by 9.6% in 2020-21. grievances etc. • The latest Poverty and Shared Prosperity • Also, as per the provisions of the Report estimates that Covid-19 will push Prohibition of Employment as Manual over 1.4% of the world’s population into Scavengers and their Rehabilitation Act, extreme poverty. the NCSK has been assigned the work to • The Global Financial Stability Report is monitor the implementation of the Act, released by the International Monetary tender advice for its effective Fund. implementation to the Centre and State • The Global Gender Gap Report is released Governments and enquire into complaints by the World Economic Forum. regarding contravention/non- 9. With reference to the recently launched implementation of the provisions of the Earthshot Prize, consider the following Act.

statements: 8. Which of the following reports are released by 1. It aimed at funding the most innovative the World Bank? solutions to some of the world’s most 1. South Asia Economic Focus report pressing environmental challenges. 2. Poverty and Shared Prosperity Report 2. It is launched under the aegis of United 3. Global Financial Stability Report Nations Framework Convention on 4. Global Gender Gap Report Climate Change. Select the correct answer using the code given Which of the statements given above is/are below: correct? (a) 1 and 2 only (a) 1 only (b) 1, 2 and 3 only (b) 2 only (c) 1, 3 and 4 only (c) Both 1 and 2 (d) 1, 2, 3 and 4 (d) Neither 1 nor 2 www.iasgatewayy.com  Daily Current Affairs | Monthly 044- Magazines | Online Tests 5 | P a g e

26265326/9884421666/9884472636 UPSC OCTOBER - 2020 DAILY CURRENT AFFAIRS CLASS TEST – 10 Answer: a) solutions make a substantial contribution to Explanation: achieving the Earthshots. • The Earthshot Prize is the most prestigious global environment prize in history, 10. Which of the following factors is/are designed to incentivise change and help responsible for the warming of the Arctic? repair our planet over the next ten years. 1. Presence of Iodic acid • It is aimed at funding the most innovative 2. Higher albedo solutions to some of the world’s most 3. Changing ocean currents pressing environmental challenges. Hence, Choose the correct answer using code given statement 1 is correct. below: • The Earthshot Prize is centred around five (a) 1 and 2 only “Earthshots”, simple but ambitious goals (b) 1 and 3 only for the planet, which if achieved by 2030 (c) 2 and 3 only will improve life for all, for generations to (d) 1, 2 and 3 come. Answer: b) • Five Earthshots are: Protect and restore Explanation: nature; Clean our air; Revive our oceans; • A team of scientists have identified iodic Build a waste-free world and Fix our acid (HIO3) as a novel driver of new climate. aerosol particle formation in the Arctic • Britain’s Prince William has launched a which is responsible for Arctic new 50-million pound Earthshot Prize. The Amplification or Arctic Warming. prize is managed by the Prize Council, the Presence of Iodic acid in the region had not Earthshot Prize will be supported by its been observed previously. Global Alliance, a network of organisations • These aerosol particles influence the worldwide which share the ambition of the formation of clouds. As these clouds reflect Prize to repair the planet. Hence, statement solar radiation (known as Aerosol 2 is not correct. Prizes could be awarded to Radiative Forcing) but also retain heat on a wide range of individuals, teams or the Earth's surface, they have an influence collaborations – scientists, activists, on the warming of the Arctic. economists, community projects, leaders, • Ocean currents normally bring in warmer governments, banks, businesses, cities, and water from the Pacific, and colder water countries – anyone whose workable exits out of the Arctic into the Atlantic.

www.iasgatewayy.com  Daily Current Affairs | Monthly 044- Magazines | Online Tests 6 | P a g e

26265326/9884421666/9884472636 UPSC OCTOBER - 2020 DAILY CURRENT AFFAIRS CLASS TEST – 10 • But those currents may be changing because more melting ice is injecting the Arctic Ocean with freshwater. The missing ice also exposes the surface waters to more wind. This mixes up colder freshwater at the surface and warmer saltwater below, raising surface temperatures and further melting ice. • Change in Albedo: When bright and reflective ice (with more albedo) melts, it gives way to a darker ocean (lowering albedo); this amplifies the warming trend because the ocean surface absorbs more heat from the Sun than the surface of snow and ice. • A high albedo means the surface reflects the majority of the radiation that hits it and absorbs the rest. • A low albedo means a surface reflects a small amount of the incoming radiation and absorbs the rest. For instance, fresh snow reflects up to 95% of the Incoming Radiation.

www.iasgatewayy.com  Daily Current Affairs | Monthly 044- Magazines | Online Tests 7 | P a g e

26265326/9884421666/9884472636 UPSC OCTOBER - 2020 DAILY CURRENT AFFAIRS CLASS TEST – 12 1. Which of the following monetary policy • It is one of the main components of the measures can be taken by the Reserve Bank of RBI's monetary policy, which is used to India for infusion of liquidity in the market? regulate the money supply, level of 1. Increase in Cash Reserve Ratio (CRR) inflation and liquidity in the country. The requirement of commercial banks higher the CRR, the lower is the liquidity 2. Buying Government Securities from with the banks and vice-versa. Increasing commercial banks the CRR requirement will decrease 3. Long Term Reverse Repo Operation liquidity. (LTRO) • Long Term Reverse Repo Operation Select the correct answer using the codes given (LTRO): It is a mechanism to facilitate the below: transmission of monetary policy actions (a) 1 and 2 only and the flow of credit to the economy. This (b) 2 and 3 only helps in injecting liquidity in the system. (c) 1 and 3 only • Funds through LTRO are provided at the (d) 1, 2 and 3 repo rate. This means that banks can avail Answer: b) one year and three-year loans at the same Explanation: interest rate of one day repo. But usually, • Open Market Operations (OMO): OMOs loans with higher maturity period (here like are conducted by the RBI by way of sale or 1 year and 3 years) will have a higher purchase of government securities (g-secs) interest rate compared to short term (repo) to adjust money supply conditions. loans. • The central bank sells g-secs to commercial • Repo Rate: It is the rate at which the central banks to remove liquidity from the system bank of a country lends money to and buys back g-secs to infuse liquidity into commercial banks in the event of any the system. shortfall of funds. • Cash Reserve Ratio (CRR): Banks are • Low repo rates encourage banks to borrow required to hold a certain proportion of from the RBI, allowing them to lend more. their deposits in the form of cash. This This adds liquidity in the economy. minimum ratio (that is the part of the total deposits to be held as cash) is stipulated by the RBI and is known as the CRR.

www.iasgatewayy.com  Daily Current Affairs | Monthly 044- Magazines | Online Tests 1 | P a g e

26265326/9884421666/9884472636 UPSC OCTOBER - 2020 DAILY CURRENT AFFAIRS CLASS TEST – 12 2. Consider the following, with regard to missile • It can play a key role in neutralising any ‘Rudram-1’: jamming platforms of the enemy or take out 1. It is an air-to-surface missile, designed and radar stations thereby clearing a path for developed by the DRDO. own fighters to carry out an offensive and 2. It is the first indigenous anti-radiation also prevent own systems from being missile of the country. jammed. 3. It will enhance the Suppression of Enemy 3. In context to the World Food Programme Air Defence (SEAD) capability of Indian (WFP), consider the following statements: Air Force. 1. It is a non-governmental organization Which of the statements given above are independent of the United Nation. correct? 2. It focuses on emergency assistance as well (a) 1 and 2 only as rehabilitation and development aid. (b) 2 and 3 only 3. The 2020 Nobel Peace Prize has been (c) 1 and 3 only awarded to the World Food Programme. (d) 1, 2 and 3 Which of the statements given above are Answer: d) correct? Explanation: (a) 1 and 3 only • It is an air-to-surface missile, designed and (b) 2 and 3 only developed by the DRDO. It can be (c) 1 and 2 only launched from altitudes of 500 m to 15 km (d) 1, 2 and 3 and speeds of 0.6 to 2 mach. Hence, Answer: b) statement 1 is correct. Explanation: • It is the first indigenous anti-radiation • The World Food Programme (WFP) was missile of the country for Indian Air Force established in 1961 by the Food and (IAF). Once the missile locks on the target, Agriculture Organisation (FAO) and the it is capable of striking accurately even if United Nations General Assembly the radiation source switches off in (UNGA) and became a full-fledged UN between. Hence, statement 2 is correct. programme in 1965. Hence, statement 1 is • It will enhance IAF’s Suppression of not correct. Enemy Air Defense (SEAD) capability. • Its headquarter is in Rome, Italy.

• WFP focuses on emergency assistance as

well as rehabilitation and development aid. www.iasgatewayy.com  Daily Current Affairs | Monthly 044- Magazines | Online Tests 2 | P a g e

26265326/9884421666/9884472636 UPSC OCTOBER - 2020 DAILY CURRENT AFFAIRS CLASS TEST – 12 Two-thirds of its work is in conflict- • The report measures the economic freedom affected countries, where people are three or the ability of individuals to make their times more likely to be undernourished own economic decisions in a country, by than elsewhere. Hence, statement 2 is analysing policies and institutions of these correct. countries. • The 2020 Nobel Peace Prize has been • India slipped 26 places to 105 among 162 awarded to the World Food Programme countries and territories on the index of (WFP), a United Nations (UN) agency, for global economic freedom. its efforts to combat hunger, bettering • Indicators of the report: Regulation, the conditions for peace in conflict-affected freedom to trade internationally, size of areas and preventing the use of hunger as a government, property rights, government weapon of war and conflict. spending and taxation. • In India, the report was co-published by 4. Consider the following statements with Delhi-based Centre for Civil Society. reference to Global Economic Freedom Index • The report measures economic freedom 2020: (levels of personal choice, ability to enter 1. It is released by the Fraser Institute in markets, security of privately owned Canada. property and rule of law, among others 2. In 2020, India’s ranking in Global 5. Consider the following statements: Economic Freedom Index was higher than 1. Project 17A frigates is a design derivative the ranking in 2019. of the Shivalik class stealth frigates. 3. India has been ranked higher than China. 2. The Indian Navy plans to have all the Which of the above statements is/are correct? Project 17A class ships in service by 2022. (a) 1 and 2 only 3. INS Nilgiri is the first ship of Project 17A. (b) 2 and 3 only Which of the above statements is/are correct? (c) 1 and 3 only (a) 1 and 2 only (d) 1, 2 and 3 (b) 2 and 3 only Answer: c) (c) 1 and 3 only Explanation: (d) 1, 2 and 3 • Global Economic Freedom Index has Answer: c) recently released by the Fraser Institute in

Canada.

www.iasgatewayy.com  Daily Current Affairs | Monthly 044- Magazines | Online Tests 3 | P a g e

26265326/9884421666/9884472636 UPSC OCTOBER - 2020 DAILY CURRENT AFFAIRS CLASS TEST – 12 Explanation: • CSCAF initiative intends to inculcate a • The Project 17A-class frigate is a follow- climate-sensitive approach to urban on of the Project 17 Shivalik-class frigate planning and development in India. for the Indian Navy. • To provide a clear roadmap for cities • A total of seven ships will be built at towards combating Climate Change while Mazagon Dock and Garden Reach planning and implementing their actions, Shipbuilders & Engineers (company). including investments, Ministry launched • The Indian Navy plans to have all the seven this Framework in 2019, for 100 smart Project 17A class ships in service by 2025- cities. 27. The P-17A class frigates are being built • The framework was developed after review using indigenously developed steel and of existing frameworks and assessment fitted with weapons and sensors along with approaches adopted throughout the world. Integrated Platform Management System. • The framework has 28 indicators across • The Project 17-A involves an estimated five categories namely; cost of nearly Rs 50,000 crore. ✓ Energy and Green Buildings • The construction of the first ship started in ✓ Urban Planning, Green Cover & 2017 and the first ship is expected to Biodiversity delivered by 2022. It has been named ✓ Mobility and Air Quality ‘Nilgiri’. ✓ Water Management

✓ Waste Management 6. Which of the following union ministry has ✓ The Climate Centre for Cities under recently launched climate smart cities National Institute of Urban Affairs assessment framework 2.0? (NIUA) is supporting MoHUA in (a) Ministry of Jal Shakti implementation of CSCAF.

(b) Ministry of Environment, Forest and Climate Change Tag: Science & Technology (c) Ministry of Health and Family Welfare 7. ‘WHO’ has mentioned that one way to avoid (d) Ministry of Housing & Urban Affairs the SARS Cov-2 infection is by washing hands Answer: d) using soaps. In this context consider the Explanation: following statements: • Ministry of Housing & Urban Affairs has 1. All viruses including Corona Virus have launched the Climate Smart Cities the lipid envelop that helps the virus to Assessment Framework (CSCAF) 2.0. stick onto the skin. www.iasgatewayy.com  Daily Current Affairs | Monthly 044- Magazines | Online Tests 4 | P a g e

26265326/9884421666/9884472636 UPSC OCTOBER - 2020 DAILY CURRENT AFFAIRS CLASS TEST – 12 2. The oleophilic part of the soap molecule envelope and tends to have a ‘crowbar’ tends to have an affinity for and competes effect that breaks the lipid envelope of with the lipids in the virus envelope. the virus. 3. It also competes with the bond that binds ✓ The tail also competes with the bond the RNA and the lipid envelop thus that binds the RNA and the lipid dissolving the virus into its components envelop thus dissolving the virus into which are then removed by water. its components which are then removed Which of the statement(s) given above is/are by water. So, Statement 3 is correct. correct? • Certain viruses do not have the lipid (a) 1 only envelop and are called the non-enveloped (b) 1 and 2 only viruses. Rotavirus which causes severe (c) 2 and 3 only diarrhoea, poliovirus, and adenovirus that (d) 1 and 3 only cause pneumonia and even human Answer: c) papillomavirus (HPV) do not contain the Explanation: lipid envelop. So, Statement 1 is not • Using soap is more effective in removing correct. microbes on our hands. • The oil-loving tail of the soap molecule ✓ Viruses such as corona virus, influenza- also disrupts the bond that binds dirt and causing viruses, Ebola, Zika have their non-enveloped viruses to the hand. The dirt genetic material encased in a layer of fat and viruses are surrounded by several tails called the lipid envelop. making them remain as suspended ✓ Soap molecules are pin-shaped with a particles. Rinsing with water washes away head that is water-loving (hydrophilic) the suspended particles leading to clean and a tail that is oil-loving (oleophilic). hands. Being oleophilic, the tail portion of the molecule tends to have an affinity for 8. Consider the following statements regarding and ‘competes’ with the lipids in the the Blue Flag Tag for Beaches: virus envelope. So, Statement 2 is 1. The Blue Flag Programme for beaches and correct. marinas is run by the German Based ✓ Since the chemical bonds holding the international, non-governmental, non- virus together are not very strong, the profit organisation Greenwatch. long oleophilic tail gets inserted into the

www.iasgatewayy.com  Daily Current Affairs | Monthly 044- Magazines | Online Tests 5 | P a g e

26265326/9884421666/9884472636 UPSC OCTOBER - 2020 DAILY CURRENT AFFAIRS CLASS TEST – 12 2. Eight Beaches from India were recently 2. Constitution prescribes that; a member accorded Blue Flag Tag which includes chosen to fill a casual vacancy will serve Kasarkod and Padubidri beach. for the remainder of his predecessor’s term 3. Spain tops the list with 566 such beaches; of office. Greece and France follow with 515 and Which of the statement(s) given above is/are 395, respectively. correct? Which of the statement(s) given above is/are (a) 1 only correct? (b) 2 only (a) 1 and 2 only (c) Both 1 and 2 (b) 1 and 3 only (d) Neither 1 nor 2 (c) 2 and 3 only Answer: d) (d) 1, 2 and 3 Explanation: Answer: c) • Statement 1 is not correct because a Explanation: person of 25 years of age can contest in Lok • The Blue Flag Programme for beaches and Sabha Elections, but Rajya Sabha MPs marinas is run by the international, non- should have 30 years of age. governmental, non-profit organisation FEE • Statement 2 is not correct because (the Foundation for Environmental according to Section 154 of the Education). So, statement 1 is not correct. Representation of the People Act 1951, a • Eight Beaches from India were recently member chosen to fill a casual vacancy will accorded Blue Flag Tag which includes serve for the remainder of his predecessor’s Kasarkod and Padubidri beach. So, term of office. Constitution does not statement 2 is correct. prescribe such conditions. • Spain tops the list with 566 such beaches; Greece and France follow with 515 and 395, 10. With reference to the Indra Sawhney case, respectively. So, statement 3 is correct. which of the statement(s) given below is/are correct? 9. With reference to the members of the Rajya 1. The Court rejected the additional Sabha. Consider the following statements: reservation of 10% for poorer sections of 1. A member who is eligible to become the higher castes, but it upheld the member of the Lok Sabha can also become constitutional validity of 27% reservation a member of Rajya Sabha. for the OBCs with certain conditions.

www.iasgatewayy.com  Daily Current Affairs | Monthly 044- Magazines | Online Tests 6 | P a g e

26265326/9884421666/9884472636 UPSC OCTOBER - 2020 DAILY CURRENT AFFAIRS CLASS TEST – 12 2. No reservation in promotions; reservation ✓ The total reserved quota should not exceed should be confined to initial appointments 50% except in some extraordinary only. situations. This rule should be applied 3. The carry forward rule in case of unfilled every year. vacancies should not violate 50% rule. ✓ The ‘carry forward rule’ in case of unfilled Select the correct answer using the code given (backlog) vacancies is valid. But it should below: not violate 50% rule. (a) 1 only ✓ A permanent statutory body should be (b) 1 and 2 only established to examine complaints of over- (c) 2 and 3 only inclusion and under-inclusion in the list of (d) 1, 2 and 3 OBCs. Answer: d) • However, it is also to be noted that the Explanation: Parliament has passed the 124th • In the famous Mandal case (Indra Constitution Amendment Bill (10% Quota Sawhney v. Union of India 1992), the Bill) to provide for 10% reservation for scope and extent of Article 16(4), which economically weaker sections (EWS) provides for reservation of jobs in favour of among the general category candidates in backward classes, has been examined higher education and government thoroughly by the Supreme Court. employment in the year 2019. • Though the Court has rejected the • So, all the three statements are correct. additional reservation of 10% for poorer sections of higher castes, it upheld the constitutional validity of 27% reservation for the OBCs with certain conditions, viz, ✓ The advanced sections among the OBCs (the creamy layer) should be excluded from the list of beneficiaries of reservation. ✓ No reservation in promotions; reservation should be confined to initial appointments only. Any existing reservation in promotions can continue for five years only (i.e., up to 1997).

www.iasgatewayy.com  Daily Current Affairs | Monthly 044- Magazines | Online Tests 7 | P a g e

26265326/9884421666/9884472636 UPSC OCTOBER - 2020 DAILY CURRENT AFFAIRS CLASS TEST – 13 1. Consider the following statements, with • In the Atlantic, the Saffir-Simpson wind reference to Hurricane: scale is used to measure their destructive 1. It is an extratropical disturbance. power. 2. It moves counter clockwise in the Northern 2. In context to Central Information Hemisphere and clockwise in the Southern Commissioners, consider the following Hemisphere. statements: 3. The Saffir-Simpson wind scale is used to 1. They are appointed by the President on the measure the Power of Hurricanes. recommendation of the council of Which of the statements given above is/are ministers. correct? 2. They are not eligible for Reappointment. (a) 1 and 2 only Which of the statements given above is/are (b) 1 and 3 only correct? (c) 2 and 3 only (a) 1 only (d) 1, 2 and 3 (b) 2 only Answer: c) (c) Both 1 and 2 Explanation: (d) Neither 1 nor 2 • A hurricane starts out as a tropical Answer: b) disturbance. This is an area over warm Explanation: ocean waters where rain clouds are • Central Information Commissioners are building. Hence, statement 1 is not correct. appointed by the President on the • In the western North Pacific, they are called recommendation of a committee consisting "typhoons". In the Bay of Bengal and of the Prime Minister as Chairperson, the Arabian Sea, they are called "cyclones". Leader of Opposition in the Lok Sabha and • In Australia, they are called “Willy-Willy”. a Union Cabinet Minister nominated by the • Hurricanes are characterised by a low- Prime Minister. Hence, statement 1 is not pressure centre surrounded by a spiral correct. arrangement of thunderstorms that produce • The Chief Information Commissioner and strong winds and heavy rain. an Information Commissioner shall hold • Hurricanes move with a counter clockwise office for such term as prescribed by the wind flow in the Northern Hemisphere and Central Government or until they attain the clockwise wind flow in the Southern age of 65 years, whichever is earlier. Hemisphere. Hence, statement 2 is correct. • They are not eligible for reappointment. www.iasgatewayy.com  Daily Current Affairs | Monthly 044- Magazines | Online Tests 1 | P a g e

26265326/9884421666/9884472636 UPSC OCTOBER - 2020 DAILY CURRENT AFFAIRS CLASS TEST – 13 3. With reference to Lepidoptera, which of the Explanation: following is correct? • The World Bank has recently released its (a) It is a genus of insects. biennial South Asia Economic Focus (b) It is an order of insects. report. The fall 2020 edition is titled (c) It is a kingdom of insects. “Beaten or Broken? Informality and (d) It is a species of insects. COVID-19”. Answer: b) Explanation: 5. Consider the following statements: • Lepidoptera is an order of insects that 1. The Central Information Commission is a includes butterflies, moths and skippers. statutory body established under the Hence, option A is correct. provisions of the Right to Information Act • The name Lepidoptera is derived from the (2005). Greek, meaning “scaly winged,” and refers 2. The Commission consists of a Chief to the characteristic covering of Information Commissioner and not more microscopic dustlike scales on the wings. than ten Information Commissioners. • Lepidoptera are ecologically important 3. The Chief Information Commissioner and because they transform large amounts of an Information Commissioner (of Centre as plant matter into the animal matter and in well as States) shall hold office for term turn serve as food for many other groups of fixed of 5 years or 65 years of age Animals. whichever is earlier. • The adults of many species are important Which of the statements given above are for their role in pollination, which occurs as correct? they visit Flowers for Nectar. (a) 1 and 2 only (b) 1 and 3 only 4. South Asia Economic Focus report is released (c) 2 and 3 only by (d) 1, 2 and 3 (a) World Bank Answer: c) (b) UNDP Explanation: (c) WEF • The Central Information Commission is a (d) IMF statutory body established under the Answer: a) provisions of the Right to Information Act

(2005). So, statement 1 is correct.

www.iasgatewayy.com  Daily Current Affairs | Monthly 044- Magazines | Online Tests 2 | P a g e

26265326/9884421666/9884472636 UPSC OCTOBER - 2020 DAILY CURRENT AFFAIRS CLASS TEST – 13 • The Commission consists of a Chief State Information Commissioner due to Information Commissioner and not more pension or any other retirement benefits than ten Information Commissioners. So, received by them for their previous statement 2 is correct. Government service. • Right to Information (Amendment) Act, 6. Recently bulk call data records were sought by 2019 provided that the Chief Information the Department of Telecommunication (DoT) Commissioner and an Information for improving the quality of telecom services. Commissioner (of Centre as well as States) With reference to this context, consider the shall hold office for such term as following statements: prescribed by the Central Government. 1. The Government is empowered under Before this amendment, their term was Telecom Regulatory Authority of India fixed for 5 years. So, statement 3 is not Act, 1997 to access such anonymous data correct. for improving network quality. • It provided that the salary, allowances and 2. Any authorisation of such access to call other service conditions of the Chief drop data can be approved only by very Information Commissioner and an senior officers. Information Commissioner (of Centre as 3. Seeking such data can be done only for well as States) shall be such as prescribed short time period i.e. three to six hours by the Central Government. normally covering the peak load of traffic • Before this amendment, the salary, on the network for any cell tower. allowances and other service conditions of Which of the statements given above are the Chief Information Commissioner were correct? similar to those of the Chief Election (a) 1 and 2 only Commissioner and that of an Information (b) 2 and 3 only Commissioner were similar to those of an (c) 1 and 3 only Election Commissioner (State Election (d) 1, 2 and 3 Commissioners in case of States). Answer: b) • It removed the provisions regarding Explanation: deductions in salary of the Chief • The Government is empowered under Rule Information Commissioner, an 419 of the Indian Telegraph Rules, 1951, Information Commissioner, the State to access such anonymous data for Chief Information Commissioner and a

www.iasgatewayy.com  Daily Current Affairs | Monthly 044- Magazines | Online Tests 3 | P a g e

26265326/9884421666/9884472636 UPSC OCTOBER - 2020 DAILY CURRENT AFFAIRS CLASS TEST – 13 improving network quality. So, statement Explanation: 1 is not correct. • MACS 6478 is a variety of Wheat. • Any authorisation of such access to call • The wheat variety called MACS 6478 has drop data can be approved only by very been developed by Scientists from senior officers. So, statement 2 is correct. Agharkar Research Institute (ARI), an • Further, it has been decided to seek such autonomous institute of the Department of data only for short time period i.e. three to Science and Technology (DST), six hours normally covering the peak load Government of India. of traffic on the network for any cell tower. • It has doubled the crop yield for farmers in So, statement 3 is correct. Karanjkhop, a village in Maharashtra. • The farmers are now getting a yield of 45- 7. Consider the following pairs: 60 quintal per hectare with the new variety Name of the Port Country as against earlier average yield ranging 25- 1. Sitwe Port - Myanmar 30 quintal per hectare. 2. Chabahar Port - Iran • The newly developed common wheat or 3. Gwadar Port - Pakistan bread wheat, also called high yielding 4. Hambantota Port - Sri Lanka Aestivum, matures in 110 days and is Which of the pairs given above are correct? resistant to most races of leaf and stem rust. (a) 1, 2 and 3 only • The amber coloured medium sized grain (b) 2, 3 and 4 only contains 14% protein, 44.1 ppm zinc and (c) 1, 3 and 4 only 42.8 ppm iron which is higher than other (d) All the above cultivated varieties. Answer: d) • The chapati quality of flour of this wheat is Explanation: excellent, having score of 8.05 with good • All the above pairs are correctly matched. bread quality scoring 6.93.

8. MACS 6478, recently in news, is a variety of? 9. Consider the following statements with (a) Rice reference to Dhawan-1 rocket engine: (b) Gram 1. It is world’s first privately developed (c) Wheat indigenous fully . (d) Cotton 2. The rocket engine runs on liquid natural gas Answer: c) (LNG). 3. The cryogenic engine 100% 3D printed. www.iasgatewayy.com  Daily Current Affairs | Monthly 044- Magazines | Online Tests 4 | P a g e

26265326/9884421666/9884472636 UPSC OCTOBER - 2020 DAILY CURRENT AFFAIRS CLASS TEST – 13 Which of the above statements is/are correct? National Oceanic and Atmospheric (a) 1 and 2 only Administration. (b) 2 and 3 only • NPP is the Earth-observing satellite system (c) 1 and 3 only that collects data on long-term climate (d) All of the above change and short-term weather conditions. Answer: b) • It is the result of a partnership between Explanation: NASA, the National Oceanic and • Space rocket startup Skyroot Aerospace Atmospheric Administration, and the unveiled its cryogenic engine Dhawan-1 on Department of Defense. the occasion of 100th birth anniversary of • NPP will improve the Earth system data Indian rocket scientist on records established by NASA’s Earth 25 September 2020. Observing System fleet of satellites that • Dhawan-1 is India’s first privately provide critical insights into the dynamics developed indigenous fully cryogenic of the entire Earth system: clouds, oceans, rocket engine. vegetation, ice, solid Earth and atmosphere. • It runs on liquid natural gas (LNG) as fuel. • It was launched in 2011 and continues to • The cryogenic engine Dhawan-1 is 100% operate. 3D printed and fuelled by LNG and liquid oxygen as oxidizer.

• The cryogenic engine will be powering its

Vikram II rocket.

10. Suomi National Polar orbiting Partnership is a weather satellite operated by: (a) France (b) US (c) Japan (d) Russia Answer: b) Explanation: • The Suomi National Polar-orbiting Partnership or Suomi NPP is a weather satellite operated by the United States www.iasgatewayy.com  Daily Current Affairs | Monthly 044- Magazines | Online Tests 5 | P a g e

26265326/9884421666/9884472636 UPSC OCTOBER - 2020 DAILY CURRENT AFFAIRS CLASS TEST – 14 1. 'Namath Basai' initiative, sometimes seen in 2. With reference to Human Cost of Disasters News is related to? report, Consider the following statements: (a) It is a programme for teaching tribal 1. It is published by the United Nations Office children in their mother tongue. for Disaster Risk Reduction. (b) It provides access to safe and adequate 2. It also considers the biological hazards and water for the tribal communities. disease-related Disasters. (c) It is a capacity building of scheduled tribe Which of the statements given above is/are representatives in local self-governments. correct? (d) It aims to provide e-marketplace for (a) 1 only handicraft and organic products for tribals. (b) 2 only Answer: a) (c) Both 1 and 2 Explanation: (d) Neither 1 nor 2 • The Kerala State government is carrying Answer: a) out a unique programme called “Namath Explanation: Basai” of teaching tribal children in their • In a new report “The Human Cost of mother tongue. Disasters 2000-2019”, the United Nations • The programme is being implemented by pointed out that climate change is largely to the Samagra Shiksha Kerala (SSK). SSK is blame for a near doubling of natural an overarching programme for the school disasters in the past 20 years. education sector extending from pre-school • The report is published by the United to class 12. Nations Office for Disaster Risk Reduction • It aims at improving school effectiveness (UNDRR). measured in terms of equal opportunities • The report did not touch on biological for schooling and equitable learning hazards and disease-related disasters like outcomes. Kerala has become the first state the coronavirus pandemic. in the country to have high-tech classrooms • Report was released on 13th October 2020, or high-tech labs in all government-run and the day observed as International Day for aided schools. The Kerala government has Disaster Risk Reduction. also started digital classrooms known as ‘First bell’ to impart education to 41 lakh students in the state after schools shut due to the lockdown.

www.iasgatewayy.com  Daily Current Affairs | Monthly 044- Magazines | Online Tests 1 | P a g e

26265326/9884421666/9884472636 UPSC OCTOBER - 2020 DAILY CURRENT AFFAIRS CLASS TEST – 14 3. In context Sovereign Gold Bond (SGB) • National Stock Exchange of India Limited scheme, consider the following statements: and Bombay Stock Exchange, either 1. Its objective is to increase the demand for directly or through agents. physical gold and increase the domestic savings. 4. ’27 Point Action Plan’, sometimes seen in 2. The Gold Bonds are issued by the Reserve news is related to Bank of India on behalf of the Union (a) FATF Government. (b) EU Which of the statements given above is/are (c) G20 correct? (d) UNEP (a) 1 only Answer: a) (b) 2 only Explanation: (c) Both 1 and 2 • The International Co-operation Review (d) Neither 1 nor 2 Group (ICRG) of the Financial Action Answer: b) Task Force (FATF) recently Explanation: recommended that Pakistan should be • The Sovereign Gold Bond (SGB) scheme retained on the “Grey List” as it had failed was launched in November 2015 with an to implement the 27 Point Action Plan. objective to reduce the demand for physical • The Financial Action Task Force (FATF) is gold and shift a part of the domestic savings an inter-governmental body established - used for the purchase of gold - into in 1989 during the G7 Summit in Paris. financial savings. • The objectives of the FATF are to set • The Gold Bonds are issued as Government standards and promote effective of India Stock under the Government implementation of legal, regulatory and Securities (GS) Act, 2006. operational measures for combating • These are issued by the Reserve Bank of money laundering, terrorist financing India (RBI) on behalf of the Government of and other related threats to the integrity India. The Gold Bonds are sold through of the international financial system. Commercial banks, Stock Holding • FATF has two lists: Corporation of India Limited (SHCIL), ✓ Grey List: Countries that are considered designated post offices (as may be notified) safe haven for supporting terror funding • and recognised stock exchanges viz., and money laundering are put in the FATF

www.iasgatewayy.com  Daily Current Affairs | Monthly 044- Magazines | Online Tests 2 | P a g e

26265326/9884421666/9884472636 UPSC OCTOBER - 2020 DAILY CURRENT AFFAIRS CLASS TEST – 14 grey list. This inclusion serves as a warning 2. cartoonist, editor, publishers of registered to the country that it may enter the blacklist. newspaper ✓ Black List: Countries known as Non- 3. Judge, government servants or employee of Cooperative Countries or Territories any corporation (NCCTs) are put in the blacklist. These 4. Member of any legislature countries support terror funding and money 5. Political parties. laundering activities. The FATF revises the • Despite being a law related to financial blacklist regularly, adding or deleting regulation, this law does not fall within the entries. purview of the RBI but under the Home Ministry as it is internal security 5. In the context of Foreign Contribution legislation.

Regulation Act (FCRA) 2010, consider the following statements: 6. Consider the following statements: 1. Under this act, candidate contesting the 1. Uttar Pradesh was the first state to pass a election is debarred from receiving law against lynching in the state. contribution. 2. Rajasthan, Manipur and West Bengal are 2. FCRA comes under the purview of the the only states to pass law against Mob Department of Economic Affairs of Lynching. Ministry of Finance. 3. The Supreme Court has given guideline to Which of the statement(s) given above is/are the State governments to prepare a correct? lynching/mob violence victim (a) 1 only compensation scheme. (b) 2 only Which of the statements given above are (c) Both 1 and 2 correct? (d) Neither 1 nor 2 (a) 1 and 2 only Answer: a) (b) 2 and 3 only Explanation: (c) 1 and 3 only • As per FCRA 2010, the following (d) 1, 2 and 3 organizational individuals are debarred Answer: b) from receiving foreign contribution. They Explanation: are • Manipur was the first state to pass a law 1. Candidate for election against lynching in the state. So, statement 1 is not correct. www.iasgatewayy.com  Daily Current Affairs | Monthly 044- Magazines | Online Tests 3 | P a g e

26265326/9884421666/9884472636 UPSC OCTOBER - 2020 DAILY CURRENT AFFAIRS CLASS TEST – 14 • Rajasthan, Manipur and West Bengal are can be established that the detention is the only states to pass law against Mob illegal. There have been cases where the Lynching. So, statement 2 is correct. High Court has interfered and quashed the • The Supreme Court has given guideline to detention. So, statement 1 is not correct. the State governments to prepare a • According to Section 13(2), the detaining lynching/mob violence victim authority need not even inform the detained compensation scheme. So, statement 3 is individual as to the reason for the action, if correct. it decides that it goes against public interest. So, statement 2 is correct. 7. Consider the following statements regarding

the Public Safety Act of Jammu & Kashmir: 8. The Island of Kastellorizo is located in: 1. Usually when a person is arrested, they (a) Mediterranean Sea have the right to legal representation and (b) Black Sea can challenge the arrest. (c) South China Sea 2. According to Section 13(2), the detaining (d) East China Sea authority need not even inform the detained Answer: a) individual as to the reason for the action, if Explanation: it decides that it goes against public • Kastellorizo Island also known as Meis is interest. located in the Eastern Mediterranean.

Which of the statement(s) given above is/are correct? 9. Which of the following countries are members (a) 1 only of both the Asia-Pacific Group on Money (b) 2 only Laundering and the Financial Action Task (c) Both 1 and 2 Force (FATF)? (d) Neither 1 nor 2 1. India Answer: a) 2. Hong Kong Explanation: 3. United States • Usually when a person is arrested, they 4. Pakistan have the right to legal representation and 5. Australia can challenge the arrest. But, when a Choose the correct option: person is arrested under the PSA, they (a) 1, 2, 4 and 5 only do not have these rights before the (b) 1, 2, 3 and 5 only Advisory Board unless sufficient grounds (c) 1, 4 and 5 only www.iasgatewayy.com  Daily Current Affairs | Monthly 044- Magazines | Online Tests 4 | P a g e

26265326/9884421666/9884472636 UPSC OCTOBER - 2020 DAILY CURRENT AFFAIRS CLASS TEST – 14 (d) 1, 2 and 3 only rates to developing countries, which has to Answer: b) be repaid by the borrowing government. Explanation: • The LOCs also help to promote exports of • Pakistan is not a member state of FATF. It Indian goods and services, as 75% of the is an FATF Associate Member of the value of the contract must be sourced from Asia/Pacific Group on Money Laundering India. (APG). FATF Asia-Pacific Group is one of the regional affiliates of the Financial Action Task Force. • India, China, Australia, Canada, Hong Kong, Japan, Korea, Malaysia, New Zealand Singapore and the United States are members of both APG as well as head FATF.

10. Consider the following statements with respect to Line of Credit (LoC): 1. It is a soft loan provided on concessional interest rates to developing countries. 2. The LoCs promote exports of Indian goods and services. 3. Under an LoC, 75% of the value of the contract must be sourced from India. Which of the given statement/s is/are correct? (a) 1 only (b) 1 and 3 only (c) 1, 2 and 3 (d) 1 and 2 only Answer: c) Explanation: • The Line of Credit is not a grant but a ‘soft loan’ provided on concessional interest

www.iasgatewayy.com  Daily Current Affairs | Monthly 044- Magazines | Online Tests 5 | P a g e

26265326/9884421666/9884472636 UPSC OCTOBER - 2020 DAILY CURRENT AFFAIRS CLASS TEST – 15 1. Consider the following pairs: African Elephant: Species IUCN Status • The African elephant is a genus- consisting 1. Asian Elephant - Endangered of 2 species: the African forest elephant and 2. African Elephant - Vulnerable the African bush elephant. 3. Great Indian Bustard - Critically • They are listed as vulnerable species by Endangered IUCN. African elephant species are found Which of the pair(s) given above is/are in the sub-Saharan Africa. correctly matched? • The African forest elephant occurs only in (a) 1 only the Central African region. (b) 1 and 2 only Great Indian Bustard: (c) 1 and 3 only • The Great Indian Bustard is one of the (d) 1, 2 and 3 heaviest flying birds in the world. Answer: d) • It usually resides in dry grasslands and Explanation: scrublands on the Indian subcontinent; its Asian Elephant largest populations are found in the Indian • India is the natural home of the largest state of Rajasthan. population of Asian elephants. It is also • The Great Indian Bustard is the state bird found in Nepal, Bangladesh, Bhutan and of Rajasthan. Myanmar. • It is listed as ‘Critically Endangered’ on • It usually resides in shrub lands, the IUCN Red List. It is also listed in artificial/terrestrial forests and grasslands. Appendix I of CITES and Schedule I of • It is listed as ‘Endangered’ on the IUCN the Indian Wildlife (Protection) Act, 1972. Red List of threatened species. It is also • 90% of its population has been reduced listed in Appendix I of the Convention on within 50 years (six generations) majorly International Trade in Endangered due to poaching. Species of Wild Fauna and Flora (CITES) and Schedule I of the Wildlife 2. With reference to Indian elephants, consider (Protection) Act, 1972. the following statements: • The challenges confronting Asian elephant 1. The leader of an elephant group is female. conservation in most elephant Range States 2. The maximum gestation period can be 22 are habitat loss and fragmentation, human- months. elephant conflict, and poaching and illegal 3. An elephant can normally go on calving till trade of elephants. the a of 40 years only. www.iasgatewayy.com  Daily Current Affairs | Monthly 044- Magazines | Online Tests 1 | P a g e

26265326/9884421666/9884472636 UPSC OCTOBER - 2020 DAILY CURRENT AFFAIRS CLASS TEST – 15 4. Among the States in India, the highest construction costs are met by the elephant population is in Kerala. Government. Which of the statement(s) given above is/are Which of the statement(s) given above is/are correct? correct? (a) 1 and 2 only (a) 1 only (b) 2 and 4 only (b) 2 only (c) 3 only (c) Both 1 and 2 (d) 1, 3 and 4 only (d) Neither 1 nor 2 Answer: a) Answer: b) Explanation: Explanation: • Calves generally live with the mother till • Under this model, the cost is completely they are five years old after which the borne by the government. So, statement 1 males leave the herd but the females is not correct. remain. Statement 1 is correct and • Government invites bids for engineering Statement 3 is incorrect. knowledge from the private players. • Elephants have the longest gestation period Procurement of raw materials and of all mammals. These gentle giants' construction costs are met by the pregnancies last for more than a year and a Government. So, statement 2 is correct. half. The average gestation period of an 4. Consider the following pairs: elephant is about 640 to 660 days, or Name of Missile Type roughly 95 weeks. Statement 2 is correct. 1. K-4 Missile - Submarine launched • Karnataka has the highest number of Ballistic Missile elephants in India. Statement 4 is 2. Akash - Surface to Air Missile Incorrect. 3. Nag - Anti Tank guided missile 3. With respect to the Engineering, Procurement Which of the following pair(s) is/are correctly and Construction (EPC) Model, consider the matched? following statements: (a) 1 only 1. Under this model, the cost borne by the (b) 1 and 2 only government and the private player is 50:50. (c) 3 only 2. Government invites bids for engineering (d) 1, 2 and 3 knowledge from the private players and the Answer: d) Procurement of raw materials and www.iasgatewayy.com  Daily Current Affairs | Monthly 044- Magazines | Online Tests 2 | P a g e

26265326/9884421666/9884472636 UPSC OCTOBER - 2020 DAILY CURRENT AFFAIRS CLASS TEST – 15 Explanation: • Electronic Vaccine Intelligence Network • All the three are Correct. (eVIN) is an indigenously developed technology system that digitizes vaccine 5. Consider the following statements with respect stocks and monitors the temperature of the to eVIN: cold chain through a smartphone 1. It is an indigenously developed technology application. system that digitizes vaccine stocks and • It is being implemented under the National monitors the temperature of the cold chain Health Mission (NHM) by the Ministry of through a smartphone application. Health and Family Welfare in partnership 2. It is an initiative by the Ministry of Health with the United Nations Development and Family Affairs that started in 2004 with Programme (UNDP).

the assistance of the World Bank. 3. It is being implemented under the National 6. “Aquaponics” refers to: Health Mission (NHM). (a) The technique of cultivating, harvesting, Which of the given statement/s is/are correct? freshwater and saltwater fish as well as (a) 1 and 2 only shellfish in a controlled environment. (b) 2 and 3 only (b) The technique of growing plants without (c) 1 and 3 only soil. (d) 1, 2 and 3 (c) The technique used for regulating water Answer: c) temperatures to prevent Coral Bleaching. Explanation: (d) The technique of growing both fishes as • In 2015, in partnership with Gavi, the well as plants in an integrated manner. Vaccine Alliance, UNDP supported the Answer: d) Government of India to launch the • Aquaponics is an emerging technique in electronic Vaccine Intelligence Network which both fishes as well as plants are (eVIN), an electronic logistics grown in an integrated manner. management information system (eLMIS) • Aquaponics is a combination of that uses smartphone and cloud-based aquaculture and hydroponics. technology to capture real-time data across • Aquaculture is growing fish and other the entire vaccine cold chain, from the aquatic animals. zonal store depots to the last-mile health • Hydroponics is growing plants without facilities. soil.

www.iasgatewayy.com  Daily Current Affairs | Monthly 044- Magazines | Online Tests 3 | P a g e

26265326/9884421666/9884472636 UPSC OCTOBER - 2020 DAILY CURRENT AFFAIRS CLASS TEST – 15 Tag: Environment Answer: c) 7. Which of the following pollutants are Explanation: considered by SAFAR to measure air quality? • AMRUT (Atal mission for rejuvenation 1. Mercury and urban transformation) Mission was 2. Carbon Dioxide launched to develop 500 Indian cities by 3. Ozone providing better facilities and civic 4. Toluene amenities. It envisages capacity building, 5. Benzene reform implementation, water supply, Choose the correct option: sewerage and septage management, storm (a) 1, 2, 3, 4 and 5 water drainage, urban transport and (b) 1, 3, 4 and 5 only development of green spaces and parks. (c) 3, 4 and 5 only • It is a central sponsored scheme whereby (d) 2 and 3 only Centre and states both contribute finances. Answer: b) • AMRUT has made states equal partners in Explanation: planning and implementation of projects by • Pollutants monitored by SAFAR to approval of SAAP once a year by MoHUA measure the air quality are: PM2.5, PM10, and states have to give project sanctions Ozone, Carbon Monoxide (CO), Nitrogen and approval at their end, therefore Oxides (NOx), Sulfur Dioxide (SO2), actualisation of cooperative federalism. Benzene, Toluene, Xylene, and Mercury. • It was launched by the Ministry of Housing and Urban Affairs in 2017. 8. In context to AMRUT Mission, consider the

following statements: 9. Consider the following statements: 1. It seeks urban renewal in 500 Indian cities. 1. Manodarpan Initiative is an initiative of the 2. It is a center sector scheme. Ministry of Education under Atmanirbhar 3. It comes under the aegis of Ministry of Bharat Abhiyan. Housing and Urban Affairs. 2. KIRAN is a toll-free helpline to provide Which of the statements given above are support to people facing mental health correct? concerns. (a) 1 and 2 only Which of the statements given above is/are (b) 2 and 3 only correct? (c) 1 and 3 only (a) 1 only (d) 1, 2 and 3 (b) 2 only www.iasgatewayy.com  Daily Current Affairs | Monthly 044- Magazines | Online Tests 4 | P a g e

26265326/9884421666/9884472636 UPSC OCTOBER - 2020 DAILY CURRENT AFFAIRS CLASS TEST – 15 (c) Both 1 and 2 Explanation: (d) Neither 1 nor 2 • Due to the ‘opposition’ effect, Mars will Answer: c) look much brighter and bigger than usual in Explanation: October 2020. • Recently the Ministry of Social Justice and • Opposition, in astronomy, is the Empowerment launched a 24/7 toll-free circumstance in which two celestial bodies helpline called KIRAN to provide support appear in opposite directions in the sky. to people facing anxiety, stress, depression, Examples: suicidal thoughts and other mental health • The Moon, when full, is said to be in concerns. opposition to the Sun; the Earth is then • Manodarpan Initiative is an initiative of the approximately between them. Ministry of Education under Atmanirbhar • A superior planet (one with an orbit farther Bharat Abhiyan. from the Sun than Earth’s) is in opposition • It is aimed to provide psychosocial support when Earth passes between it and the Sun. to students, family members and teachers • The opposition of a planet is a good time to for their mental health and well-being observe it, because the planet is then during the times of Covid-19. typically at its nearest point to the Earth for a given year and because it is close, the 10. Consider the following: planet appears brighter in the sky. 1. The planets Venus and Mercury can never • The planets Venus and Mercury, whose be in opposition to the Sun. orbits are smaller than Earth’s, can never be 2. The opposition effect is the circumstance in in opposition to the Sun. which two celestial bodies appear in opposite directions in the sky. Which of the statements given above is/are correct? (a) 1 only (b) 2 only (c) Both 1 and 2 (d) Neither 1 nor 2 Answer: c)

www.iasgatewayy.com  Daily Current Affairs | Monthly 044- Magazines | Online Tests 5 | P a g e

26265326/9884421666/9884472636 UPSC OCTOBER - 2020 DAILY CURRENT AFFAIRS CLASS TEST – 16 1. In context to Hologram Imaging, consider the Which of the statements given above is/ are following statements: correct? 1. These are three-dimensional images. (a) 1 only 2. It can be used to detect viruses and (b) 2 only antibodies. (c) Both 1 and 2 3. They are generated by interference and (d) Neither 1 nor 2 diffraction of laser beams. Answer: c) Which of the statements given above are Explanation: correct? • Recently, the World Wildlife Fund (WWF) (a) 1 and 2 only India has published reports titled (b) 2 and 3 only ‘Prevalence of Endoparasitic in (c) 1 and 3 only Free-Ranging Greater One-Horned (d) 1, 2 and 3 Rhinoceros’ for Assam and West Bengal. Answer: d) • There are three species of rhino in Asia — Explanation: Greater one-horned (Rhinoceros • Holography is a process that creates three- unicornis), Javan and Sumatran. dimensional images called holograms. • The species is restricted to small habitats in Hence, statement 1 is correct. Indo-Nepal terai and northern West Bengal • Holograms are created by using laser and Assam. beams, the properties of interference and • In India, rhinos are mainly found in diffraction, light intensity recording, and Kaziranga NP, Pobitora WLS, Orang NP, illumination of the recording. Hence, Manas NP in Assam, Jaldapara NP and statement 2 is correct. Gorumara NP in West Bengal and Dudhwa • Recently, scientists from New York TR in Uttar Pradesh. Hence, statement 1 is University have developed a method using not correct. holographic imaging to detect both viruses Protection Status: and Antibodies. • Javan and Sumatran Rhino are critically endangered and the Greater one-horned (or 2. Consider the following: Indian) rhino is vulnerable in IUCN Red 1. The habitat of one-horned rhinoceros is List. Hence, statement 2 is correct. restricted to North-East India only. • All three listed under Appendix I (CITES). 2. The one-horned rhinoceros is listed as vulnerable in the IUCN Red List. www.iasgatewayy.com  Daily Current Affairs | Monthly 044- Magazines | Online Tests 1 | P a g e

26265326/9884421666/9884472636 UPSC OCTOBER - 2020 DAILY CURRENT AFFAIRS CLASS TEST – 16 • Greater one-horned rhino is listed under the Schedule I of the Wildlife Protection Act, Explanation: 1972. • Madhuca Diplostemon is a species of Conservation Efforts by India: flowering plant from the Sapotaceae • Recently, the Ministry of Environment family. It has been listed as Endangered by Forest and Climate Change (MoEFCC) has the International Union for Conservation of begun a project to create DNA profiles of Nature (IUCN) Red List of Threatened all rhinos in the country. Species. • The National Rhino Conservation Strategy • A tree species, long believed extinct, has was launched in 2019 to conserve the been rediscovered after a gap of 184 years greater one-horned rhinoceros. Hence, from a sacred grove in Kollam district or statement 3 is not correct. Kerala. • Indian Rhino Vision 2020 was Launched in • Scientists at the Jawaharlal Nehru Tropical 2005, it is an ambitious effort to attain a Botanic Garden and Research Institute wild population of at least 3,000 greater (JNTBGRI) at Palode have identified the one-horned rhinos spread over seven tree of Madhuca diplostemon (family protected areas in the Indian state of Assam Sapotaceae), as a threatened species of the by the year 2020. Western Ghats whose specimen was first collected in 1835. • The JNTBGRI is planning to undertake the 3. Consider the following, with regard to ex situ conservation of this species through Madhuca Diplostemon: the institute’s Species Recovery 1. It is a species of flowering plant Programme. 2. It has been listed as threatened species in

the IUCN Red List. 4. ‘Tulip and Melon Revolution’, sometimes seen 3. It was long believed to be extinct and has in news is related to which of the following been rediscovered after a gap of 184 years. country? Which of the above statements are correct? (a) Bangladesh (a) 1 and 2 only (b) Ukraine (b) 2 and 3 only (c) Kyrgyzstan (c) 1 and 3 only (d) Russia (d) 1, 2 and 3 Answer: c) Answer: d) www.iasgatewayy.com  Daily Current Affairs | Monthly 044- Magazines | Online Tests 2 | P a g e

26265326/9884421666/9884472636 UPSC OCTOBER - 2020 DAILY CURRENT AFFAIRS CLASS TEST – 16 Explanation: (c) MDR is discount rate given for merchant • In 2005 and 2010, sitting presidents of by bank for accepting payment from their Kyrgyzstan were forced out of office in customers in usage of credit or debit cards. popular protests- ‘Tulip’ and ‘Melon’ (d) MDR is discount rate given for merchant revolutions. by government for accepting payment from their customers in usage of credit or debit 5. Which of the following country is often cards. referred to as Central Asia’s only Democracy? Answer: a) (a) Uzbekistan Explanation: (b) Kyrgyzstan • MDR is charge or fee imposed on merchant (c) Turkmenistan by bank for accepting payment from their (d) Tajikistan customers in credit and debit cards every Answer: b) time card is used for payments (like Explanation: swiping) in their stores. • Kyrgyzstan often referred to as Central • MDR charges are usually shared in pre- Asia’s only democracy had recently seen agreed proportion between them and are violent anti-government protests. expressed in percentage of transaction • Similarly, Uzbekistan in Central Asia is the amount. one among the two double landlocked • MDR compensates bank issuing card, bank country, another being Liechtenstein in which puts up swiping machine (Point-of- Europe. Sale or PoS terminal) and network providers such as MasterCard or Visa for

6. Which of the following best describes the their services. Merchant Discount Rate (MDR)? • In India, the RBI specifies maximum MDR (a) MDR is charge or fee imposed on merchant charges that can be levied on every card by bank for accepting payment from their transaction. customers in usage of credit or debit cards. • But now, Finance Ministry has notified that (b) MDR is charge or fee imposed on merchant Digital transactions made using RuPay by the government for accepting payment credit cards, or UPI QR codes will not face from their customers in usage of credit or additional charges for merchants or debit cards. customers from the beginning of next year.

www.iasgatewayy.com  Daily Current Affairs | Monthly 044- Magazines | Online Tests 3 | P a g e

26265326/9884421666/9884472636 UPSC OCTOBER - 2020 DAILY CURRENT AFFAIRS CLASS TEST – 16 • All shops, business establishments and • Within its geographic range, SnowEx companies with an annual turnover of Rs. assesses where snow has fallen, how much 50 crore or more have been mandated to there is and how its characteristics change offer these modes of payment to customers. as it melts. • It uses airborne measurements, ground 7. ‘SnowEx’ sometimes seen in news is a measurements and computer modelling. (a) An initiative to protect the endangered • The airborne campaign will fly radar and animals in Himalayan Region lidar to measure snow depth, microwave (b) An express started in Himalayan states for radar and radiometers to measure SWE, about 1500 Kms. optical cameras to photograph the surface, (c) Seasonal campaign launched by the NASA infrared radiometers to measure surface to lay the groundwork for a future snow temperature, and hyper spectral imagers for satellite mission. snow cover and composition. (d) An initiative done by the UNFCCC to • Ground teams will measure snow depth, prevent the melting of glaciers that leads to density, accumulation layers, temperature, rise in sea level. wetness and snow grain size — the size of Answer: c) a typical particle. Explanation:

• For a better understanding of how much 8. Consider the following statements: water is contained in each winter’s 1. It is a classical dance form that was initially snowfall and how much will be available practiced by male dancers only. when it melts in the spring, NASA has 2. It includes a display of dexterity of the launched a seasonal campaign — part of a dancer’s footwork through techniques such five-year programme called SnowEx. as dancing on the rim of a brass plate and • Around 1.2 billion people, or nearly one- with a pitcher full of water on the head. sixth of the world, depend on seasonal 3. The dance form being talked about is: snow and glaciers for their water supply. (a) Bharatanatyam • SnowEx was initiated in 2016-17. The (b) Mohiniattam overall target is optimal strategies for (c) Kuchipudi mapping global snow water equivalent (d) Sattriya (SWE) with remote sensing and models Answer: c) leading to a Decadal Survey “Earth System Explanation: Explorer mission. • Self-explanatory. www.iasgatewayy.com  Daily Current Affairs | Monthly 044- Magazines | Online Tests 4 | P a g e

26265326/9884421666/9884472636 UPSC OCTOBER - 2020 DAILY CURRENT AFFAIRS CLASS TEST – 16 9. Consider the following statements with respect Answer: b) to Formosa Strait: Explanation: 1. It separates Taiwan and mainland China. • Nandankanan Zoological Park is located in 2. It is a part of the South China Sea and Odisha. It is the first zoo in India to become connects to the Sea of Japan to the north. a member of the World Association of Zoos Which of the given statement/s is/are correct? & Aquariums (WAZA). It is recognized as (a) 1 only a leading zoo for the breeding of the Indian (b) 2 only pangolin and white tiger. (c) Both 1 and 2 (d) Neither 1 nor 2 Answer: a) Explanation: • Formosa Strait, also known as Taiwan Strait, separates Taiwan and mainland China. • The strait is currently part of the South China Sea and connects to the East China Sea to the north.

10. Consider the following statements with respect to Nandankanan Zoological Park: 1. It is located in Tamil Nadu. 2. It is the first zoo in India to become a member of the World Association of Zoos & Aquariums (WAZA). 3. It is recognized as a leading zoo for the breeding of the Indian pangolin and white Tiger. Which of the given statement/s is/are correct? (a) 1 and 2 only (b) 2 and 3 only (c) 1 only (d) None of the above www.iasgatewayy.com  Daily Current Affairs | Monthly 044- Magazines | Online Tests 5 | P a g e

26265326/9884421666/9884472636 UPSC OCTOBER - 2020 DAILY CURRENT AFFAIRS CLASS TEST – 17 1. Consider the following statements with respect Which of the given statement/s is/are to Thalassemia: Incorrect? 1. It is a blood disorder commonly (a) 1 only characterised by the abnormal production (b) 1 and 2 only of haemoglobin in the body. (c) 1, 2 and 3 2. Bone marrow transplant is the only cure (d) None of the above available for the disease. Answer: b) 3. Thalassemia is contagious. Explanation: Which of the given statement/s is/are correct? • The Strengthening Teaching-Learning and (a) 1 only Results for States (STARS) program would (b) 1 and 2 only be under the National Education Policy (c) 2 and 3 only 2020. (d) 1, 2 and 3 • The project is partly funded by the World Answer: b) Bank. Explanation: • The project covers 6 States namely • Thalassemia is a genetic blood disorder Himachal Pradesh, Rajasthan, commonly characterised by the abnormal Maharashtra, Madhya Pradesh, Kerala and production of haemoglobin in the body. Odisha. • Bone marrow transplant is the only cure • The identified States will be supported by available for the disease. various interventions for improving the • Thalassemias are not contagious quality of education. • It would be implemented as a Centrally 2. Consider the following statements about the Sponsored Scheme under the Department Strengthening Teaching-Learning and Results of School Education and Literacy. for States (STARS) program: 3. Philip’s curve describes the relationship 1. The project is partly funded by the Asian between which of the following? Development Bank. (a) Inflation and unemployment 2. The project covers all the states across (b) Tax rates and tax revenue India. (c) Demand and Supply of money 3. It would be implemented as a Centrally (d) Growth and equality Sponsored Scheme under the Department Answer: a) of School Education and Literacy.

www.iasgatewayy.com  Daily Current Affairs | Monthly 044- Magazines | Online Tests 1 | P a g e

26265326/9884421666/9884472636 UPSC OCTOBER - 2020 DAILY CURRENT AFFAIRS CLASS TEST – 17 Explanation: Belgrade, which was held in September • Philip’s curve describes the relationship 1961. Hence, statement 2 is not correct. between inflation and unemployment. • It was held under the leadership of Josip • Phillips curve suggests there is an inverse Broz Tito of Yugoslavia, Gamal Abdel relationship between inflation and Nasser of Egypt, Jawaharlal Nehru of unemployment. India, Kwame Nkrumah of Ghana, and Sukarno of Indonesia. 4. In context to Non-Alignment Movement • NAM doesn’t have a formal constitution or (NAM), consider the following statements: permanent secretariat or any headquarters, 1. The concept of NAM originated from the and its administration is non-hierarchical Balfour Declaration. and rotational. 2. The first summit of NAM was held at 5. Consider the following, with reference to Bandung in 1961. Geneva Conventions (1949): 3. The headquarters of NAM is located in 1. It is an international treaty that contains Kathmandu. rules limiting the barbarity of war. Which of the statements given above are not 2. The convention only protects wounded and correct? sick soldiers on land during war. (a) 1 and 3 only 3. UN Human Rights Council monitors (b) 2 and 3 only signatories to follow the rules in situations (c) 1 and 2 only of conflict. (d) 1, 2 and 3 Which of the statements given above is/are not Answer: d) correct? Explanation: (a) 1 and 3 only • The basic concept for NAM originated in (b) 1 and 2 only 1955 during discussions that took place at (c) 2 and 3 only the Asia-Africa Bandung Conference held (d) 1, 2 and 3 in Indonesia. Hence, statement 1 is not Answer: b) correct. Explanation: • Six years after the Bandung Conference, • The Geneva Conventions (1949) and their the Movement of Non-Aligned Countries Additional Protocols are international was founded on a wider geographical basis treaties that contain the most important at the First Summit Conference of rules limiting the barbarity of war. www.iasgatewayy.com  Daily Current Affairs | Monthly 044- Magazines | Online Tests 2 | P a g e

26265326/9884421666/9884472636 UPSC OCTOBER - 2020 DAILY CURRENT AFFAIRS CLASS TEST – 17 • They also protect people who do not take (c) Both 1 and 2 part in the fighting (civilians, medics, aid (d) Neither 1 nor 2 workers) and those who can no longer fight Answer: b) (wounded, sick and shipwrecked troops, Explanation: prisoners of war). • Saviour Sibling refers to babies that are • The first Geneva Convention protects created to serve an older sibling as a donor wounded and sick soldiers on land during of organs, bone marrow or cells. Hence, war. statement 1 is correct. • The second Geneva Convention protects • Stem cells from the umbilical cord blood or wounded, sick and shipwrecked military blood of the saviour sibling are used for personnel at sea during war. treatment of serious blood disorders like • The third Geneva Convention applies to Thalassemia, Sickle Cell Anaemia. prisoners of war. • They are created with In vitro fertilisation • The fourth Geneva Convention affords (IVF) so that they can undergo pre- protection to civilians, including in implantation genetic diagnosis (or testing) occupied territory. to rule out any genetic disorders and also • The International Committee for the Red check bone marrow compatibility. Hence, Cross (ICRC), an international statement 2 is not correct. humanitarian organisation, has the mandate • In vitro comes from the latin word ‘in to monitor that signatories follow the rules glass’, i.e. studies are done in a test tube in situations of conflict. rather than in a human or animal. • The opposite to ‘In-vitro’ is ‘In-vivo’, 6. Consider the following statements, with regard which comes from the latin word ‘within to ‘Saviour Sibling’: the living’. In vivo refers to 1. It refers to the babies that are created to experimentation being done in a living serve an older sibling as a donor of bone organism.

Marrow. 2. The babies are born via the natural process 7. In the context to the Minimum Support Price with In-vivo fertilisation. for the Minor Forest Produce, consider the Which of the given statements is/are not following statements: correct? 1. The Ministry of Tribal Affairs will be the (a) 1 only nodal Ministry for implementation and (b) 2 only monitoring of the scheme. www.iasgatewayy.com  Daily Current Affairs | Monthly 044- Magazines | Online Tests 3 | P a g e

26265326/9884421666/9884472636 UPSC OCTOBER - 2020 DAILY CURRENT AFFAIRS CLASS TEST – 17 2. The scheme supports primary value Which of the statement(s) given above is/are addition as well as provides for supply correct? chain infrastructure like cold storage, (a) 1 only warehouses etc. (b) 1 and 2 only 3. It ensures that the tribal population gets a (c) 1 and 3 only remunerative price for the produce they (d) 1, 2 and 3 collect from the forest and provide Answer: b) alternative employment avenues to them. Explanation: Which of the statements given above are • The Minimum Support Price is fixed on the correct? recommendations of the Commission for (a) 1 and 2 only Agricultural Costs and Prices (CACP). So, (b) 2 and 3 only statement 1 is correct. (c) 1 and 3 only • The Cabinet Committee on Economic (d) 1, 2 and 3 Affairs (CCEA) of the Union government Answer: d) takes a final decision on the level of MSPs Explanation: and other recommendations made by the • All the three statements are factual CACP. So, statement 2 is correct. statements of the scheme. • The Food Corporation of India (FCI), the nodal central agency of the Government of 8. Consider the following statements: India, along with other State Agencies 1. The Minimum Support Price is fixed on the undertakes procurement of MSP crops. So, recommendations of the Commission for statement 3 is not correct.

Agricultural Costs and Prices (CACP). 2. The Cabinet Committee on Economic 9. Global Hunger Index is released by Affairs (CCEA) of the Union Government (a) World Bank takes a final decision on the level of MSPs (b) UNDP and other recommendations made by the (c) WEF CACP. (d) Welthungerhilfe and Concern Worldwide 3. The Indian Agriculture Research Institute Answer: d) (ICAR), the nodal central agency of the Explanation: Government of India, along with other • The report is a peer-reviewed publication State Agencies undertakes procurement of released annually by Welthungerhilfe and MSP crops. Concern Worldwide. www.iasgatewayy.com  Daily Current Affairs | Monthly 044- Magazines | Online Tests 4 | P a g e

26265326/9884421666/9884472636 UPSC OCTOBER - 2020 DAILY CURRENT AFFAIRS CLASS TEST – 17 • The GHI scores are based on a formula that Rainfall, Temperature, Wind, Humidity captures three dimensions of hunger— etc. insufficient caloric intake, child under Select the correct answer using the code given nutrition, and child mortality—using four below: component indicators: (a) 1 only ✓ UNDERNOURISHMENT: the share of (b) 2 only the population that is under-nourished, (c) Both 1 and 2 reflecting insufficient caloric intake. (d) Neither 1 nor 2 ✓ CHILD WASTING: the share of children Answer: c) under the age of five who are wasted (low Explanation: weight-for-height), reflecting acute under • Both are factual statements. nutrition. About Pradhan Mantri Fasal Bima Yojana ✓ CHILD STUNTING: the share of (PMFBY): children under the age of five who are • The Pradhan Mantri Fasal Bima Yojana stunted (low height-for-age), reflecting was launched in 2016 and is being chronic under nutrition. administered by the Ministry of ✓ CHILD MORTALITY: the mortality rate Agriculture and Farmers Welfare. of children under the age of five. • It provides a comprehensive insurance cover against the failure of the crop thus 10. With respect to the Pradhan Mantri Fasal Bima helping in stabilizing the income of the Yojana and the Restructured Weather Based farmers. All food & oilseed crops and Crop Insurance Scheme, which of the annual commercial/horticultural crops for statements given above is/are correct? which past yield data is available. 1. Both the Pradhan Mantri Fasal Bima • The prescribed premium is 2% to be paid Yojana (PMFBY) and the Restructured by farmers for all Kharif crops and 1.5% Weather Based Crop Insurance Scheme for all Rabi crops. In the case of annual (RWBCI) is being administered by the commercial and horticultural crops, the Ministry of Agriculture and Farmers premium is 5%. Welfare. • The scheme is implemented by empanelled 2. While PMFBY provides a comprehensive general insurance companies. The selection insurance cover against the failure of the of the Implementing Agency (IA) is done crop, RWBCI provides insurance only due by the concerned State Government to adverse weather conditions relating to through bidding. www.iasgatewayy.com  Daily Current Affairs | Monthly 044- Magazines | Online Tests 5 | P a g e

26265326/9884421666/9884472636 UPSC OCTOBER - 2020 DAILY CURRENT AFFAIRS CLASS TEST – 17 About Restructured Weather Based Crop Insurance Scheme: • Restructured Weather Based Crop Insurance Scheme (RWBCIS) was launched in 2016 and is being administered by the Ministry of Agriculture and Farmers Welfare. • It aims to mitigate the hardship of the insured farmers against the likelihood of financial loss on account of anticipated crop loss resulting from adverse weather conditions relating to rainfall, temperature, wind, humidity etc. • WBCIS uses weather parameters as “proxy” for crop yields in compensating the cultivators for deemed crop losses.

www.iasgatewayy.com  Daily Current Affairs | Monthly 044- Magazines | Online Tests 6 | P a g e

26265326/9884421666/9884472636 UPSC OCTOBER - 2020 DAILY CURRENT AFFAIRS CLASS TEST – 19 1. Nagorno and Karabakh are sometimes seen in • Taxes on Property and Capital news are places related to Transactions (land revenue, stamps and (a) Recently discovered uranium deposits registration fees, urban immovable (b) Tropical rain forests property tax) (c) Underground cave systems • Taxes on Commodities and Services (sales (d) Armenia and Azerbaijan Conflict areas tax, state sales tax/VAT, central sales tax, Answer: d) surcharge on sales tax, receipts of turnover Explanation: tax, other receipts, state excise, taxes on • Recently, three decades-old unresolved vehicles, taxes on goods and passengers, ethno territorial conflict between taxes and duties on electricity, Armenia and Azerbaijan over Nagorno- entertainment tax, state GST, and “other Karabakh flared up once again. The taxes and duties”). conflict is between two relatively small countries and is territorial in nature. 3. With respect to the Capital Adequacy Ratio (CAR), consider the following statements: 2. Which among the following are the state’s own 1. CAR is a measure of the amount of a bank’s Tax Revenues? core capital expressed as a percentage of its 1. Taxes on agricultural income risk-weighted asset. 2. Taxes on Land Revenue 2. It is decided by the Cabinet Committee on 3. Taxes and duties and electricity Economic Affairs to prevent commercial 4. Entertainment Tax banks from taking excess leverage and Select the correct answer using the code given becoming insolvent in the process. below: 3. Indian scheduled commercial banks are (a) 1 and 2 only required to maintain a CAR of 8% as per (b) 1 and 3 only the Basel I norms. (c) 2, 3 and 4 only Which of the statement(s) given above is/are (d) All the above correct? Answer: d) (a) 1 only Explanation: (b) 1 and 3 only • Taxes on Income (agricultural income tax (c) 3 only and taxes on professions, trades, callings (d) 2 and 3 only and employment) Answer: a)

www.iasgatewayy.com  Daily Current Affairs | Monthly 044- Magazines | Online Tests 1 | P a g e

26265326/9884421666/9884472636 UPSC OCTOBER - 2020 DAILY CURRENT AFFAIRS CLASS TEST – 19 Explanation: 5. Consider the following, With reference to • The CRAR, also known as the Capital Nitrous Oxide (N2O): Adequacy Ratio (CAR), is the ratio of a 1. It is a Green House Gas, which is more bank’s capital to its risk. It is a measure of potent than carbon dioxide. the amount of a bank’s core capital 2. It has the third-highest concentration in the expressed as a percentage of its risk- earth’s atmosphere after carbon dioxide weighted asset. So, statement 1 is correct. and methane in the earth's atmosphere. • It is decided by central banks and bank Which of the statements given above is/are regulators to prevent commercial banks correct? from taking excess leverage and becoming (a) 1 only insolvent in the process. So, statement 2 is (b) 2 only not correct. (c) Both 1 and 2 • The Basel III norms stipulated a capital to (d) Neither 1 nor 2 risk weighted assets of 8%. Answer: c) • However, as per RBI norms, Indian Explanation: scheduled commercial banks are required • Nitrous Oxide (N2O) as a greenhouse gas to maintain a CRAR of 9%. So, statement (GHG) is 300 times more potent than 3 is not correct. carbon dioxide (CO2). • Nitrous Oxide (N2O) has the third-highest 4. Mount Harriet National park is located in concentration, after CO2 and methane which of the following state/UT? (CH4), in Earth’s atmosphere among (a) Lakshadweep GHGs responsible for global warming. (b) Andaman & Nicobar Island • N2O is also a threat to the ozone layer, for (c) Jammu & Kashmir it accumulates in the atmosphere over a (d) Himachal Pradesh long period of time, just like CO2. Answer: b) • It can live in the atmosphere for up to 125 Explanation: years. • Mount Harriet National Park is a national • Its global concentration levels have park located in the Andaman and Nicobar increased from 270 parts per billion (ppb) Islands union territory of India. in 1750 to 331 ppb in 2018, a jump of 20%.

www.iasgatewayy.com  Daily Current Affairs | Monthly 044- Magazines | Online Tests 2 | P a g e

26265326/9884421666/9884472636 UPSC OCTOBER - 2020 DAILY CURRENT AFFAIRS CLASS TEST – 19 6. Consider the following statements, with regard • They are also informally referred to as the to ‘National Security Guard (NSG)’: ‘Black Cats’ because of their menacing, all- 1. It is an anti-terrorist force which is given black uniforms. the specific role to handle all the facets of 7. In context to Bioremediation, consider the terrorism in any part of the country. following statements: 2. It comprises personnels drawn from State 1. In this process microorganisms are used to Police and Central Armed Police Force neutralize contaminants. only. 2. It can be used to treat oil spills in the ocean. Select the correct answer using the given code 3. The microbial community serves as below: energetic primary degraders of a complex (a) 1 only mixture of petroleum hydrocarbons into (b) 2 only various aldehydes, ketones and acidic (c) Both 1 and 2 Metabolites. (d) Neither 1 nor 2 Which of the above statements is/are correct? Answer: b) (a) 1 and 2 only Explanation: (b) 2 and 3 only • National Security Guard (NSG) Raising (c) 1 and 3 only Day is celebrated on 16th October. (d) All the above • The NSG is an anti-terrorist force. They Answer: c) have been given the specific role to handle Explanation: all the facets of terrorism in any part of the • Bioremediation can be defined as any country as a federal contingency force. process that uses microorganisms or their • It came into existence through the National enzymes to remove and or neutralize Security Guard Act, 1986. contaminants within the environment to • It is modelled on the elite anti-terror forces their Original Condition. GSG-9 of Germany and SAS of the United • Application of Bioremediation in the Kingdom. marine ecosystem is the ability of microbial • NSG has Two Complementary Elements: consortiums to degrade hydrocarbons. This • Special Action Group (SAG): comprising plays an important role in breaking down Army personnel. oil in the event of a spill. • Special Rangers Group (SRG): comprising • The microbial community serves as personnels drawn from State Police and energetic primary degraders of a complex Central Armed Police Force. www.iasgatewayy.com  Daily Current Affairs | Monthly 044- Magazines | Online Tests 3 | P a g e

26265326/9884421666/9884472636 UPSC OCTOBER - 2020 DAILY CURRENT AFFAIRS CLASS TEST – 19 mixture of petroleum hydrocarbons into Functions of NSSO: various aldehydes, ketones and acidic • To conduct large scale sample surveys on metabolites. subjects like household consumer • The complete breakdown and degradation expenditure, employment and of crude oil is achievable using wheat bran unemployment, health and medical marine bacterial consortia (which are low- services. cost non-toxic agro-residues) in an • It decides the topics to be covered in a environmentally sustainable manner. particular survey round. • Agricultural wing of FOD, NSSO has the 8. With reference to National Sample Survey overall responsibility of assisting the states office, consider the following functions: by developing suitable survey techniques 1. It coordinates the results of the crop for obtaining reliable and timely estimates estimation surveys conducted by the states. of crop yield. 2. It conducts large scale sample surveys on • It conducts annual survey of industries household consumer expenditure. (ASI)every year. 3. It collects information used to those • It brings out annual reports on status of engaged in economic activities in the estimation of agricultural production in country. India. Which of the above statements is/are correct? • It has the central responsibility of (a) 1 and 2 only coordinating the results of the crop (b) 2 and 3 only estimation surveys conducted by the states. (c) 1 and 3 only (d) All of the above 9. As per the Sexual Harassment of Women at Answer: d) Workplace (Prevention) Act, 2013, Sexual Explanation: Harassment Includes: • The NSSO was set up in 1950to conduct 1. Physical contact and advances large-scale sample surveys throughout 2. Making sexually coloured remark India. 3. Showing pornography • The employees of the NSSO are from the Which of the above statements is/are correct? Indian Statistical Service and the (a) 1 and 2 only Subordinate Statistical Service. (b) 2 and 3 only (c) 1 and 3 only (d) All of the above www.iasgatewayy.com  Daily Current Affairs | Monthly 044- Magazines | Online Tests 4 | P a g e

26265326/9884421666/9884472636 UPSC OCTOBER - 2020 DAILY CURRENT AFFAIRS CLASS TEST – 19 Answer: d) (a) 1 and 2 only Explanation: (b) 2 and 3 only • Sexual Harassment of Women at (c) 1 and 3 only Workplace (Prevention, Prohibition and (d) All of the above Redressal) Act, 2013: Answer: c) • Objective: Protection of Women, Explanation: Prevention and Redressal of sexual • Gangetic Dolphin has been spotted in Uttar harassment complaints. Pradesh, Bihar, Jharkhand, West Bengal, • Sexual harassment includes any one or Assam, Madhya Pradesh and Rajasthan. more of the following unwelcome acts or • The Gangetic dolphin is a mammal and is behaviour (directly or by implication) one of five species of river dolphin. namely: • The animal is popularly referred to as the • Physical contact and advances Susu. • A demand or request for sexual favours • It is found mainly in the Indian • Making sexually coloured remarks subcontinent, particularly in Ganga- • Showing pornography Brahmaputra-Meghna and Karnaphuli- • Any other unwelcome physical, verbal, or Sangu river systems. non-verbal conduct of sexual Nature. • The Gangetic Dolphins found in the Indian • The law is applicable to government territory account for 80 per cent of the total offices, the private sector, NGOs and the dolphin count. unorganised sector. • It has been spotted in Uttar Pradesh, Bihar, • A woman can be of any age, whether Jharkhand, West Bengal, Assam, Madhya employed or not, who alleges to have been Pradesh and Rajasthan. subjected to any act of sexual harassment. • The dolphin relies on a bio-sonar method to • It means the rights of all the women move around and catch their prey. working or visiting any workplace, in any • It feeds on several species of fishes, capacity, are protected under the Act. invertebrates etc.

10. Gangetic Dolphin is found in the states?

1. Uttar Pradesh

2. Odisha 3. West Bengal Which of the above statements is/are correct? www.iasgatewayy.com  Daily Current Affairs | Monthly 044- Magazines | Online Tests 5 | P a g e

26265326/9884421666/9884472636 UPSC OCTOBER - 2020 DAILY CURRENT AFFAIRS CLASS TEST – 20 1. In the Context to the Monsoon in India, which • The onset over Kerala, which marks the of the statement(s) given below is/are correct? arrival of the monsoon into mainland India, 1. IMD’s official definition on Normal will continue to be June 1. There is no Monsoon in India is designated as 89 cm of change in the final withdrawal date over rainfall. south India, i.e., 15th October”. So, 2. It also recently changed the official onset of statement 2 is correct. monsoon dates in various cities but the onset of monsoon in mainland India 2. Abraham Accord, recently seen in news is remains the same. referred to which of the following countries? Select the correct answer using the code given 1. Israel below: 2. UAE (a) 1 only 3. Bahrain (b) 2 only 4. Saudi Arabia (c) Both 1 and 2 Select the correct answer using the code given (d) Neither 1 nor 2 below: Answer: b) (a) 1 and 2 only Explanation: (b) 1, 2 and 4 only • IMD has also officially redefined the (c) 1, 2 and 3 only definition of what constitutes ‘normal’ (d) All the above rainfall and reduced it by 1 cm to 88 cm. Answer: c) So, statement 1 is not correct. Explanation: ✓ “Quantitatively, the monsoon seasonal • The Abraham Accord between Israel, the rainfall is likely to be 100% of the Long United Arab Emirates and Bahrain is Period Average (LPA) with a model mediated by the USA. It is the first Arab- error of ± 5%. The LPA of the season Israeli peace deal in 26 years. rainfall over the country as a whole for 3. Israel is bordered by which of the following the period 1961-2010 is 88 cm. sea(s)? • It also gave new dates for the monsoon’s 1. Mediterranean Sea onset in several cities as part of an update, 2. Red Sea which it said was essential for a variety of 3. Sea of Galilee economic activities ranging from 4. Dead Sea agricultural planning to power distribution.

www.iasgatewayy.com  Daily Current Affairs | Monthly 044- Magazines | Online Tests 1 | P a g e

26265326/9884421666/9884472636 UPSC OCTOBER - 2020 DAILY CURRENT AFFAIRS CLASS TEST – 20 Select the correct answer using the code given 5. Consider the following statements, with regard below: to Skinks: (a) 1 only 1. They are highly venomous and slow (b) 1 and 2 only moving reptile’s species. (c) 1, 2 and 3 only 2. It is found around homes, garages, and (d) All the above open spaces and around lakes. Answer: a) 3. India accounts for about one-fourth of the Explanation: total skink Population across the Globe. • Israel is bordered by Mediterranean Sea Which of the statements given above are only. Sea of Galilee is a fresh water lake in Incorrect? Israel and Dead Sea is a salt lake in Israel (a) 1 and 2 only • Mediterranean Sea is in the West of Israel. (b) 2 and 3 only • Red Sea bordering countries are Djibouti, (c) 1 and 3 only Eritrea, Saudi Arabia, Sudan, Egypt and (d) All the above Yemen. Answer: c) Explanation: 4. ‘One Country Two Systems approach’, was • The Zoological Survey of India (ZSI) sometimes seen in news is related to brought a study on skinks (a kind of lizard), (a) China and Hong Kong titled Skinks of India. (b) Israel and Palestine • Skinks are non-venomous, highly alert, (c) European Union and The Great Britain agile and fast moving and actively forage (d) Russia and Crimea for a variety of insects and small Answer: a) invertebrates. Explanation: • It is found around homes, garages, and • As per the policy, the Hong Kong and open spaces such as sparks and school Macau Special Administrative Regions playgrounds, and around lakes. (SAR), both former colonies, can have • With 1,602 species of skinks across the different economic and political systems world, It is the largest family of lizards. from that of mainland China, while being • India is home to less than 4% of the skinks part of the People’s Republic of China. across the Globe.

www.iasgatewayy.com  Daily Current Affairs | Monthly 044- Magazines | Online Tests 2 | P a g e

26265326/9884421666/9884472636 UPSC OCTOBER - 2020 DAILY CURRENT AFFAIRS CLASS TEST – 20 6. In context to BrahMos Missile, consider the 7. Pearl River estuary, sometime seen in news is following statements: located in which of the following water bodies? 1. It is a Hypersonic Missile. (a) Gulf of Thailand 2. It does not require further guidance after (b) Andaman Sea launch. (c) Philippine Sea 3. It can hit targets upto 450-600 Km. (d) South China Sea Which of the statements given above are Answer: d) Correct? Explanation: (a) 1 and 2 only • According to a recent report, Chinese pink (b) 2 and 3 only dolphins are making a comeback in the (c) 1 and 3 only Pearl River Estuary (PRE). (d) 1, 2 and 3 only • Pearl River Estuary includes Hong Kong, Answer: b) Macau as well as the mainland Chinese Explanation: cities of Shenzhen, Guangzhou and • BrahMos is a supersonic missile travelling Dongguan. Around 22 million people live at a speed of Mach 2.8 (nearly three times in the area. the speed of sound) whereas Hypersonic • The Pearl River Delta, the low-lying area speed is above Mach 5. surrounding the PRE where the Pearl River • It is a joint venture between the Defence flows into the South China Sea, is one of Research and Development Organisation the world’s most densely urbanised, of India (DRDO) and the NPOM of Russia. heavily industrialised and busiest shipping • It operates on the "Fire and Forget" lanes on Earth.

principle i.e it does not require further 8. Consider the following statements with respect guidance after launch. to the New START Treaty: • Initially, the range of BrahMos Missile was 1. It is a treaty for cooperation in the field of up to 290km. Its range has been recently nuclear energy. enhanced from 300 Km to 450-600 Km. 2. The treaty was signed between the United • The increasing the missile’s range became States, the Russian Federation and China. possible after India’s induction into the 3. New START replaced the Treaty of Missile Technology Control Regime Moscow (SORT). (MTCR) in June 2016. Which of the given statement/s is/are correct?

(a) 1 and 2 only

www.iasgatewayy.com  Daily Current Affairs | Monthly 044- Magazines | Online Tests 3 | P a g e

26265326/9884421666/9884472636 UPSC OCTOBER - 2020 DAILY CURRENT AFFAIRS CLASS TEST – 20 (b) 2 and 3 only Explanation: (c) 3 only • The oil-rich and gas-rich Reed Bank Island (d) None of the above is located in the South China Sea. The Answer: c) disputed island is claimed by the People’s Explanation: Republic of China and the Philippines. • New START is a nuclear arms reduction 10. Which of the following statements best treaty between the United States and the describes “Knesset”? Russian Federation with the formal name (a) It is the unicameral national legislature of of Measures for the Further Reduction and Israel. Limitation of Strategic Offensive Arms. (b) It is a special type of legal assembly, in the • It is the last major nuclear arms reduction traditional code of laws of the Pashtun accord between Russia and the U.S. people. • The New START deal was signed in April (c) It is the federal authority of the United Arab 2010 but went into force in February 2011. Emirates formed to represent the general • New START replaced the Treaty of Emirati people. Moscow (SORT), which was due to expire (d) It is a mass national gathering that brings in December 2012. together representatives from the various

9. Consider the following statements with respect ethnic, religious, and tribal communities in to ‘Reed Bank’ island: Afghanistan. 1. It is an oil-rich and gas-rich island located Answer: a) in the Caspian Sea. Explanation: 2. The disputed island is claimed by Turkey • The Knesset is the unicameral national and Greece. legislature of Israel. Which of the given statement/s is/are • As the legislative branch of the Israeli Incorrect? government, the Knesset passes all laws, (a) 1 only elects the President and Prime Minister (b) 2 only (although the latter is ceremonially (c) Both 1 and 2 appointed by the President), approves the (d) Neither 1 nor 2 cabinet, and supervises the work of the Answer: c) Government.

www.iasgatewayy.com  Daily Current Affairs | Monthly 044- Magazines | Online Tests 4 | P a g e

26265326/9884421666/9884472636 UPSC OCTOBER - 2020 DAILY CURRENT AFFAIRS CLASS TEST – 21 1. In context to Heeng Cultivation, consider the • During extreme weather, the plant can get following statements: dormant. 1. It is resin extracted from its plant roots. 2. Which of the following autonomous 2. It is endemic to Iran and Afghanistan. institutions are working under the Ministry of 3. It thrives in dry and cold desert conditions. Environment, Forest and Climate Change Which of the statements given above is/are (MoEFCC)? correct? 1. Wildlife Institute of India a) 1 and 3 only 2. Centre for Environment Education b) 2 and 3 only 3. Institute of Forest Management c) 1 and 2 only Select the correct answer using the code given d) 1, 2 and 3 below: Answer: d) (a) 1 and 3 only Explanation: (b) 2 and 3 only • Heeng is a herbaceous plant of the (c) 1 and 2 only umbelliferae family. It is a perennial plant (d) 1, 2 and 3 whose oleo gum resin is extracted from its Answer: d) thick roots and rhizome. The plant stores Explanation: most of its nutrients inside its deep fleshy • The Ministry of Finance has recommended roots. Hence, statement 1 is correct. that the Ministry of Environment, Forest • It is endemic to Iran and Afghanistan, and Climate Change (MoEFCC) which are also the main global suppliers of “disengage” from five autonomous it. Heeng is not cultivated in India. Hence, institutions working under it and merge two statement 2 is correct. others. • It thrives in dry and cold desert conditions. • These autonomous institutions are Wildlife The plant can withstand a maximum Institute of India (WII-Dehradun), Indian temperature between 35 and 40 degree, Institute of Forest Management (IIFM- whereas during winters, it can survive in Bhopal), Indian Plywood Industries temperatures up to minus 4 degree. Hence, Research & Training Institute (Bengaluru), statement 3 is correct. CPR Environmental Education Centre • Regions with sandy soil, very little (Chennai) and Centre for Environment moisture and annual rainfall of not more Education (Ahmedabad). than 200mm are considered conducive for

heeng cultivation. www.iasgatewayy.com  Daily Current Affairs | Monthly 044- Magazines | Online Tests 1 | P a g e

26265326/9884421666/9884472636 UPSC OCTOBER - 2020 DAILY CURRENT AFFAIRS CLASS TEST – 21 3. With reference to the Ayushman Sahakar healthcare related activities would be able scheme, consider the following statements: to access the NCDC fund. 1. It is launched to assist cooperatives in • The scheme covers establishment, creation of healthcare infrastructure in the modernization, expansion, repairs, country. renovation of hospital and healthcare and 2. National Bank for Agriculture and Rural education infrastructure. Development will extend term loans to • NCDC assistance will flow either through cooperatives. the State Governments/ UT 3. The scheme provides interest subvention to Administrations or directly to the eligible Women Majority Cooperatives. cooperatives. Which of the statements given above are • The scheme also provides working capital correct? and margin money to meet operational (a) 1 and 2only requirements. (b) 1 and 3 only • Further, the scheme provides interest (c) 2 and 3 only subvention of 1% to women majority (d) 1, 2 and 3 cooperatives. Answer: b) Explanation: 4. Consider the following statements: • The Ministry of Agriculture and Farmers 1. Though states could come out with their Welfare has launched Ayushman Sahakar, own Bills to some extent to override the a scheme to assist cooperatives in creation statutes of passed by the Parliament, none of healthcare infrastructure in the country. of those Bills would be effective unless the Hence, statement 1 is correct. President accords his consent to such Bills. • Ayushman Sahakar is formulated by 2. It’s the sole prerogative of the President National Cooperative Development whether to sign the state Bills or not. Corporation (NCDC). Which of the statement(s) given above is/are • NCDC would extend term loans to correct? prospective cooperatives to the tune of (a) 1 only Rs.10,000 crore in the coming years. (b) 2 only Hence, statement 2 is not correct. (c) Both 1 and 2 • Any Cooperative Society with suitable (d) Neither 1 nor 2 provision in its byelaws to undertake Answer: c)

www.iasgatewayy.com  Daily Current Affairs | Monthly 044- Magazines | Online Tests 2 | P a g e

26265326/9884421666/9884472636 UPSC OCTOBER - 2020 DAILY CURRENT AFFAIRS CLASS TEST – 21 Explanation: villages and improve the capacities of • Both the statements are correct. tanks. Which of the statement(s) given above is/are 5. ‘Mission Kakatiya’, sometimes seen in news is correct? related to (a) 1 only (a) Rejuvenation of water tanks and other (b) 1 and 2 only water storage structures in Telangana. (c) 1 and 3 only (b) Reviving the glory of Kakatiya Dynasty by (d) 2 and 3 only Archaeological Department Answer: c) (c) Reconstruction of Damaged Temples in the Explanation: Kakatiya Dynasty Region • The Kaleshwaram Lift Irrigation Scheme (d) Building Government Schools and of Telangana is a multi-purpose irrigation Colleges with latest infrastructure in project on the Godavari River in Telangana. Kaleshwaram, Bhupalpally, Telangana. So, Answer: a) statement 1 is correct. Explanation: • The project starts at the confluence point of • It is a flagship programme launched by the Pranahita River and Godavari River. So, Government of Telangana which aims at statement 2 is not correct. rejuvenation of water tanks and other water • The project will also support Mission storage structures to provide assistance and Kakatiya and Mission Bhagiratha schemes help to the small and marginal farmers of designed to provide drinking water to many the state. villages and improve the capacities of tanks. So, statement 3 is correct.

6. Consider the following statements: 1. The Kaleshwaram Lift Irrigation Scheme 7. With reference to the differences between the of Telangana is a multi-purpose irrigation National Green Tribunal (NGT) and the project on the Godavari River in Central Pollution Control Board (CPCB), Kaleshwaram, Bhupalpally, Telangana. consider the following statements: 2. The project starts at the confluence point of 1. The NGT has been established by an Act Krishna River and Godavari River. whereas the CPCB has been created by an 3. The project will also support Mission executive order of the Government. Kakatiya and Mission Bhagiratha schemes 2. The NGT provides environmental justice designed to provide drinking water to many and helps reduce the burden of litigation in www.iasgatewayy.com  Daily Current Affairs | Monthly 044- Magazines | Online Tests 3 | P a g e

26265326/9884421666/9884472636 UPSC OCTOBER - 2020 DAILY CURRENT AFFAIRS CLASS TEST – 21 the higher courts whereas the CPCB 9. Which of the following statement/s is/are promotes cleanliness of streams and wells, correct? and aims to improve the quality of air in the 1. Currently, India is the largest sugar- country. producing country in the world. Which of the statement(s) given above is/are 2. Maharashtra is the largest sugar-producing correct? state in India. (a) 1 only Options: (b) 2 only (a) 1 only (c) Both 1 and 2 (b) 2 only (d) Neither 1 nor 2 (c) Both 1 and 2 Answer: b) (d) Neither 1 nor 2 Explanation: Answer: d) • The NGT was established by the National Explanation: Green Tribunal Act, 2010 whereas the • Brazil has historically been the world’s CPCB has been created by the Water act, largest sugar producer but ceded its place to 1974. So, statement 1 is not correct. India in the 2018/2019 crop year. Currently, however, Brazil continues to 8. The ‘Mutual Logistics Support Agreement’ remain the largest sugar producer in the was signed by India with which of the world. India is the world’s second-largest following countries? producer after Brazil. (a) Australia • The major sugar-producing states in India (b) Russia include Uttar Pradesh and Maharashtra (c) Japan with UP accounting for the largest (d) The U.S. production. Answer: a) Explanation: 10. The 90-90-90 strategy is associated with which • In June 2020, India and Australia signed of the following? the Mutual Logistics Support Agreement (a) Green House Gas emission mitigation (MLSA) and also announced a joint (b) Poverty alleviation measure declaration on a shared vision for maritime (c) AIDS Epidemic cooperation in the Indo-Pacific. (d) Achieving land degradation neutrality

Answer: c)

www.iasgatewayy.com  Daily Current Affairs | Monthly 044- Magazines | Online Tests 4 | P a g e

26265326/9884421666/9884472636 UPSC OCTOBER - 2020 DAILY CURRENT AFFAIRS CLASS TEST – 21 Explanation: • 90-90-90 marks treatment targets to help end the AIDS epidemic. • Although many strategies will be needed to halt the AIDS epidemic, it will be impossible to end the epidemic without bringing HIV treatment to all who need it. • There are the envisaged goals of ending the AIDS epidemic by 2030. • The main targets under the 90-90-90 strategy include the following: • By 2020, 90% of all people living with HIV will know their HIV status. • By 2020, 90% of all people with diagnosed HIV infection will receive sustained antiretroviral therapy. • By 2020, 90% of all people receiving antiretroviral therapy will have viral suppression.

www.iasgatewayy.com  Daily Current Affairs | Monthly 044- Magazines | Online Tests 5 | P a g e

26265326/9884421666/9884472636 UPSC OCTOBER - 2020 DAILY CURRENT AFFAIRS CLASS TEST – 22 1. With respect to the Association of Democratic 2. With respect to the Central Drugs Standard Reforms (ADR), consider the following Control Organisation (CDSCO), consider the statements: following statements: 1. The ADR is a non-partisan, non- 1. The Central Drugs Standard Control governmental organization which works in Organisation (CDSCO) under Directorate the area of electoral and political reforms. General of Health Services, Ministry of 2. ADR aims at bringing transparency and Health & Family Welfare is the National accountability in Indian politics and Regulatory Authority (NRA) of India. reducing the influence of Money and 2. It regulates control over the import of Muscle Power in Elections. drugs, approval of new drugs, clinical trials Which of the statement(s) given above is/are and approves licences to all kind of drugs correct? in India. (a) 1 only Which of the statement(s) given above is/are (b) 2 only correct? (c) Both 1 and 2 (a) 1 only (d) Neither 1 nor 2 (b) 2 only Answer: c) (c) Both 1 and 2 Explanation: (d) Neither 1 nor 2 • The ADR is a non-partisan, non- Answer: a) governmental organization which works in Explanation: the area of electoral and political reforms. • The Central Drugs Standard Control So, statement 1 is correct. Organisation (CDSCO) under Directorate • ADR aims at bringing transparency and General of Health Services, Ministry of accountability in Indian politics and Health & Family Welfare is the National reducing the influence of money and Regulatory Authority (NRA) of India. So, muscle power in elections. So, statement 2 statement 1 is correct. is correct. • It is the Central Drug Authority for • National Election Watch (NEW) is a discharging functions assigned to the conglomeration of over 1200 organizations Central Government under the Drugs and across the country. Cosmetics Act. • Major Functions are as follows:

www.iasgatewayy.com  Daily Current Affairs | Monthly 044- Magazines | Online Tests 1 | P a g e

26265326/9884421666/9884472636 UPSC OCTOBER - 2020 DAILY CURRENT AFFAIRS CLASS TEST – 22 ✓ Regulatory control over the import of 2. Freedom of speech and expression is drugs, approval of new drugs and clinical recognized as a fundamental right in trials. whatever medium it is exercised under the ✓ Approval of certain licences as Central Constitution of India and it cannot be Licence Approving Authority. abrogated under any circumstances. • Drug Controller General of India Which of the statement(s) given above is/are (DCGI): DCGI is responsible for correct? approval of licences of specified (a) 1 only categories of drugs such as blood and (b) 2 only blood products, IV fluids, and (c) Both 1 and 2 sera in India. It comes under the Ministry (d) Neither 1 nor 2 of Health & Family Welfare. So, statement Answer: a) 2 is correct. Explanation: • The Chief Justice of Tripura High Court in 3. Consider the following pairs: his order broadly remarked that posting on Pass State/UT social media was tantamount to a 1. Nathu La - Sikkim “fundamental right” applicable to all 2. Rohtang La - Himachal Pradesh citizens, including government employees. 3. Zoji La - Jammu & Kashmir So, statement 1 is correct Which of the pairs given above are correct? • It is seen that freedom of speech and (a) 1 and 2 only expression is recognized as a fundamental (b) 2 and 3 only right in whatever medium it is exercised (c) 1 and 3 only under the Constitution of India and other (d) 1, 2 and 3 international documents. Answer: a) • Freedom of Speech and Expression can be Explanation: abrogated during the National Emergency. • Zoji La pass in present in the UT of So, statement 2 is not correct. Ladakh. 5. Consider the following statements: 4. Consider the following statements: 1. Global Innovation Index (GII) is published 1. Posting on social media is a fundamental by the World Economic Forum. right applicable to all citizens, including 2. India has shown no improvement in its GII government employees. ranking for the last 5 years. www.iasgatewayy.com  Daily Current Affairs | Monthly 044- Magazines | Online Tests 2 | P a g e

26265326/9884421666/9884472636 UPSC OCTOBER - 2020 DAILY CURRENT AFFAIRS CLASS TEST – 22 3. India has never been among the top 50 in Which of the given statement/s is/are correct? the Global Innovation Index. (a) 1 only Which of the given statement/s is/are correct? (b) 1 and 2 only (a) 1 only (c) 2 and 3 only (b) 2 and 3 only (d) 1, 2 and 3 (c) 1, 2 and 3 Answer: d) (d) None of the above Explanation: Answer: d) • FELUDA has been so named because Explanation: technically it is an acronym and stands for • Global Innovation Index is published by a FNCAS9 Editor Linked Uniform Detection specialized agency of the United Nations – Assay. the World Intellectual Property • It is a paper-strip test for SARS-CoV-2 Organisation (WIPO) in association with diagnosis. Cornell University and graduate business • The test uses an indigenously-developed, school INSEAD. cutting-edge CRISPR technology. CRISPR • In the Global Innovation Index 2020, India is a genome-editing technology to diagnose is at the 48th position. diseases. • India has shown improvement in its • The test will deploy a specially adapted innovation ranking for the last 5 years. Cas9 protein. • India ranks in the top 15 in indicators such as the Information and Communication 7. Consider the following statements with respect Technology (ICT) services exports, to Kaleshwaram Lift Irrigation Project: government online services, graduates in 1. It is in Andhra Pradesh. science and engineering, and Research and 2. It is being built at the confluence of the Development-intensive global companies. Pranhita and Godavari rivers. Which of the given statement/s is/are 6. Consider the following statements with respect Incorrect? to FELUDA: (a) 1 only 1. It is a paper-strip test for SARS-CoV-2 (b) 2 only diagnosis. (c) Both 1 and 2 2. The test uses an indigenously-developed, (d) Neither 1 nor 2 cutting-edge CRISPR technology. Answer: a) 3. The test will deploy Cas9 protein.

www.iasgatewayy.com  Daily Current Affairs | Monthly 044- Magazines | Online Tests 3 | P a g e

26265326/9884421666/9884472636 UPSC OCTOBER - 2020 DAILY CURRENT AFFAIRS CLASS TEST – 22 Explanation: China. The strait is currently part of the • Kaleshwaram Lift Irrigation Project is South China Sea and connects to the East being constructed in Telangana. China Sea to the north. • It is being built at the confluence of • The Luzon Strait is the strait between Pranhita and Godavari Rivers. Taiwan and Luzon island of the Philippines. The strait thereby connects the 8. Consider the following statements about Philippine Sea to the South China Sea. Taiwan:

1. Taiwan became the newest state to join the 9. With reference to Nazca lines, consider the United Nations General Assembly. following statements: 2. Taiwan Strait is also known as the Formosa 1. These are groups of geoglyphs known for Strait the depictions of larger-than-life animals, 3. The Luzon Strait is the strait between plants and imaginary beings. Taiwan and Luzon island of the Philippines 2. This is on the leeward side of Andes Which of the statements given above are mountain in Argentina. correct? Which of the statements given above is/are (a) 1 and 2 only correct? (b) 2 and 3 only (a) 1 only (c) 1 and 3 only (b) 2 only (d) All the above (c) Both 1 and 2 Answer: b) (d) Neither 1 nor 2 Explanation: Answer: a) • Taiwan - the Republic of China (ROC), Explanation: home to twenty-three million people, is an • The Nazca Lines are a group of geoglyphs island off the southern coast of China that known for the depictions of larger-than-life has been governed independently from animals, plants and imaginary beings. mainland China since 1949. Hence, statement 1 is correct. • Taiwan is the most populous state that is • Geoglyphs are the large designs made on not a member of the United Nations and the the ground by creators using elements of largest economy outside the UN. the landscape such as stones, gravel, dirt or • The Taiwan Strait, also known as the lumber. Formosa Strait, is a 180-kilometer wide strait separating Taiwan and mainland www.iasgatewayy.com  Daily Current Affairs | Monthly 044- Magazines | Online Tests 4 | P a g e

26265326/9884421666/9884472636 UPSC OCTOBER - 2020 DAILY CURRENT AFFAIRS CLASS TEST – 22 • These are believed to be the greatest known • It continues the bipartisan process of archaeological enigma, owing to their size, verifiably reducing U.S. and Russian continuity, nature and quality. strategic nuclear arsenals by limiting both • The site is present in Peru. Recently, a giant sides to 700 strategic launchers and 1,550 cat geoglyph was discovered on a hill at operational warheads. this Nazca Lines site in Peru. • Recently, the Russian President has proposed extending by one year the New 10. Consider the following statements, with regard START (Strategic Arms Reduction Treaty) to New START Treaty: between the USA and Russia expiring in 1. It is a treaty between the United States of February 2021. America and the Russian Federation for the • Intermediate-Range Nuclear Forces Treaty reduction of strategic offensive arms. is another treaty that was signed during the 2. The United States of America withdrew Cold War. from the Intermediate-Range Nuclear • It was a nuclear arms-control accord Forces Treaty in the year 2019. reached by the United States and the Soviet Which of the statements given above is/are Union in 1987 in which the two nations correct? agreed to eliminate their stocks of (a) 1 only intermediate-range and shorter-range (or (b) 2 only “medium-range”) land-based missiles (c) Both 1 and 2 (which could carry nuclear warheads). (d) Neither 1 nor 2 • The United States withdrew from the Answer: c) Treaty on 2nd August 2019. Explanation:

• The New START Treaty is a treaty

between the United States of America and

the Russian Federation on measures for the further reduction and limitation of strategic offensive arms. Hence, statement 1 is correct. • It is a successor to the START framework of 1991 (at the end of the Cold War) that limited both sides to 1,600 strategic delivery vehicles and 6,000 warheads. www.iasgatewayy.com  Daily Current Affairs | Monthly 044- Magazines | Online Tests 5 | P a g e

26265326/9884421666/9884472636 UPSC OCTOBER - 2020 DAILY CURRENT AFFAIRS CLASS TEST – 23 1. In context to Production-Linked Incentive • The incentivised sectors may be (PLI) scheme, consider the following automobile, networking products, food statements: processing, advanced Chemistry and solar 1. It is an outcome and output-oriented PV Manufacturing. scheme. 2. It incentivizes are only for the sunrise 2. Consider the following statements, with regard sectors. to OSIRIS-REx Mission 3. It will give cash incentives for five to Seven 1. It has recently touched down the surface of Years asteroid Bennu to collect the rock samples. Which of the statements given above is/are 2. The mission is a ten-year-long voyage and Correct? will return back to Earth in 2026. (a) 1 and 2 only Which of the statements given above is/are (b) 2 and 3 only correct? (c) 1 and 3 only (a) 1 only (d) All the above (b) 2 only Answer: c) (c) Both 1 and 2 Explanation: (d) Neither 1 nor 2 • The government will soon extend the Answer: a) Production-Linked Incentive (PLI) scheme Explanation: to eight more sectors to boost domestic • Recently, NASA’s OSIRIS-REx spacecraft manufacturing. briefly touched down on the surface of • PLI Scheme is an outcome- and output- asteroid Bennu to collect rock and dust oriented scheme where incentives will be samples. paid only if the manufacturers make the • Bennu is an ancient asteroid, currently goods. more than 200 million miles from Earth. • This scheme will give cash incentives for • So far, it is known that this asteroid is a B- five to seven years and all the sunrise and type asteroid, implying that it contains important sectors are proposed to be significant amounts of carbon and various covered in this. Sunrise sectors are other minerals. promising sectors but they may need • It is the United States’ first asteroid sample support in the Initial Stage. return mission, aiming to collect and carry a pristine, unaltered sample from an asteroid back to earth for scientific study. www.iasgatewayy.com  Daily Current Affairs | Monthly 044- Magazines | Online Tests 1 | P a g e

26265326/9884421666/9884472636 UPSC OCTOBER - 2020 DAILY CURRENT AFFAIRS CLASS TEST – 23 • The OSIRIS-REx (Origins, Spectral under a single commander, for Interpretation, Resource Identification, geographical theatres (areas) that are of Security, Regolith Explorer) spacecraft strategic and security concern. was launched in 2016 for the journey to • The commander of such a force will be able Bennu. to bear all resources at his disposal — from • The mission is essentially a seven-year- the Army, the Indian Air Force, and the long voyage and will conclude when at Navy — with seamless efficacy. least 60 grams of samples are delivered • Andaman and Nicobar Command is the back to the Earth (in 2023). first and only integrated theatre command in India, formed in 2001 after the Kargil 3. Consider the following, with reference to war. Theatre Command in India: • The Shekatkar committee has 1. It is a unified command of the three armed recommended the creation of 3 integrated services. theatre commands — northern for the 2. India’s first theatre command was China border, western for the Pakistan established at Andaman and Nicobar border, and southern for the maritime role. Island. 3. There are three integrated theatre 4. State of Global Air 2020 is released by commands in India. (a) Health Effects Institute Which of the statements given above are not (b) UNEP correct? (c) Greenpeace (a) 1 and 3 only (d) WEF (b) 2 and 3 only Answer: a) (c) 1 and 2 only Explanation: (d) 1, 2 and 3 • Recently, a global study, State of Global Answer: c) Air 2020 (SoGA 2020) has been released Explanation: by the Health Effects Institute (HEI).

• A theatre command is an organisational structure designed to control all military 5. Consider the following statements regarding assets in a theatre of war to achieve military Unlawful Activities (Prevention) Act (UAPA), effects. 1967: • An integrated theatre command envisages a 1. Under UAPA, both Indian and foreign unified command of the three Services, nationals and can be charged even if crime www.iasgatewayy.com  Daily Current Affairs | Monthly 044- Magazines | Online Tests 2 | P a g e

26265326/9884421666/9884472636 UPSC OCTOBER - 2020 DAILY CURRENT AFFAIRS CLASS TEST – 23 is committed on a foreign land, outside Explanation: India. • Maharashtra gets its first Ramsar site 2. Under the UAPA, the investigating agency (Nandur Madhameshwar), Punjab can file a charge sheet in maximum 180 added 3 more (Keshopur-Miani, Beas days after the arrests and the duration can Conservation Reserve, Nangal) and UP be extended further after intimating the has added 6 more (Nawabganj, Parvati court. Agra, Saman, Samaspur, Sandi and Which of the statement(s) given above is/are SarsaiNawar). So, only 1st pair is correct? incorrect. (a) 1 only • With this, the numbers of Ramsar sites in (b) 2 only India are now 37 and the surface area (c) Both 1 and 2 covered by these sites is now 1,067,939 (d) Neither 1 nor 2 hectares. Answer: c) • The Ramsar Convention was signed in Explanation: 1971 to maintain an international network • Both the statements are correct. of wetlands which are important for the conservation of global biological diversity. 6. Consider the following pairs: • Wetlands declared as Ramsar sites are Ramsar Sites State protected under strict guidelines of the 1. Nandur Madhameshwar Convention. - Madhya Pradesh 2. Beas Conservation Reserve 7. With respect to the Asiatic Cheetah, consider - Punjab the following statements: 3. Nawabganj 1. The cheetah has been classified as - Uttar Pradesh Vulnerable by the IUCN. Which of the pairs given above is/are 2. It inhabits a variety of mostly arid habitats Incorrectly matched? like dry forests, scrub forests, and (a) 1 only savannahs. (b) 1 and 2 only 3. The Shahgarh bulge landscape in (c) 3 only Jaisalmer, Rajasthan is one of the sites (d) 1 and 3 only chosen for reintroduction of the species in Answer: a) India.

www.iasgatewayy.com  Daily Current Affairs | Monthly 044- Magazines | Online Tests 3 | P a g e

26265326/9884421666/9884472636 UPSC OCTOBER - 2020 DAILY CURRENT AFFAIRS CLASS TEST – 23 Which of the statements given above is/are 8. Consider the following statements: correct? 1. Offer-for-sale (OFS) allows an existing (a) 1 only Shareholder to dilute their stake. (b) 1 and 2 only 2. Follow-on public offer allows unlisted (c) 2 and 3 only companies to raise capital. (d) 1, 2 and 3 3. Initial Public Offer results in the Transfer Answer: c) of ownership. Explanation: Which of the above statements is/are correct? • Cheetahs (Acinonyx jubatus) are large cat (a) 1 and 2 only of the subfamily Felinae and are considered (b) 2 and 3 only as the fastest land animal. (c) 1 and 3 only • The 4 subspecies of cheetahs are Southeast (d) 1, 2 and 3 African cheetah, Asiatic cheetah, Answer: a) Northeast African cheetah and Explanation: Northwest African cheetah. Initial Public Offer: • They are found in North, Southern and East • A company issuing shares for the first time Africa, and a few localities in Iran. to the public is called an Initial Public Offer • It inhabits a variety of mostly arid habitats (IPO). like dry forests, scrub forests, and • It allows an unlisted company to raise fresh savannahs. The African Cheetah has been funds from the primary market and also list classified as Vulnerable by the IUCN; the stock in the stock exchange. while the Asiatic Cheetah (found only in • An IPO necessarily leads to the expansion Iran) has been classified as Critically of the capital of the company. Endangered and both are listed under Follow-on public offer (FPO): Appendix I of CITES (Convention on • A slight variant of the Initial Public International Trade in Endangered Offering. Species). The Nauradehi Wildlife • FPO allows a listed company to raise Sanctuary and Kuno-Palpur Wildlife additional capital through from the primary Sanctuaries in Madhya Pradesh and the market. Shahgarh bulge landscape in Jaisalmer, • Helps expand the capital base. Rajasthan are the three sites chosen for reintroduction of the Species in India.

www.iasgatewayy.com  Daily Current Affairs | Monthly 044- Magazines | Online Tests 4 | P a g e

26265326/9884421666/9884472636 UPSC OCTOBER - 2020 DAILY CURRENT AFFAIRS CLASS TEST – 23 Offer-for-sale (OFS): (b) Only 2 • In an OFS, an existing shareholder dilutes (c) Both 1 and 2 its stake through the primary market. (d) Neither 1 nor 2 • OFS does not result in the fresh raising of Answer: a) funds but only results in a transfer of Explanation: ownership from one shareholder to another. Exercise Suraksha Kavach: • Since an OFS does not involve the raising • Exercise Suraksha Kavach is a joint of fresh capital and is purely a change of exercise of the Indian Army and ownership, the regulatory compliance is Maharashtra Police. much lower than in case of IPOs and FPOs. • It was held near Pune, Maharashtra. • The exercise aimed to harmonize the drills 9. The State of Climate Services 2020 report was and procedures of both Army and Police for prepared by? activating anti-terrorist Quick Reaction (a) World Meteorological Organization Teams (QRTs) to counter any terrorist (b) World Wildlife Fund actions in Pune. (c) World Economic Forum (d) Greenpeace Answer: a) Explanation: • State of Climate Services 2020 report was prepared by the World Meteorological Organization (WMO).

10. With respect to Exercise Suraksha Kavach, consider the following statements: 1. Exercise Suraksha Kavach is a joint exercise of the Indian Army and Nepal 2. The exercise aimed to Harmonize Procedures to deal with terrorism-related Threats. Which of the above statements is/are Incorrect? (a) Only 1 www.iasgatewayy.com  Daily Current Affairs | Monthly 044- Magazines | Online Tests 5 | P a g e

26265326/9884421666/9884472636 UPSC OCTOBER - 2020 DAILY CURRENT AFFAIRS CLASS TEST – 24 1. In context to ‘monoclonal Antibodies (mAbs)’, • Encouraging results for Covid-19 antibody consider the following statements: treatment have emerged from preclinical 1. These are man-made proteins that act like a research and from initial clinical trials. human antibody in the . 2. The Global Wealth Report is released by which 2. They can be used to carry drugs, toxins, or of the following organisations? radioactive substances directly to affected (a) World Economic Forum cells (b) International Monetary Fund Which of the statements given above is/are (c) Credit Suisse Group correct? (d) World Bank (a) 1 only Answer: c) (b) 2 only Explanation: (c) Both 1 and 2 • Recently, Credit Suisse has released the (d) Neither 1 nor 2 Global Wealth Report 2020. Credit Suisse Answer: c) is a financial services company based in Explanation: Switzerland. • Monoclonal Antibodies are man-made • The Credit Suisse Global Wealth Report proteins that act like a human antibody in provides the most comprehensive and up- the immune system. They are made by to-date coverage of information on cloning a unique white blood cell. household wealth worldwide. • They are designed to perform many roles, • Highlights of Global Wealth Report 2020 like they can be used to carry drugs, toxins, or radioactive substances directly to • In 2019, total global wealth rose by USD affected cells. mAbs are used to treat many 36.3 trillion and wealth per adult reached diseases, including some types of cancer. USD 77,309, which is 8.5% more when compared to 2018. • Recently, neutralising monoclonal antibodies (mAbs) against SARS-CoV-2 • In India, wealth inequality remains quite were co-invented by IAVI and Scripps high.There is considerable poverty Research. Hence, statement 2 is not correct. reflected in the fact that 73% of the adult population had wealth below USD 10,000 • They are widely considered to be at the end of 2019. promising candidates for Covid-19 treatment and prevention. • At the other extreme, a small fraction of the population (2.3% of adults) had a net worth over USD 1,00,000. www.iasgatewayy.com  Daily Current Affairs | Monthly 044- Magazines | Online Tests 1 | P a g e

26265326/9884421666/9884472636 UPSC OCTOBER - 2020 DAILY CURRENT AFFAIRS CLASS TEST – 24 • Further, between January and April 2020, 4. ’27 Point Action Plan’, by FATF sometimes unemployment rates approximately tripled seen in news is related to which country? in India to 24%. (a) Pakistan (h) Iran 3. Which of the following Missiles are included (i) Israel in Integrated Guided Missile Development (j) Afghanistan Program (IGMDP)? Answer: a) 1. Prithvi Explanation: 2. Agni • The International Co-operation Review 3. Trisul Group (ICRG) of the Financial Action Select the correct answer using the code given Task Force (FATF) recently below: recommended that Pakistan should be (a) 1 and 3 only retained on the “Grey List” as it had failed (e) 2 and 3 only to implement the 27 Point Action Plan. (f) 1 and 2 only • The Financial Action Task Force (FATF) is (g) 1, 2 and 3 an inter-governmental body established Answer: d) in 1989 during the G7 Summit in Paris. Explanation: • The objectives of the FATF are to set • IGMDP (Integrated Guided Missile standards and promote effective Development Program) was conceived by implementation of legal, regulatory and Dr. A.P.J. Abdul Kalam to enable India operational measures for combating attain self-sufficiency in the field of missile money laundering, terrorist financing technology. The 5 missiles (P-A-T-N-A) and other related threats to the integrity developed under this program are: of the international financial system. • Prithvi: Short range surface to surface • FATF has two lists: ballistic missile. 1. Grey List: Countries that are considered • Agni: Ballistic missiles with different safe haven for supporting terror funding ranges, i.e. Agni (1,2,3,4,5) and money laundering are put in the FATF • Trishul: Short-range low-level surface to grey list. This inclusion serves as a air missile. warning to the country that it may enter rd • Nag: 3 generation anti-tank missile. the blacklist. • Akash: Medium range surface to air missile. www.iasgatewayy.com  Daily Current Affairs | Monthly 044- Magazines | Online Tests 2 | P a g e

26265326/9884421666/9884472636 UPSC OCTOBER - 2020 DAILY CURRENT AFFAIRS CLASS TEST – 24 2. Black List: Countries known as Non- • The CPI calculates the difference in the Cooperative Countries or Territories price of commodities and services such as (NCCTs) are put in the blacklist. These food, medical care, education, electronics countries support terror funding and etc, which Indian consumers buy for use. money laundering activities. The FATF • The CPI has several sub-groups including revises the blacklist regularly, adding or food and beverages, fuel and light, housing deleting entries. and clothing, bedding and footwear. • At present, India has five consumer price 5. With respect to Consumer Price Index (CPI), indexes (CPIs), three of which are working- consider the following statements: class specific. These are: 1. CPI measures price changes from the 1. CPI for Industrial Workers (IW). perspective of a retail buyer and is released 2. CPI for Agricultural Labourer (AL). by the National Statistical Office (NSO). 3. CPI for Rural Labourer (RL). So, 2. At present, India has three consumer price statement 2 is not correct. indexes (CPIs), which are mainly working- • These three indexes are compiled by the class specific. Labour Bureau in the Ministry of Labour 3. These three indexes are compiled by the and Employment. So, statement 3 is not National Statistical Office (NSO) in the correct. Ministry of Statistics and Programme • CPIs AL and RL are used to fix minimum Implementation. wages of agricultural labourers and rural Which of the statements given above are not unskilled employees. correct? • The Labour Bureau is also expected to (a) 1 and 2 only bring out the new series of the CPI- AL/RL, (a) 1 and 3 only which currently has the base year of 1986- (b) 2 and 3 only 87 by August 2021. (c) 1, 2 and 3 • The other two are CPI-Urban and CPI Answer: c) Rural. These two indexes are compiled by Explanation: the National Statistical Office (NSO) in the • CPI measures price changes from the Ministry of Statistics and Programme perspective of a retail buyer. It is released Implementation. by the National Statistical Office (NSO). • The combined rise in retail prices is So, statement 1 is correct. captured by CPI Combined.

www.iasgatewayy.com  Daily Current Affairs | Monthly 044- Magazines | Online Tests 3 | P a g e

26265326/9884421666/9884472636 UPSC OCTOBER - 2020 DAILY CURRENT AFFAIRS CLASS TEST – 24 • A committee set up by the National • NWDA is the agency of the Union Ministry Statistical Commission (NSC) in 2005 of Water Resources. It was set up in July, under the recommendation of Dr C 1982 as autonomous society under the Rangarajan Commission, has suggested Societies Registration Act, 1860. that CPI-Rural and CPI-Urban could be a • It was established to carry out the water substitute for CPI-AL/RL and CPI-IW, balance and other studies on a scientific and respectively. realistic basis for optimum utilisation of Water Resources of the peninsular rivers 6. Consider the following statements: system. 1. The National Water Development Agency • Besides it was tasked to prepare feasibility is responsible for the formulation of reports and give concrete shape to proposals of the linking of rivers. Peninsular Rivers Development 2. NWDA was tasked to prepare feasibility Component of National Perspective. So, reports and give concrete shape to statement 2 is correct. Peninsular Rivers Development • In 1990, it was also entrusted with the task Component of National Perspective. of Himalayan Rivers Development 3. The task of Himalayan Rivers Component of National Perspectives. Development Component of National • Recently, the functions of NWDA have Perspectives is done by the National been further modified and the work of Interlinking of Rivers Authority (NIRA). preparation of detailed Project Reports Which of the statement(s) given above is/are (DPR) of various link proposals and Pre- correct? feasibility Reports. (a) 1 only • It has been also entrusted to prepare (b) 1 and 2 only feasibility reports of intra-State links as (c) 3 only proposed by the States. (d) 1 and 3 only • The Central government is just working on Answer: b) the establishment of an exclusive body Explanation: named National Interlinking of Rivers • The National Water Development Authority (NIRA) to implement projects Agency (NWDA) is responsible for the for linking rivers. So, statement 3 is not formulation of proposals of the linking of correct. rivers. So, statement 1 is correct.

www.iasgatewayy.com  Daily Current Affairs | Monthly 044- Magazines | Online Tests 4 | P a g e

26265326/9884421666/9884472636 UPSC OCTOBER - 2020 DAILY CURRENT AFFAIRS CLASS TEST – 24 7. With respect to the National Council for Which of the given statement/s is/are Transgender Persons, consider the following Incorrect? statements: (a) 1 only 1. The Council was constituted the National (b) 1 and 2 only Council for Transgender Persons, under the (c) 3 only Transgender Persons (Protection of Rights) (d) None of the above Act, 2019. Answer: d) 2. Its chairperson will be the Union Minister Explanation: of the Ministry of Social Justice and • All the statements are correct. Empowerment. 3. It advises the Central government on the 9. Consider the following statements: formulation of policies, programmes, 1. The Disorders Inquiry Committee of 1919 legislation and projects with respect to was appointed by the Secretary of State for transgender persons. India, Edwin Montagu. Which of the statements given above are 2. The committee did not have Indian correct? representation. (a) 1 and 2 only 3. The committee was appointed in the (b) 2 and 3 only aftermath of the Jallianwala Bagh (c) 1 and 3 only Massacre. (d) 1, 2 and 3 4. It was tasked with the investigation of the Answer: d) disturbances in Bombay, Delhi and Punjab Explanation: in 1919. Which of the given statement/s is/are correct? • All the three statements are correct. (a) 1, 2 and 4 only 8. Consider the following statements with respect (b) 1, 3 and 4 only to Organization of American States (OAS): (c) 2 and 3 only 1. OAS brings together all 35 independent (d) 1, 2, 3 and 4 states of the Americas. Answer: b) 2. European Union, India, China and Pakistan Explanation: are permanent observers of OAS. • The Disorders Inquiry Committee of 1919 3. The four main pillars of the organisation or the Hunter Committee of 1919 was are democracy, human rights, security, and appointed by the Secretary of State for Development. India, Edwin Montagu. www.iasgatewayy.com  Daily Current Affairs | Monthly 044- Magazines | Online Tests 5 | P a g e

26265326/9884421666/9884472636 UPSC OCTOBER - 2020 DAILY CURRENT AFFAIRS CLASS TEST – 24 • The committee was appointed in the • The ship has up to 90% indigenous content aftermath of the Jallianwala Bagh and the use of carbon composites for the Massacre. superstructure is a commendable feat • It was tasked with the investigation of the achieved in Indian shipbuilding. disturbances in Bombay, Delhi and Punjab in 1919 and their causes. • The committee also had Indian Representation.

10. Consider the following statements with respect to INS Kavaratti: 1. It is an anti-submarine warfare corvette built under Project 28. 2. The ship has up to 90% indigenous content. 3. It belongs to the same class as INS Kamorta, INS Kadmatt and INS Kiltan. Which of the given statement/s is/are Incorrect? (a) 1 only (b) 2 only (c) 2 and 3 only (d) None of the above Answer: d) Explanation: • INS Kavaratti is the last of the four indigenously built Anti-Submarine Warfare (ASW) stealth corvettes built under Project 28 (Kamorta class), by Garden Reach Shipbuilders & Engineers (GRSE), Kolkata. • The first three built under the Kamorta class – INS Kamorta, INS Kadmatt and INS Kiltan have been commissioned.

www.iasgatewayy.com  Daily Current Affairs | Monthly 044- Magazines | Online Tests 6 | P a g e

26265326/9884421666/9884472636 UPSC OCTOBER - 2020 DAILY CURRENT AFFAIRS CLASS TEST – 26 1. Consider the following, with regard to Snow • Gangotri National Park, Uttarakhand is the Leopard: third largest National Park in India. 1. It inhabits the higher Himalayan and trans- Protection: Himalayan landscape. • IUCN Red List- Vulnerable. 2. It is listed as endangered in IUCN Red List. • Convention on International Trade in Which of the statements given above is/are Endangered Species (CITES)- Appendix I. correct? • Convention on Migratory Species- (a) 1 only Appendix I (b) 2 only • Appendix I includes species threatened (c) Both 1 and 2 with extinction. (d) Neither 1 nor 2 Answer: a) 2. With reference to the Sir Syed Ahmad Khan, Explanation: consider the following statements: • The Snow Leopard (also known as Ghost 1. He established the Scientific Society in of the mountains) acts as an indicator of the 1864 to translate Western works into Indian health of the mountain ecosystem in which languages. they live, due to their position as the top 2. He was one of the founders of Darul Uloom predator in the food web. Deoband. • Habitation: The Snow Leopard lives at high 3. He founded the Muhammadan Anglo altitudes in the steep mountains of Central Oriental College on the pattern of Oxford and Southern Asia, and in an extremely Which of the statements given above is/are cold climate. correct? (a) 1 and 2 only • They inhabit the higher Himalayan and (b) 2 and 3 only trans-Himalayan landscape in the (c) 1 and 3 only states/union territories of Jammu and (d) All the above Kashmir, Himachal Pradesh, Uttarakhand, Answer: a) Sikkim, and Arunachal Pradesh. Explanation: • Snow Leopard capital of the world: Hemis, • Sir Syed Ahmad Khan established the Ladakh. Hemis National Park is the biggest Scientific Society in 1864, in Aligarh to national park of India and also has a good translate Western works into Indian presence of Snow Leopard. languages to prepare the Muslims to accept

Western Education and to inculcate www.iasgatewayy.com  Daily Current Affairs | Monthly 044- Magazines | Online Tests 1 | P a g e

26265326/9884421666/9884472636 UPSC OCTOBER - 2020 DAILY CURRENT AFFAIRS CLASS TEST – 26 Scientific Temperament among the Explanation: Muslims. Hence, statement 1 is correct. • The Financial Action Task Force (FATF) is • The Aligarh Institute Gazette, a magazine established to set standards and promote published by Sir Syed was an organ of the effective implementation of legal, Scientific Society. regulatory and operational measures for • In 1877, he founded the Muhammadan combating money laundering, terrorist Anglo Oriental College on the pattern of financing and other related threats to the Oxford and Cambridge universities. The integrity of the international financial college later grew into Aligarh Muslim system. University. • It is an inter-governmental body • Darul Uloom Deoband was established on established in 1989 during the G7 Summit 31 May 1866 by Muhammad Qasim in Paris. Hence, statement 1 is not correct. Nanotawi and other scholars in his circle. • Its Secretariat is located at the Organisation The founders include Sayyid Muhammad for Economic Cooperation and Abid, Mehtab Ali and Nehal Ahmad. Development (OECD) headquarters in Paris. Hence, statement 2 is correct. 3. In context to Financial Action Task Force • Member Countries: The FATF currently (FATF), consider the following statements: has 39 members including two regional 1. It is an inter-governmental body organisations — the European Commission established under the United Nations and Gulf Cooperation Council. India is a Organisation. member of the FATF. 2. Its Secretariat is located at the Organisation • Countries known as Non-Cooperative for Economic Cooperation and Countries or Territories (NCCTs) are put in Development (OECD) headquarters. the blacklist. These countries support terror 3. At present no country is under the FATF’s funding and money laundering activities. blacklist. The FATF revises the blacklist regularly, Which of the statements given above is/are not adding or deleting entries. correct? • As of this year (2020), there are only two (a) 1 and 2 only countries on the FATF's black list - North (b) 1 and 3 only Korea and Iran. (c) 2 and 3 only

(d) 1, 2 and 3

Answer: b)

www.iasgatewayy.com  Daily Current Affairs | Monthly 044- Magazines | Online Tests 2 | P a g e

26265326/9884421666/9884472636 UPSC OCTOBER - 2020 DAILY CURRENT AFFAIRS CLASS TEST – 26 4. Kalapani, Limpiyadhura and Lipulekh Explanation: sometimes seen in news are related to • Nepal shares it border with the states of (a) Disputed territories between India and Uttarakhand, Uttar Pradesh, Sikkim, Bihar Nepal and West Bengal. (b) More arsenic contamination in drinking water (c) Places which became plastic free completely (d) None of these

Answer: a)

Explanations: 6. Consider the following statements: • Nepal’s new official map is “artificial” and 1. Active pharmaceutical ingredient is a unacceptable says India after Nepal chemical compound that is the most unveiled a new political map that claimed important raw material to produce a Kalapani, Limpiyadhura and Lipulekh of finished medicine. Uttarakhand as part of its sovereign 2. About 40% of the paracetamol API territory. manufactured in the country is consumed in

5. Which of the following states share their the domestic market, while the rest is meant border with Nepal? for exports. 1. Uttarakhand 3. Since, being a leading supplier of high- 2. Uttar Pradesh quality medicines to several countries, 3. Sikkim Indian pharmaceutical industry is highly 4. Bihar self-sufficient in APIs. Select the correct answer using the code given Which of the statement(s) given above is/are below: Not correct? (a) 1 and 2 only (a) 1 only (b) 1, 2 and 3 only (b) 2 only (c) 2, 3 and 4 only (c) 3 only (d) All the above (d) None Answer: d) Answer: c)

www.iasgatewayy.com  Daily Current Affairs | Monthly 044- Magazines | Online Tests 3 | P a g e

26265326/9884421666/9884472636 UPSC OCTOBER - 2020 DAILY CURRENT AFFAIRS CLASS TEST – 26 Explanation: (b) 2 and 3 only • Active pharmaceutical ingredient is a (c) 1 and 3 only chemical compound that is the most (d) 1, 2 and 3 important raw material to produce a Answer: b) finished medicine. So, statement 1 is Explanation: correct. • Statutory Body: NHRC was established • About 40% of the paracetamol API on 12th October, 1993. The statute under manufactured in the country is consumed in which it is established is the Protection of the domestic market, while the rest is meant Human Rights Act (PHRA), 1993 as for exports. So, statement 2 is correct. amended by the Protection of Human • Despite, being a leading supplier of high- Rights (Amendment) Act, 2006.So, quality medicines to several countries, statement 1 is not correct. Indian pharmaceutical industry is highly ✓ The PHRA Act also provides for the dependent on China for APIs. So, creation of a State Human Rights statement 3 is not correct. Commission at the state level. • In Line with Paris Principles: Paris 7. Consider the following statement regarding the Principles were adopted for the promotion NHRC: and protection of human rights in October 1. NHRC is a statutory body established 1991, and were endorsed by the General under the National Human Rights Assembly of the United Nations in 1993. Commission Act, 2006. • Watchdog of Human Rights in the 2. The commission is a multi-member body country: The NHRC is an embodiment of consisting of a chairman and five members India’s concern for the promotion and with the chairperson should being a retired protection of human rights. chief justice of India or a judge of the ✓ Section 2(1)(d) of the PHRA defines Supreme Court Human Rights as the rights relating to 3. The chairman and members are appointed life, liberty, equality and dignity of the by the President on the recommendations individual guaranteed by the of a six-member committee consisting of Constitution or embodied in the the Prime Minister as its head. International Covenants and Which of the statements given above are enforceable by courts in India. correct?

(a) 1 and 2only www.iasgatewayy.com  Daily Current Affairs | Monthly 044- Magazines | Online Tests 4 | P a g e

26265326/9884421666/9884472636 UPSC OCTOBER - 2020 DAILY CURRENT AFFAIRS CLASS TEST – 26 • Composition: The commission is a multi- Vikramaditya was commissioned in 2013. member body consisting of a chairman and It was recently decommissioned. five members. The chairperson should be a • INS Vikramaditya is a modified Kiev-class retired chief justice of India or a judge of aircraft carrier and the flagship of the the Supreme Court. So, statement 2 is Indian Navy, which entered into service in correct. 2013. • Appointment: The chairman and members • INS Vishal, also known as Indigenous are appointed by the President on the Aircraft Carrier 2 (IAC-2), is a planned recommendations of a six-member aircraft carrier to be built by Cochin committee consisting of the Prime Minister Shipyard Limited for the Indian Navy. It is as its head, the Speaker of the Lok Sabha, intended to be the second aircraft carrier to the Deputy Chairman of the Rajya Sabha, be built in India after INS Vikrant (IAC-1). leaders of the Opposition in both the 9. Which of the following statement/s is/are Houses of Parliament and the Union Home correct with respect to OSIRIS-REx? Minister. So, statement 3 is correct. 1. It is a NASA asteroid study and sample- • Tenure: The chairman and members hold return mission targeted at near-earth office for a term of three years or until they asteroid, Bennu. attain the age of 70 years, whichever is 2. This would be the first-ever asteroid-study earlier. The President can remove the and sample-return mission. chairman or any member from the office Options: under some circumstances.

(a) 1 only 8. Which of the following would be the first (b) 2 only indigenously built Aircraft Carrier in India? (c) Both 1 and 2 (a) INS Vishal (d) Neither 1 nor 2 (b) INS Viraat Answer: a) (c) INS Vikrant Explanation: (d) INS Vikramaditya • OSIRIS-REx (Origins, Spectral Answer: c) Interpretation, Resource Identification, Explanation: Security, Regolith Explorer) is a NASA • INS Viraat was a Centaur-class aircraft asteroid-study and sample-return mission. carrier of the Indian Navy. INS Viraat was • The mission’s primary goal is to obtain a the flagship of the Indian Navy before INS sample from Bennu, a carbonaceous near- www.iasgatewayy.com  Daily Current Affairs | Monthly 044- Magazines | Online Tests 5 | P a g e

26265326/9884421666/9884472636 UPSC OCTOBER - 2020 DAILY CURRENT AFFAIRS CLASS TEST – 26 Earth asteroid, and return the sample to superconducting electromagnets used in Earth for a detailed analysis. maglev trains, magnetic resonance imaging • Other similar missions include the (MRI) and nuclear magnetic resonance Japanese probe Hayabusa which returned (NMR) machines, magnetic confinement samples from Itokawa in 2010, and fusion reactors (e.g. tokamaks), and the Hayabusa2 which will return samples from beam-steering and focusing magnets used Ryugu in December 2020. in particle accelerators

10. Which of the following can be potential areas of applications of high temperature superconductors? 1. low-loss power cables 2. Quantum computers 3. Maglev trains 4. Particle accelerators 5. Nuclear fusion reactors Options: (a) 1 and 3 only (b) 1,2 and 3 only (c) 3,4 and 5 only (d) 1,2,3,4 and 5 Answer: d) Explanation: • The observance of superconductivity at room temperature marks a major breakthrough given that traditionally superconductivity was observed at very low temperatures only. • Superconductivity at room temperature would lead to a large number of applications from power supplies (low-loss power cables) to quantum computers (fast digital circuits) to powerful www.iasgatewayy.com  Daily Current Affairs | Monthly 044- Magazines | Online Tests 6 | P a g e

26265326/9884421666/9884472636 UPSC OCTOBER - 2020 DAILY CURRENT AFFAIRS CLASS TEST – 27 1. With reference to ‘Kisan Suryodaya Yojana’, 2. In context to UDAN Scheme, consider the Consider the following: following statements: 1. It is aimed at providing day-time electricity 1. It envisages providing connectivity to un- to farmers. served and underserved airports of the 2. It is the centrally sponsored scheme. country. 3. It would complement irrigation projects of 2. It is under the aegis of the Ministry of Civil the State- Sujalam Sufalam Aviation. Which of the statements given above are 3. It is operational for a period of 10 years correct? Which of the statements given above are (a) 1 and 2 only correct? (b) 1 and 3 only (a) 1 and 3 only (c) 2 and 3 only (b) 2 and 3 only (d) All the above (c) 1 and 2 only Answer: d) (d) 1, 2 and 3 Explanation: Answer: d) • Recently, the Prime Minister has launched Explanation: the ‘Kisan Suryodaya Yojana’ aimed at • Ude Desh Ka Aam Naagrik (UDAN) was providing day-time electricity to farmers in launched as a regional connectivity scheme Gujarat (but not in Haryana and Punjab) for under the Ministry of Civil Aviation in irrigation and farming purposes. 2016. Hence, statement 2 is correct. • It would complement irrigation projects of • The objective of the scheme is to create the State- Sujalam Sufalam and SAUNI affordable yet economically viable and (Saurashtra-Narmada Avtaran Irrigation) profitable flights on regional routes so that yojana. flying becomes affordable to the common • It is a centrally sponsored scheme being man even in small towns. implemented to expand cultivated areas • The scheme envisages providing with assured irrigation, reduce wastage of connectivity to un-served and underserved water and improve water use efficiency and airports of the country through the revival ensure "Har Khet Ko Paani". of existing air-strips and airports. The scheme is operational for a period of 10 years.

www.iasgatewayy.com  Daily Current Affairs | Monthly 044- Magazines | Online Tests 1 | P a g e

26265326/9884421666/9884472636 UPSC OCTOBER - 2020 DAILY CURRENT AFFAIRS CLASS TEST – 27 3. Consider the following statements, with regard • It was set up by the Government in to Central Vigilance Commission: February, 1964 on the recommendations of 1. It advises various authorities in Central the Committee on Prevention of Government organizations in planning, Corruption, headed by K. Santhanam. executing, reviewing and reforming their vigilance work. 4. Consider the following statements regarding 2. It is only responsible to the Parliament. the reservation in India: 3. It was set up on the recommendations of the 1. Various provisions in Article 15 and 16 of K. Santhanam Committee. the Constitution empower the State to make Which of the statements given above are special provisions in education and jobs correct? respectively for the socially and (a) 1 and 2 only educationally backward section of the (b) 2 and 3 only society and for the economically weaker (c) 1 and 3 only section. (d) 1, 2 and 3 2. Inspite of the constitutional provisions in Answer: d) the Part III of the constitution, the Right to Explanation: reservation is not fundamental right and is only a Constitutional Right. • Central Vigilance Commission (CVC) is 3. The Supreme Court has recently ruled that the apex vigilance institution, free of reservations in promotion can be control from any executive authority, considered as the fundamental right. monitoring all vigilance activity under the Which of the statement(s) given above is/are Central Government and advising various correct? authorities in Central Government (a) 1 only organizations in planning, executing, (b) 1 and 2 only reviewing and reforming their vigilance (c) 3 only work. (d) 1 and 3 only • It is an independent body and is only Answer: b) responsible to the Parliament. Explanation: • The Parliament enacted Central Vigilance • Various provisions in Article 15 and 16 of Commission Act, 2003 (CVC Act) the Constitution empower the State to make conferring statutory status on the CVC. special provisions in education and jobs

respectively for the socially and www.iasgatewayy.com  Daily Current Affairs | Monthly 044- Magazines | Online Tests 2 | P a g e

26265326/9884421666/9884472636 UPSC OCTOBER - 2020 DAILY CURRENT AFFAIRS CLASS TEST – 27 educationally backward section of the recommend steps to be taken for the society and for the economically weaker advancement of those classes. section. So, statement 1 is correct. • The Mandal Commission concluded that • Inspite of the constitutional provisions in India’s population consisted of the Part III of the constitution, the Right to approximately 52 percent OBCs, therefore reservation is not fundamental right and is 27% government jobs should be reserved only a Constitutional Right. So, statement for them. 2 is correct. • The commission has developed eleven • The Supreme Court has recently ruled that indicators of social, educational, and reservations in promotion also cannot be economic backwardness. considered as the fundamental right. So, • Apart from identifying backward classes statement 3 is not correct. among Hindus, the Commission has also identified backward classes among non- 5. ‘Mandal Commission’ was sometimes seen in Hindus (e.g., Muslims, Sikhs, Christians, news is constituted for which of the following and Buddhists. purposes? • It has generated an all-India other (a) To revamp the sick PSU’s backward classes (OBC) list of 3,743 (b) Report on the working of capital market castes and a more underprivileged infrastructure institutions “depressed backward classes” list of 2,108 (c) Reservations for OBC’s castes.

(d) To review the Methodology for Measurement of Poverty 6. With reference to the powers of the Lieutenant Answer: c) Governor of the Union Territory, consider the Explanation: following statements: • In exercise of the powers conferred by 1. The Council of Ministers headed by a Chief Article 340 of the Constitution, the Minister will aid and advise the President appointed a backward class Administrator in the exercise of his commission in December 1978 under the functions in relation to matters which the chairmanship of B. P. Mandal. Union Territory has power to make laws. • The commission was formed to determine 2. It also allows the LG to act in his discretion the criteria for defining India’s “socially in the matter of lawmaking, even though and educationally backward classes” and to the Council of Ministers has the task of aiding and advising him. www.iasgatewayy.com  Daily Current Affairs | Monthly 044- Magazines | Online Tests 3 | P a g e

26265326/9884421666/9884472636 UPSC OCTOBER - 2020 DAILY CURRENT AFFAIRS CLASS TEST – 27 3. In case of a difference of opinion between advising him. So, both statement 1 and 2 the LG and his Ministers on any matter, the are correct. Administrator is bound to refer it to the • In case of a difference of opinion between President for a decision. the LG and his Ministers on any matter, the Which of the statements given above are Administrator is bound to refer it to the correct? President for a decision and act according (a) 1 and 2 only to the decision given by the President. (b) 2 and 3 only • However, the Administrator can also claim (c) 1 and 3 only that the matter is urgent, and take (d) 1, 2 and 3 immediate action as he deems necessary. Answer: d) So, statement 3 is also correct. Explanation: • The Government of Union Territories Act, 7. Consider the following statements: 1963 provides for a Legislative Assembly 1. Under the absolute liability principle, a of Pondicherry (as Puducherry was then party is not liable and need not pay called), with a Council of Ministers to compensation if a hazardous substance govern the “Union Territory of escapes his premises by accident or by an Pondicherry”. “act of God’” among other circumstances. • The same Act says that the UT will be 2. Under the strict liability principle, the apex administered by the President of India court held that a company in a hazardous through an Administrator (LG). industry cannot claim any exemption. ✓ Section 44 of the Act, says the Council of Which of the statement(s) given above is/are Ministers headed by a Chief Minister will not correct? “aid and advise the Administrator in the (a) 1 only exercise of his functions in relation to (b) 2 only matters with respect to which the (c) Both 1 and 2 Legislative Assembly of the Union (d) Neither 1 nor 2 Territory has power to make laws”. Answer: c) ✓ The same clause also allows the LG to “act Explanation: in his discretion” in the matter of • Under the “strict liability principle”, a party ✓ lawmaking, even though the Council of is not liable and need not pay compensation Ministers has the task of aiding and if a hazardous substance escapes his

www.iasgatewayy.com  Daily Current Affairs | Monthly 044- Magazines | Online Tests 4 | P a g e

26265326/9884421666/9884472636 UPSC OCTOBER - 2020 DAILY CURRENT AFFAIRS CLASS TEST – 27 premises by accident or by an “act of God’” 3. Pakistan among other circumstances. 4. India • Under the absolute liability principle, the 5. Australia apex court held that a company in a Choose the correct option: hazardous industry cannot claim any (a) 1, 2 and 4 only exemption. (b) 1, 2 and 5 only • Since both terms are interchanged both (c) 2, 3, 4 and 5 the statements are not correct. (d) 1, 2, 3, 4 and 5 Answer: a) 8. The Chatar festival is associated with which of Explanation: the following states? • The Governing Body is the executive body (a) Chhattisgarh of the International Labour Office (the (b) Odisha Office is the secretariat of the (c) Jharkhand Organization). (d) West Bengal • In the International Labour Organization’s Answer: b) (ILO) governing body, ten of the titular Explanation: government seats are permanently held by • Chatar festival is celebrated in Odisha’s States of chief industrial importance. Kalahandi district. Lakhs of devotees from • These are: Brazil, China, France, Germany, Odisha, Chhattisgarh, Jharkhand and West India, Italy, Japan, the Russian Federation, Bengal participate in the festival. the United Kingdom and the United States. • The festival is usually marked by the sacrifice of thousands of animals and birds. 10. Consider the following statements about the • This year, the Orissa High Court had National Institute of Communicable Diseases ordered ban on animal sacrifice during (NICD): ‘Chatar Yatra’ festival. 1. It has its origin as the Central Malaria Bureau, established in 1909. 9. Which of these countries hold a permanent seat 2. The Institute has its headquarters in Patna, in the governing body of the International Bihar. Labour Organisation as the state of chief 3. It functions as the nodal agency in the industrial importance? country for disease surveillance facilitating 1. Germany prevention and control of communicable 2. China diseases. www.iasgatewayy.com  Daily Current Affairs | Monthly 044- Magazines | Online Tests 5 | P a g e

26265326/9884421666/9884472636 UPSC OCTOBER - 2020 DAILY CURRENT AFFAIRS CLASS TEST – 27 Which of the given statement/s is/are correct? (a) 1 and 3 only (b) 1 and 2 only (c) 3 only (d) 2 and 3 only Answer: a) Explanation: • The National Centre for Disease Control (NCDC), formerly National Institute of Communicable Diseases (NICD), had its origin as the Central Malaria Bureau, established at Kasauli (Himachal Pradesh) in 1909. • The Institute has its headquarters in Delhi. • It functions as the nodal agency in the country for disease surveillance facilitating prevention and control of Communicable Diseases.

www.iasgatewayy.com  Daily Current Affairs | Monthly 044- Magazines | Online Tests 6 | P a g e

26265326/9884421666/9884472636 UPSC OCTOBER - 2020 DAILY CURRENT AFFAIRS CLASS TEST – 28 1. In context to Polio, consider the following • The last case due to wild poliovirus in the statements: country was detected on 13th January 2011. 1. It is a crippling and potentially deadly viral • Every year, 24th October is observed as infectious disease that affects the skeletal World Polio Day in order to call on system. countries to stay vigilant in their fight 2. It is transmitted by person-to-person against the disease. mainly through the faecal-oral route. • It was established to commemorate the 3. India received polio-free certification by birth of , who developed an the WHO in 2020. Inactivated (killed) (IPV). Which of the statements given above is/are not correct? 2. With reference to the Circular Economy, (a) 1 and 2 only consider the following statements: (b) 2 and 3 only 1. It is an economy where almost everything (c) 1 and 3 only of a product gets reused, remanufactured, (d) 1, 2 and 3 and recycled. Answer: c) 2. It is an initiatives in India are undertaken Explanation: by the National Productivity Council • Polio is a crippling and potentially deadly (NPC). viral infectious disease that affects the Which of the statements given above is/are nervous system. Hence, statement 1 is not correct? correct. (a) 1 only • The virus is transmitted by person-to- (b) 2 only person mainly through the faecal-oral route (c) Both 1 and 2 or, less frequently, by a common vehicle (d) Neither 1 nor 2 (for example, through contaminated water Answer: c) or food). Hence, statement 2 is correct. Explanation: • It largely affects children under 5 years of • Circular Economy is an economy where age. products are designed for durability, reuse and recyclability and thus almost • India received polio-free certification by everything gets reused, remanufactured, the WHO in 2014, after three years of zero and recycled into a raw material or used as cases. Hence, statement 3 is not correct. a source of energy. Hence, statement 1 is

correct. www.iasgatewayy.com  Daily Current Affairs | Monthly 044- Magazines | Online Tests 1 | P a g e

26265326/9884421666/9884472636 UPSC OCTOBER - 2020 DAILY CURRENT AFFAIRS CLASS TEST – 28 • India is already on its path to the circular Answer: c) economy. Initiatives of the National Explanation: Productivity Council (NPC) and • The Indian Tsunami Early Warning System government show that. Hence, statement 2 (ITEWS) was established in 2007 and is is correct. based at & operated by INCOIS, • NPC is an autonomous organisation under Hyderabad. Hence, statement 1 is not the Ministry of Commerce & Industry. correct. Productivity Week 2019 Theme was • It is an integrated effort of different ‘Circular Economy for Productivity and organizations including the Department of Sustainability’. Space (DOS), Department of Science and • ‘Digital India’ Program contains a Technology (DST), the Council of significant component of the recycling of Scientific and Industrial Research (CSIR), electronic wastes. Survey of India (SOI) and National • Swachh Bharat Mission is also about Institute of Ocean Technology (NIOT). making wealth out of wastes. Hence, statement 2 is not correct. • India has a huge potential for reuse and • Recently, the Intergovernmental recycling as only around 20% of the total Oceanographic Commission (IOC) of waste generated goes into the Recycling UNESCO (also known as UNESCO-IOC) Process. accredited Indian Tsunami Early Warning Centre (ITEWC) as Tsunami Service 3. Consider the following, With reference to Provider (TSP) for 28 Indian Ocean Rim Indian Tsunami Early Warning System: (IOR) countries, along with Indonesia and 1. It is based in Chennai. Australia in 2011, for issuing regional 2. It was established by the Ministry of Earth warnings.

Sciences. 3. It also provides the services for Indian 4. Arrange the following Elephant Reserves from Ocean Rim countries. North to South: Which of the statements given above is/are 1. Dihing-Patkai Elephant Reserve Incorrect? 2. Dandeli Elephant Reserve (a) 1 and 3 only 3. Anamalai Elephant Reserve (b) 2 and 3 only 4. Mayurbhanj Elephant Reserve (c) 1 and 2 only Choose the correct option: (d) 1, 2 and 3 (a) 1, 2, 4, 3 www.iasgatewayy.com  Daily Current Affairs | Monthly 044- Magazines | Online Tests 2 | P a g e

26265326/9884421666/9884472636 UPSC OCTOBER - 2020 DAILY CURRENT AFFAIRS CLASS TEST – 28 (b) 4, 1, 2, 3 6. India holds ministerial level ‘2+2’ Dialogue (c) 1, 4, 2, 3 with which of the following countries? (d) 4, 2, 3, 1 1. The U.S.A. Answer: c) 2. Japan Explanation: 3. China • Dihing-Patkai Elephant Reserve – Assam 4. Australia • Mayurbhanj Elephant Reserve – Odisha Options: • Dandeli Elephant Reserve – Karnataka (a) 1, 2 and 4 only • Anamalai Elephant Reserve – Tamil Nadu (b) 1 and 2 only (c) 1, 2 and 3 only 5. Consider the following statements with respect (d) 1 and 4 only to Assumption Island: Answer: b) 1. It is one of the islands constituting the Explanation: Seychelles archipelago. • India holds ministerial-level ‘2+2’ 2. It is located to the north of Madagascar. Dialogue with the U.S.A and Japan. 3. India has signed an agreement to develop • It holds such talks with Australia, at the transport infrastructure facilities on the foreign secretary and defence secretary Island. level. Which of the given statement/s is/are Incorrect? 7. India is not a signatory to which of the (a) 1 only following agreements? (b) 2 and 3 only 1. COMCASA (c) 1, 2 and 3 2. GSOMIA (d) None of the above 3. BECA Answer: d) 4. LEMOA Explanation: Select the correct answer using the code given • Assumption Island is one of the islands below: constituting the Seychelles archipelago. (a) 1 and 2 only • It is located to the north of Madagascar. (b) 2 and 3 only • India has signed an agreement with (c) 1, 2 and 4 only Seychelles to jointly develop transport (d) None of the above infrastructure facilities on the Island. Answer: d)

www.iasgatewayy.com  Daily Current Affairs | Monthly 044- Magazines | Online Tests 3 | P a g e

26265326/9884421666/9884472636 UPSC OCTOBER - 2020 DAILY CURRENT AFFAIRS CLASS TEST – 28 Explanation: the government may now allow transfer of • India recently signed BECA too, while land “in favour of a person or an institution India is a signatory to all other agreements. for the purpose of promotion of healthcare or senior secondary or higher or specialized 8. In the Context to the J&K Development Act of education in J&K”. So, statement 2 is the J&K, what are the Newly Introduced correct. Laws? • According to amendments made to “The 1. Under a new provision, an Army officer not Jammu & Kashmir Land Revenue Act, below the rank of Corps Commander can Samvat, 1996”, only agriculturists of J&K declare an area as “Strategic Area” within can purchase agricultural land. “No sale, a local area, only for direct operational and gift, exchange, or mortgage of the land training requirements of the armed forces. shall be valid in favour of a person who is 2. Under the ‘transfer of land for the purpose not an agriculturist”. of promotion of healthcare or education’, • The Restriction on Conversion of the government may now allow transfer of Agricultural Land and Process for land in favour of a person or an institution Permission of Non-Agriculture clause, for the purpose of promotion of healthcare however, puts conditions on the use of or senior secondary or higher or specialized agricultural land. “No land used for education in J&K. agriculture purposes shall be used for any 3. No land used for agriculture purposes shall non-agricultural purposes except with the be used for any non-agricultural purposes permission of the district collector”. So, except with the permission of the district statement 3 is correct. collector. • Under a new provision, an Army officer not Select the correct answer using the code given below the rank of Corps Commander can below: declare an area as “Strategic Area” within (a) 1 and 2 only a local area, only for direct operational and (b) 2 and 3 only training requirements of the armed forces. (c) 1 and 3 only So, statement 1 is correct. (d) 1, 2 and 3 • The introduction of the UT of J&K Answer: d) Reorganisation (Adaptation of Central Explanation: Laws) Third Order, 2020 by the Ministry of • Under the ‘transfer of land for the purpose Home Affairs (MHA) has resulted in the of promotion of healthcare or education’, www.iasgatewayy.com  Daily Current Affairs | Monthly 044- Magazines | Online Tests 4 | P a g e

26265326/9884421666/9884472636 UPSC OCTOBER - 2020 DAILY CURRENT AFFAIRS CLASS TEST – 28 repeal of at least 11 land laws in vogue in they “cooperate” in the larger public J&K earlier, including the J&K Big Landed interest. Estates Abolition Act that had resulted in • So all the five except an industrial summit famous ‘Land to tiller’ rights. organized by a state comes under Cooperative Federalism. 9. Which of the following comes under the • An industrial summit organized by a state concept of Cooperative Federalism? comes under the concept of Competitive 1. GST Council Federalism. In Competitive federalism 2. NITI Aayog the relationship between the Central and 3. Interstate Council state governments is vertical and between 4. An industrial summit organized by a state state Governments is Horizontal.

5. Panchayati Raj Select the correct answer using the code given 10. The term Bio-terrorism recently in news refers below: to (a) 1 and 2 only (a) Terrorism using naturally occurring Virus (b) 1, 2 and 3 only and Bacteria (c) 1, 2, 3 and 5 only (b) Terrorism using modified forms of Virus (d) All the above and Bacteria Answer: c) (c) Both a and b Explanation: (d) None of the above • Cooperative federalism, also known as Answer: c) marble-cake federalism, is a concept of Explanation: federalism in which federal, state, and local • Bioterrorism is terrorism involving the governments interact cooperatively and intentional release or dissemination of collectively to solve common problems, Biological Agents. rather than making policies separately but • These agents are bacteria, viruses, fungi, or more or less equally (such as the dual toxins, and may be in a naturally occurring federalism of the 19th-century United or a human-modified form, in much the States) or clashing over a policy in a system same way in Biological Warfare. dominated by the national government. • Recently Union defence minister described • In Cooperative federalism the Centre and bio-terrorism as a “contagious plague” and states share a horizontal relationship, where asked the armed forces to be at the forefront of Combating the Menace. www.iasgatewayy.com  Daily Current Affairs | Monthly 044- Magazines | Online Tests 5 | P a g e

26265326/9884421666/9884472636 UPSC OCTOBER - 2020 DAILY CURRENT AFFAIRS CLASS TEST – 29 1. Consider the following statements, with regard period of two years (i.e. 730 days) during to ‘Child Care Leave’: their entire service for taking care of upto 1. It is an earned leave granted to the married Two Minor Children. women employees only. • The child care leave is granted either for 2. Women employees having minor children rearing children or to look after any of the may be granted child care leave for a needs of children like examination, maximum period of two years during their sickness etc. entire service. • Male Government employees who are 3. Single male parents working in private single parents will now be eligible for child sectors can also avail the benefits of child care leave. care leave. • Single male parents, includes unmarried Which of the statements given above is/are employees, widowers and divorcees, who Incorrect? may be expected to take up the (a) 1 and 2 only responsibility of caring for a child single- (b) 1 and 3 only handedly. (c) 2 and 3 only (d) 1, 2 and 3 2. Consider the following statements, with Answer: a) reference to Superconductor: Explanation: 1. A superconductor is a substance that • Child Care Leave: It is treated as earned conducts electricity without resistance. leave and is generally granted to women 2. A superconducting material does not employees. Hence, statement 1 is not Exhibit Perfect Diamagnetism. correct. Which of the statements given above is/are • Child care leave is granted at 100% of leave correct? salary for the first 365 days and 80% of (a) 1 only leave salary for the next 365 days. (b) 2 only (c) Both 1 and 2 • Male Government employees who are (d) Neither 1 nor 2 single parents will now be eligible for child Answer: a) care leave. Explanation: • Women employees having minor children • A superconductor is a substance that (children upto 18 years of age) may be conducts electricity without resistance granted child care leave for a maximum when it becomes colder than a "critical www.iasgatewayy.com  Daily Current Affairs | Monthly 044- Magazines | Online Tests 1 | P a g e

26265326/9884421666/9884472636 UPSC OCTOBER - 2020 DAILY CURRENT AFFAIRS CLASS TEST – 29 temperature." At this temperature, (a) 1 only electrons can move freely through the (b) 2 only material. Hence, statement 1 is correct. (c) Both 1 and 2 • No heat, sound or any other form of energy (d) Neither 1 nor 2 would be released from the material when Answer: c) it has reached critical temperature (Tc), or Explanation: the temperature at which the material • Basic Exchange and Cooperation becomes superconductive. Agreement (BECA) is one of the four • The critical temperature for foundational military communication superconductors is the temperature at agreements between India and the USA. which the electrical resistivity of metal The other three are: General Security Of drops to zero. Military Information Agreement • Prominent examples include aluminium, (GSOMIA), Logistics Exchange niobium, magnesium diboride, etc. Memorandum of Agreement (LEMOA), • A superconducting material kept in a and Communications and Information magnetic field expels the magnetic flux out Security Memorandum of Agreement its body when cooled below the critical (CISMOA). temperature and exhibits perfect • It will enable the exchange of geospatial diamagnetism. data and information between the two • It is also called the Meissner effect which countries and will improve the accuracy of simply means that magnetic lines do not India’s missiles in precision strikes. Hence, pass through superconductors in a statement 1 is correct. The Geospatial Maps magnetic field. and charts to be shared, will be acquired from multiple sources like satellites, 3. In context to Basic Exchange and Cooperation unmanned aerial vehicles (UAVs), Agreement (BECA), consider the following reconnaissance aircraft, aerostats among statements: others. 1. It will help improve the accuracy of India’s • BECA is an important precursor to India missiles in precision strikes. acquiring armed unmanned aerial vehicles 2. It is an important precursor to India such as the Predator-B from the USA. acquiring armed unmanned aerial vehicles Predator-B uses spatial data for accurate Which of the statements given above is/are strikes on Enemy Targets. correct? www.iasgatewayy.com  Daily Current Affairs | Monthly 044- Magazines | Online Tests 2 | P a g e

26265326/9884421666/9884472636 UPSC OCTOBER - 2020 DAILY CURRENT AFFAIRS CLASS TEST – 29 4. In the Context to the El Nino and La Nina, monsoon is known to get depressed. So, which of the statement(s) given below is/are Statement 2 is correct. correct? • This year the El Niño Southern Oscillation, 1. The northeast monsoon is known to receive or ENSO, is in neutral state and is likely to a boost from El Niño, when the sea surface remain like that for the rest of the year. temperatures in the equatorial Pacific 5. With reference to the recent developments in Ocean, off the western coast of South science which one of the following statements America, are warmer than usual. is not correct? 2. And, when the opposite phenomena La (a) Functional chromosomes can be created by Niña happens, rainfall during the northeast joining segments of DNA taken from cells monsoon is known to get depressed. of different species. Select the correct answer using the code given (b) Pieces of artificial functional DNA can be below: created in laboratories. (a) 1 only (c) A piece of DNA taken out from an animal (b) 2 only cell can be made to replicate outside a (c) Both 1 and 2 living cell in a laboratory. (d) Neither 1 nor 2 (d) Cells taken out from plants and animals can Answer: c) be made to undergo cell division in Explanation: laboratory petri dishes. • Like the southwest monsoon, the northeast Answer: a) monsoon is also impacted by the warming Explanation: and cooling of sea surface waters in the • Scientists at University of Texas created central Pacific Ocean. But the impact is the First Living Organism with ‘Artificial’ opposite. DNA. So, B is eliminated. (Because we are • The northeast monsoon is known to receive required to find wrong option, whereas B is a boost from El Niño, when the sea surface right.) temperatures in the equatorial Pacific • DNA replication can also be performed in Ocean, off the western coast of South vitro (outside a cell). So, C is eliminated. America, are warmer than usual. So, • Cell division of Chlamydomonas is usually Statement 1 is correct. achieved in the laboratory by alternation of • And, when the opposite phenomena La light and dark periods. So, D is eliminated. Niña happens, rainfall during the northeast • Thus we are left with answer A. www.iasgatewayy.com  Daily Current Affairs | Monthly 044- Magazines | Online Tests 3 | P a g e

26265326/9884421666/9884472636 UPSC OCTOBER - 2020 DAILY CURRENT AFFAIRS CLASS TEST – 29 6. With reference to the Van Dhan Yojana, which • At State level, the State Nodal Agency for of the statement(s) given below is/are correct? MFPs and the District collectors are 1. Van Dhan Yojana was launched as a envisaged to play a pivot role in scheme government pension scheme for senior implementation at grassroots level. citizens which is managed and operated by Life Insurance Corporation (LIC). 7. Global Gender Gap Report 2020 was released 2. The scheme provides an assured return of recently by which of the following? 8% p.a. payable monthly and is exempted (a) UNESCO from service tax/GST. (b) World Economic Forum Select the correct answer using the code given (c) UN Development Programme below: (d) Organization for economic cooperation (a) 1 only and development (b) 2 only Answer: b) (c) Both 1 and 2 Explanation: (d) Neither 1 nor 2 • World Economic Forum's Global Gender Answer: d) Gap Report, 2020 was released recently. Explanation: • According to the report, India moved • The above-mentioned statements are down the ladder from its 108th position related to the Pradhan Mantri Vaya last year to rank below countries like Vandana Yojana. So, both statements are China (106th), Sri Lanka (102nd), Nepal not correct. (101st), Brazil (92nd), Indonesia (85th) • The Van Dhan Scheme is an initiative of and Bangladesh (50th). the Ministry of Tribal Affairs and Tribal • India is 112th among nations ranked in Cooperative Marketing Development terms of the gap between genders, and it Federation of India Limited (TRIFED) was among the bottom five for women's aimed to improve tribal incomes through health and survival and economic value addition of tribal products. participation. • The scheme will be implemented through • Iceland remained the world's most gender- Ministry of Tribal Affairs as Nodal neutral country. Department at the Central Level and • Yemen is ranked the worst (153rd), while TRIFED as Nodal Agency at the National Iraq is 152nd and Pakistan 151st. Level. • Gender Inequality Index – United Nations Development Programme (UNDP) www.iasgatewayy.com  Daily Current Affairs | Monthly 044- Magazines | Online Tests 4 | P a g e

26265326/9884421666/9884472636 UPSC OCTOBER - 2020 DAILY CURRENT AFFAIRS CLASS TEST – 29 • Global Gender Gap Index – World • Child care leave is granted at 100% of leave Economic Forum salary for the first 365 days and 80% of • Gender Parity Index – UNESCO leave salary for the next 365 days. • Social Institutions and Gender Index • Male Government employees who are (SIGI) Report – Organization for single parents will now be eligible for child economic cooperation and development care leave. (OECD) • Women employees having minor children (children upto 18 years of age) may be 8. Consider the following statements, with regard granted child care leave for a maximum to ‘Child Care Leave’: period of two years (i.e. 730 days) during 1. It is an earned leave granted to the married their entire service for taking care of upto women employees only. two minor children. 2. Women employees having minor children • The child care leave is granted either for may be granted child care leave for a rearing children or to look after any of the maximum period of two years during their needs of children like examination, entire service. sickness etc. 3. Single male parents working in private • Male Government employees who are sectors can also avail the benefits of child single parents will now be eligible for child care leave. care leave. Which of the statements given above is/are • Single male parents, includes unmarried Incorrect? employees, widowers and divorcees, who (a) 1 and 2 only may be expected to take up the (b) 1 and 3 only responsibility of caring for a child single- (c) 2 and 3 only handedly. (d) 1, 2 and 3

Answer: a) 9. Consider the following statements, with Explanation: reference to Superconductor: • Child Care Leave: It is treated as earned 1. A superconductor is a substance that leave and is generally granted to women conducts electricity without resistance. employees. Hence, statement 1 is not 2. A superconducting material does not correct. Exhibit Perfect Diamagnetism. Which of the statements given above is/are correct? www.iasgatewayy.com  Daily Current Affairs | Monthly 044- Magazines | Online Tests 5 | P a g e

26265326/9884421666/9884472636 UPSC OCTOBER - 2020 DAILY CURRENT AFFAIRS CLASS TEST – 29 (a) 1 only 10. In context to Basic Exchange and Cooperation (b) 2 only Agreement (BECA), consider the following (c) Both 1 and 2 statements: (d) Neither 1 nor 2 1. It will help improve the accuracy of India’s Answer: a) missiles in precision strikes. Explanation: 2. It is an important precursor to India • A superconductor is a substance that acquiring armed unmanned aerial vehicles conducts electricity without resistance Which of the statements given above is/are when it becomes colder than a "critical correct? temperature." At this temperature, (a) 1 only electrons can move freely through the (b) 2 only material. Hence, statement 1 is correct. (c) Both 1 and 2 • No heat, sound or any other form of energy (d) Neither 1 nor 2 would be released from the material when Answer: c) it has reached critical temperature (Tc), or Explanation: the temperature at which the material • Basic Exchange and Cooperation becomes superconductive. Agreement (BECA) is one of the four • The critical temperature for foundational military communication superconductors is the temperature at agreements between India and the USA. which the electrical resistivity of metal The other three are: General Security Of drops to zero. Military Information Agreement • Prominent examples include aluminium, (GSOMIA), Logistics Exchange niobium, magnesium diboride, etc. Memorandum of Agreement (LEMOA), • A superconducting material kept in a and Communications and Information magnetic field expels the magnetic flux out Security Memorandum of Agreement its body when cooled below the critical (CISMOA). temperature and exhibits perfect • It will enable the exchange of geospatial diamagnetism. data and information between the two • It is also called the Meissner effect which countries and will improve the accuracy of simply means that magnetic lines do not India’s missiles in precision strikes. Hence, pass through superconductors in a statement 1 is correct. magnetic field.

www.iasgatewayy.com  Daily Current Affairs | Monthly 044- Magazines | Online Tests 6 | P a g e

26265326/9884421666/9884472636 UPSC OCTOBER - 2020 DAILY CURRENT AFFAIRS CLASS TEST – 29 • The geospatial maps and charts to be shared, will be acquired from multiple sources like satellites, unmanned aerial vehicles (UAVs), reconnaissance aircraft, aerostats among others. • BECA is an important precursor to India acquiring armed unmanned aerial vehicles such as the Predator-B from the USA. Predator-B uses spatial data for accurate strikes on enemy targets.

www.iasgatewayy.com  Daily Current Affairs | Monthly 044- Magazines | Online Tests 7 | P a g e

26265326/9884421666/9884472636 UPSC OCTOBER - 2020 DAILY CURRENT AFFAIRS CLASS TEST – 30 1. Consider the following: • (ISRO) include RESOURCESAT- 2, 2A, 1. RESOURCESAT- 2 -1, 2, 2A, 2B, RISAT-1 and 2, 2. CARTOSAT-1 OCEANSAT-2, Megha-Tropiques, 3. Megha-Tropiques SARAL and SCATSAT-1, INSAT-3DR, 4. SARAL 3D, etc. Which of the above earth observation satellite 2. In context to National Air Quality Monitoring have been launched by the Indian Space Program based Dashboard, consider the Research Organisation (ISRO)? following: (a) 1, 2 and 4 only 1. It is built on the data of CPCB's National (b) 1, 2 and 3 only Ambient Air Quality Monitoring (c) 2, 3 and 4 only (NAAQM) Network. (d) 1, 2, 3 and 4 2. Respirable suspended particulate matter Answer: d) (RSPM) are not recorded under the Explanation: National Air Quality Monitoring • India would launch its latest earth Programme (NAMP). observation satellite EOS-01 and nine Which of the statements given above is/ are international customer satellites from correct? Satish Dhawan Space Centre in Andhra (a) 1 only Pradesh’s Sriharikota. (b) 2 only • EOS-01 is an earth observation satellite and (c) Both 1 and 2 is intended for applications in agriculture, (d) Neither 1 nor 2 forestry and disaster management support. Answer: a) • Earth observation satellites are the Explanation: satellites equipped with remote sensing • The National Air Quality Monitoring technology. Earth observation is the Programme (NAMP) based dashboard is gathering of information about Earth's built on data from the Central Pollution physical, chemical and biological systems. Control Board’s National Ambient Air • Many Earth observation satellites have Quality Monitoring (NAAQM) Network been employed on sun-synchronous orbit. which was started in 1984-85 and covers • Other Earth observation satellites launched 344 cities/towns in 29 states and 6 UTs. by Indian Space Research Organisation • Under the NAMP, each station records

about 104 observations a year covering www.iasgatewayy.com  Daily Current Affairs | Monthly 044- Magazines | Online Tests 1 | P a g e

26265326/9884421666/9884472636 UPSC OCTOBER - 2020 DAILY CURRENT AFFAIRS CLASS TEST – 30 four pollutants– Sulphur dioxide (SO2), 4. Which of the following defence exercises are nitrogen dioxide (NO2), suspended held between India and France? particulate matter (SPM), and respirable 1. Exercise Shakti suspended particulate matter (RSPM). 2. Exercise Varuna 3. Exercise Indra 3. Consider the following, with reference to 4. Exercise Garuda India-Central Asia Dialogue: Select the correct answer using the codes given 1. It is a ministerial-level dialogue between below: India and all the Central Asian countries. (a) 1 and 2 only 2. All the countries participating in the (b) 1, 2 and 3 only dialogue are also members of the Shanghai (c) 2, 3 and 4 only Cooperation Organisation (SCO). (d) 1, 2 and 4 only Which of the statements given above is/ are Answer: d) correct? Explanation: (a) 1 only • Exercise SHAKTI is a biennial exercise (b) 2 only which is held between armies of India and (c) Both 1 and 2 France. It is conducted alternately in both (d) Neither 1 nor 2 countries. Answer: a) • Varuna naval exercise is an integral part of Explanation: France–India strategic relationship in the • India-Central Asia Dialogue is a 21st century and consists of naval ministerial-level dialogue between India cooperation drills. and all the Central Asian countries namely • Ex Garuda is a bilateral air training exercise Kazakhstan, Kyrgyzstan, Tajikistan, between India and France. Turkmenistan and Uzbekistan. • Exercise INDRA is a joint, tri services • All five nations became independent states exercise between India and Russia. The after the collapse of the USSR in 1991, INDRA series of exercise began in 2003. It post-Cold war. was conducted as a single service exercise • All the countries participating in the alternately between the two countries. dialogue, except for Turkmenistan, are also However, the first joint Tri Services members of the Shanghai Cooperation Exercise was conducted in 2017. Organisation.

www.iasgatewayy.com  Daily Current Affairs | Monthly 044- Magazines | Online Tests 2 | P a g e

26265326/9884421666/9884472636 UPSC OCTOBER - 2020 DAILY CURRENT AFFAIRS CLASS TEST – 30 5. In context to Indo-Tibetan Border Police, Ladakh to Jachep La in Arunachal Pradesh consider the following statements: covering 3488 km of Indo-China Border. 1. It is a Central Armed Police Forces under • The Force is also deployed for Anti Naxal the Ministry of Defence. operations and other internal security 2. It was raised during the India-Pakistan War duties. in 1971. 6. Which of the following is not an implication of 3. It also performs other Internal Security A.V. Dicey’s Rule of Law? Duties. 1. Absence of arbitrary power Which of the statements given above are not 2. Equality before the law correct? 3. The primacy of the rights of individual (a) 1 and 3 only Select the correct answer using the code given (b) 2 and 3 only below: (c) 1 and 2 only (a) 1 only (d) 1, 2 and 3 (b) 2 only Answer: c) (c) 3 only Explanation: (d) None • Indo-Tibetan Border Police Force (ITBPF) Answer: d) is a Central Armed Police Force Explanation: functioning under the Ministry of Home • A V Dicey in his book The Law of the Affairs, Government of India. Constitution (1885) has given the • Other Central Armed Police Forces are: following three implications of the doctrine Assam Rifles (AR), Border Security Force of rule of law. (BSF), Central Industrial Security Force ✓ Absence of arbitrary power, that is, no (CISF), Central Reserve Police Force man is punished except for a breach of law. (CRPF), National Security Guards (NSG) ✓ Equality before the law, that is, equal and Sashastra Seema Bal (SSB). subjection of all citizens (rich or poor, high • The ITBP was raised on 24th October, or low, official or non-official) to the 1962 during the India-China War and is a ordinary law of the land administered by border guarding police force specializing in the ordinary law courts. high altitude operations. ✓ The primacy of the rights of individual, • Presently, ITBP is deployed on border that is, the constitution is the result of the guarding duties from Karakoram Pass in rights of the individual as defined and www.iasgatewayy.com  Daily Current Affairs | Monthly 044- Magazines | Online Tests 3 | P a g e

26265326/9884421666/9884472636 UPSC OCTOBER - 2020 DAILY CURRENT AFFAIRS CLASS TEST – 30 enforced by courts of law, rather than • It is compiled and released by Office of constitution being the source of the Economic Adviser (OEA), Department for individual rights. So, answer is d. Promotion of Industry and Internal Trade (DPIIT), Ministry of Commerce & 7. Consider the following statements: Industry. So, statement 2 is not correct. 1. Index of Industrial Production is monthly • The eight infrastructure sectors constitute production volume index considered as 40.27% of total index of industrial lead indicator of monthly industrial production (IIP) with Refinery Products in performance. the top. So, statement 3 is correct. 2. It is compiled and released by Office of Economic Adviser (OEA), Central 8. In India a dam is being constructed across a Statistics Office, Ministry of Statistics and river that lies inside a national park. Who Programme Implementation. among the following are responsible for 3. The eight infrastructure sectors constitute issuing environmental clearance for the same? 40.27% of total index of industrial 1. Ministry of Environment production (IIP) with Refinery in the top. 2. National Green Tribunal Which of the statement(s) given above is/are 3. Cabinet Committee on Economic Affairs correct? 4. State Environment Impact Assessment (a) 1 only Authority (b) 1 and 2 only Select the correct answer using the codes given (c) 2 and 3 only below: (d) 1 and 3 only (a) 1 and 2 only Answer: d) (b) 2 and 3 only Explanation: (c) 3 and 4 only • It is monthly production volume index (d) 1 and 4 only considered as lead indicator of monthly Answer: d) industrial performance. It measures Explanation: performance of production in selected eight • Environmental clearance is granted by the core industries viz. Natural Gas, Coal, Ministry of Environment, and the State Crude Oil, Fertilizers, Petroleum Refinery Environment Impact Assessment Products, Steel, Cement and Electricity. Authority (SEIAA), under the So, statement 1 is correct. Environment Impact Assessment (EIA) notification of 2006. www.iasgatewayy.com  Daily Current Affairs | Monthly 044- Magazines | Online Tests 4 | P a g e

26265326/9884421666/9884472636 UPSC OCTOBER - 2020 DAILY CURRENT AFFAIRS CLASS TEST – 30 9. Right to vote and to be elected in India is a • Right to vote is only a constitutional right (a) Fundamental Right and not a fundamental right, moreover if (b) Natural Right you can find any statute which provides this (c) Constitutional Right right to the people it can then be also (d) Legal Right regarded as a legal right. Answer: c) Explanation: 10. With reference to the Gangetic Dolphin, consider the following statements: • Right to vote has been recognized under 1. The Ganges river dolphin is a fresh water Article -326 of the Constitution of India and dolphin found in India, Bangladesh, and it states that no person can be denied right to Nepal only. vote if he fulfils the need of Adult Suffrage 2. They are placed under “Critically i.e. he is above the age of 18. Endangered” category by the International • As this right has not been mentioned under Union for Conservation of Nature (IUCN). Part -III of the constitution it is not a 3. They are distributed across seven states in fundamental right, India which includes Assam, Uttar • Since, it has been mentioned under the Pradesh, Madhya Pradesh, Rajasthan, constitution of India, even though not under Bihar, Jharkhand and West Bengal. the Part - 3 of the constitution, it is merely a Which of the statement(s) given above is/are constitutional right and not a fundamental correct? right. Legal rights are the one's endowed on (a) 1 only the people from any statute enacted by the (b) 1 and 2 only legislature and has no explicit mention (c) 1 and 3 only under the Constitution. Like Right to (d) 1, 2 and 3 Information emanating from the Right to Answer: c) Information Act, 2005. Explanation: • CONCLUSION - No legal right can be • According to the World Wildlife Fund against the constitutional right, and no (WWF), the Gangetic river dolphins were constitutional right can go against the officially discovered in 1801 and are one of Fundamental rights, Thus, Fundamental the oldest creatures in the world along with rights are at the top of hierarchy, followed some species of turtles, crocodiles and by constitutional rights and lastly Legal sharks. rights.

www.iasgatewayy.com  Daily Current Affairs | Monthly 044- Magazines | Online Tests 5 | P a g e

26265326/9884421666/9884472636 UPSC OCTOBER - 2020 DAILY CURRENT AFFAIRS CLASS TEST – 30 • The Ganges river dolphin is found in parts of the Ganges-Meghna-Brahmaputra and Karnaphuli-Sangu river systems in India, Nepal, and Bangladesh. • The Gangetic river dolphin is India's National Aquatic Animal and is popularly known as ‘Susu’. • The Gangetic river dolphins can only live in freshwater, are blind and catch their prey in a unique manner, using ultrasonic sound waves. These dolphins prefer deep waters and, as per WWF, they are distributed across seven states in India: Assam, Uttar Pradesh, Madhya Pradesh, Rajasthan, Bihar, Jharkhand and West Bengal. • It is among the four freshwater dolphins in the world- the other three are: 1. The ‘Baiji’ now likely extinct from the Yangtze River in China, 2. The ‘Bhulan’ of the Indus in Pakistan, and 3. The ‘Boto’ of the Amazon River in Latin America. • These four species live only in rivers and lakes. • Its presence indicates the health of the riverine ecosystem. • Some of the efforts made to preserve and increase the numbers of these dolphins include the setting up of the Conservation Action Plan for the Gangetic Dolphin (2010-2020).

www.iasgatewayy.com  Daily Current Affairs | Monthly 044- Magazines | Online Tests 6 | P a g e

26265326/9884421666/9884472636 UPSC OCTOBER - 2020 DAILY CURRENT AFFAIRS CLASS TEST – 31 1. In context to Commission for Air Quality • It will have members from several Management in National Capital Region, Ministries as well as representatives from consider the following statements: the stakeholder States. 1. It is a statutory authority that will replace • It will also have experts from the Central non-statutory Environment Pollution Pollution Control Board (CPCB), Indian (Prevention and Control) Authority Space Research Organisation (ISRO) and (EPCA). Civil Society. 2. It will have experts from Central Pollution • In matters of air pollution and air quality Control Board (CPCB), Indian Space management, the Commission will Research Organisation (ISRO) and Civil supersede all existing bodies such as the Society. CPCB, and even the state governments of 3. It will not have the powers to issue Haryana, Punjab, Rajasthan, and Uttar Directions to the states. Pradesh. It will have the powers to issue Which of the statements given above are directions to the states. correct? 2. With reference to Institution of Eminence (a) 1 and 2 only Scheme, consider the following: (b) 2 and 3 only 1. Freedom of recruiting the faculties from (c) 1 and 3 only outside India. (d) 1, 2 and 3 2. Additional funding up to 1000 crore for Answer: a) Government Institutions. Explanation: 3. No limitation on admitting Foreign • Recently, the President of India has signed Students. ‘The Commission for Air Quality Which of the statements given above are Management in National Capital Region correct? and Adjoining Areas Ordinance, 2020’. (a) 1 and 3 only • The Ordinance sets up a statutory authority (b) 2 and 3 only - the Commission for Air Quality (c) 1 and 2 only Management in National Capital Region (d) 1, 2 and 3 (NCR) and Adjoining Areas. Answer: c) • Through the Ordinance, the Centre has also Explanation: dissolved the Environment Pollution • Recently, the OP Jindal Global University (Prevention and Control) Authority officially got an ‘Institution of Eminence (EPCA) for the NCR. www.iasgatewayy.com  Daily Current Affairs | Monthly 044- Magazines | Online Tests 1 | P a g e

26265326/9884421666/9884472636 UPSC OCTOBER - 2020 DAILY CURRENT AFFAIRS CLASS TEST – 31 (IoE) tag. The university was selected by (a) 1 only the UGC, along with 13 other higher (b) 2 only education institutions, for IoE status in (c) Both 1 and 2 2019. However, for official confirmation, (d) Neither 1 nor 2 the institution had to fulfil certain Answer: b) legislative and procedural requirements. Explanation: • Some of the benefits of being included in • The Dam Rehabilitation and Improvement Institutions of Eminence (IoEs) includes: Project (DRIP) was started in 2012 and was • Government Institutions to get additional scheduled to be completed in 2018. funding upto 1000 Crore. • In September 2018, the Government • Complete academic, administrative as well approved its revised cost estimates along as financial autonomy. with a two-year time extension from 2018 • Freedom to recruit faculty from outside to 2020, for the completion. India (limit of 25% of its faculty strength • It envisages comprehensive rehabilitation for public institutions). of 736 existing dams located across the • Freedom to admit additionally foreign country and complements the Dam Safety students on merit (maximum of 30% of the Bill, 2019. strength of admitted domestic students). • It was initially taken up for the repair and • Freedom to have the flexibility of course rehabilitation of dam projects across the structure. seven states of India, namely Jharkhand, • University Grants Commission (UGC) Karnataka, Kerala, Madhya Pradesh, inspection shall not apply to Institutions of Odisha, Tamil Nadu and Uttarakhand. Eminence. • The Drip project is jointly financed by the World Bank and the Asian Infrastructure 3. Consider the following statements, with Investment Bank (AIIB). reference to Dam Rehabilitation and

Improvement Project: 4. With reference to the differences between the 1. It envisages the repair and rehabilitation of National Green Tribunal (NGT) and the dam projects in North-Eastern states only. Central Pollution Control Board (CPCB), 2. It is being funded by the World Bank and consider the following statements: the Asian Infrastructure Investment Bank 1. The NGT has been established by an Act (AIIB). whereas the CPCB has been created by an Which of the above statements is/are correct? executive order of the Government. www.iasgatewayy.com  Daily Current Affairs | Monthly 044- Magazines | Online Tests 2 | P a g e

26265326/9884421666/9884472636 UPSC OCTOBER - 2020 DAILY CURRENT AFFAIRS CLASS TEST – 31 2. The NGT provides environmental justice in the form of members of the General and helps reduce the burden of litigation in Body in the working of NAFED. the higher courts whereas the CPCB Which of the statements given above are promotes cleanliness of streams and wells, correct? and aims to improve the quality of air in the (a) 1 and 2 only country. (b) 2 and 3 only Which of the statement(s) given above is/are (c) 1 and 3 only correct? (d) 1, 2 and 3 (a) 1 only Answer: c) (b) 2 only Explanation: (c) Both 1 and 2 • National Agricultural Cooperative (d) Neither 1 nor 2 Marketing Federation of India Ltd. Answer: b) (NAFED), established in 1958, is Explanation: registered under the Multi State Co- • The NGT was established by the National operative Societies Act. So, statement 1 is Green Tribunal Act, 2010 whereas the correct. CPCB has been created by the Water act, • NAFED was setup with the object to 1974. So, statement 1 is not correct. promote Co-operative marketing of Agricultural Produce to benefit the 5. Consider the following statements regarding farmers not consumers. So, statement 2 NAFED: is not correct. 1. National Agricultural Cooperative • Agricultural farmers are the main members Marketing Federation of India Ltd. of NAFED, who have the authority to say (NAFED), established in 1958, is in the form of members of the General registered under the Multi State Co- Body in the working of NAFED. So, operative Societies Act. statement 3 is correct. 2. NAFED was setup with the object to promote Co-operative marketing of 6. Consider the following statements with respect Agricultural Produce to benefit the to Intersex Children: consumers. 1. India became only the third country after 3. Agricultural farmers are the main members Malta and Taiwan, to have a legal regime of NAFED, who have the authority to say which protects the rights of intersex children. www.iasgatewayy.com  Daily Current Affairs | Monthly 044- Magazines | Online Tests 3 | P a g e

26265326/9884421666/9884472636 UPSC OCTOBER - 2020 DAILY CURRENT AFFAIRS CLASS TEST – 31 2. Intersex refers to people born with • invasive and irreversible medical surgeries biological or physical characteristics that and other medical treatment on intersex are more diverse than stereotypical male or children. female bodies. • While the term intersex is confused with 3. Individuals who identify as transgender or transgender, the two in-fact have very transsexual have a gender that is as same as different meanings. of the one traditionally associated with the • Individuals who identify as transgender or sex they were Assigned at Birth. transsexual have a gender that is different Which of the statements given above are not from the one traditionally associated with correct? the sex they were assigned at birth. (a) 1 and 2 only • Intersex refers to people born with (b) 2 and 3 only biological or physical characteristics that (c) 1 and 3 only are more diverse than stereotypical male or (d) 1, 2 and 3 female bodies. Answer: c) Explanation: 7. As per the ‘India State of Forest Report (ISFR), • The demand for a nation-wide ban on 2019’ arrange the states in the order of unnecessary medical surgeries conducted decreasing Forest Cover performance in India: on children with intersex traits comes 1. Andhra Pradesh months after the Tamil Nadu government 2. Karnataka banned normative surgeries on infants and 3. Kerala children except in life-threatening 4. Himachal Pradesh situations after a historic judgment of the Select the correct answer using the code given Madras High Court on April 22, 2019. below: • If adopted nationally, India could become (a) 2-1-3-4 only the third country after Malta and (b) 1-2-3-4 Taiwan, to have a legal regime which (c) 4-3-1-2 protects the rights of intersex children. (d) 4-3-2-1 India still not yet adopted. Answer: a) • The World Health Organization and the Explanation: United Nations Human Rights Council • The total forest cover of the country is 7, have called upon Member States to end 12,249 sq.km which is 21.67% of the geographical area of the country. The tree www.iasgatewayy.com  Daily Current Affairs | Monthly 044- Magazines | Online Tests 4 | P a g e

26265326/9884421666/9884472636 UPSC OCTOBER - 2020 DAILY CURRENT AFFAIRS CLASS TEST – 31 cover of the country is estimated as 95,027 Answer: a) sq.km which is 2.89% of the geographical Explanation: area. • The eight core sector industries include • The total Forest and Tree cover of the coal, crude oil, natural gas, refinery country is 8, 07,276 sq.km which is 24.56% products, fertilizer, steel, cement and of the geographical area of the country. electricity. • The current assessment shows an increase • Eight Core Industries in decreasing order of of 3,976 sq.km (0.56%) of forest cover, their weightage in the Index of Industrial 1,212 sq.km (1.29%) of tree cover and Production: Refinery Products> 5,188 sq.km (0.65%) of forest and tree Electricity> Steel> Coal> Crude Oil> cover put together, at the national level as Natural Gas> Cement> Fertilizers. compared to the previous assessment i.e. 9. Consider the following statements: ISFR 2017. 1. The excess of the government’s total • Forest cover within the Recorded Forest expenditure over its total non-borrowed Area/Green Wash (RFA/GW) has shown a receipts is called the fiscal deficit. slight decrease of 330 sq.km (0.05%) 2. The primary deficit is the fiscal deficit whereas there is an increase of 4,306 sq.km Minus Interest Payments. of forest cover outside the RFA/GW as 3. The FRBM Act suggests bringing the fiscal compared to previous assessment of 2017.

deficit down to about 4 percent of the GDP 8. Arrange the eight Core Industries in decreasing as the ideal target for India. order of their weightage in the Index of Which of the given statement/s is/are Industrial Production: Incorrect? 1. Electricity (a) 1 only 2. Fertilizers (b) 1 and 3 only 3. Natural Gas (c) 3 only 4. Crude oil (d) 2 only 5. Steel Answer: c) Choose the correct option: Explanation: (a) 1, 5, 4, 3, 2 • When the government’s non-borrowed (b) 1, 4, 3, 5, 2 receipts fall short of its entire expenditure, (c) 5, 1, 4, 2, 3 it has to borrow money from the public to (d) 4, 2, 1, 5, 3 meet the Shortfall. www.iasgatewayy.com  Daily Current Affairs | Monthly 044- Magazines | Online Tests 5 | P a g e

26265326/9884421666/9884472636 UPSC OCTOBER - 2020 DAILY CURRENT AFFAIRS CLASS TEST – 31 • The excess of the government’s total expenditure over its total non-borrowed receipts is called the fiscal deficit. • The primary deficit is the fiscal deficit minus interest payments. It tells how much of the government’s borrowings are going towards meeting expenses other than interest payments. • The FRBM Act suggests that bringing the fiscal deficit down to about 3 percent of the GDP is the ideal target for India.

10. Consider the following statements with respect to River Mahanadi: 1. It originates in Odisha. 2. It is an east-flowing river draining into the Bay of Bengal. 3. The Hirakud dam is built across this river. Which of the given statement/s is/are correct? (a) 1 and 2 only (b) 2 and 3 only (c) 1, 2 and 3 (d) 3 only Answer: b) Explanation: • The river Mahanadi originates in Chhattisgarh. • It is an east-flowing river draining into the Bay of Bengal. • The Hirakud dam is built across the river Mahanadi.

www.iasgatewayy.com  Daily Current Affairs | Monthly 044- Magazines | Online Tests 6 | P a g e

26265326/9884421666/9884472636